दृष्टि आईएएस अब इंदौर में भी! अधिक जानकारी के लिये संपर्क करें |   अभी कॉल करें
ध्यान दें:

प्रारंभिक परीक्षा


प्रारंभिक परीक्षा

प्रारंभिक परीक्षा, 2020: एक विश्लेषणात्मक अध्ययन

  • 05 Oct 2020
  • 249 min read

प्रश्न-पत्र-I

सिविल सेवा परीक्षा, 2020 का सामान्य अध्ययन प्रश्न-पत्र सिविल सेवा परीक्षा के इतिहास में कठिनतम प्रश्न-पत्रों में से एक माना गया है। कृषि क्षेत्र पर पूछे गए प्रश्नों की प्रकृति को देखते हुए यह माना जा रहा है कि इस वर्ष के प्रश्न-पत्र की कठिनाई का स्तर पिछले वर्ष के प्रश्न-पत्र से अधिक था। इसके निम्नलिखित कारण हो सकते है:

  • प्रश्नों की प्रकृति में परिवर्तन
  • विषयवार प्रश्नों के भारांश में परिवर्तन
  • प्रश्नों का अस्पष्ट स्वरुप

1. निम्नलिखित कथनों पर विचार कीजिये:

  1. ‘आधार’ मेटाडेटा को तीन महीने से अधिक संग्रहित नहीं रखा जा सकता है। 
  2. राज्य निजी निगमों (Corporations) से ‘आधार’ डेटा को साझा करने के लिये कोई अनुबंध नहीं कर सकता।
  3. ‘आधार’ बीमा उत्पादों को प्राप्त करने के लिये अनिवार्य है।
  4. ‘आधार’ भारत की संचित निधि से हित/लाभ प्राप्त करने के लिये अनिवार्य है।

उपर्युक्त कथनों में से कौन-सा/से सही है/हैं?

(a) केवल 1 और 4

(b) केवल 2 और 4 

(c) केवल 3

(d) केवल 1, 2 और 3

उत्तर : (b) 

व्याख्या

  • सितंबर, 2018 के सर्वोच्च न्यायालय के फैसले के अनुसार, आधार मेटाडेटा को छः महीने से अधिक समय तक संग्रहीत नहीं किया जा सकता है। सुप्रीम कोर्ट ने आधार अधिनियम की धारा 2(d) को रद्द कर दिया है, जो सरकारी अधिकारियों को लेन-देन के मेटाडेटा को संग्रहीत करने से रोकने के लिये पाँच वर्षों की अवधि के लिये ऐसे डेटा के संग्रहण की अनुमति देता है। अतः कथन 1 सही नहीं है।
  • सर्वोच्च न्यायालय ने आधार विनियम 26(c) को भी रद्द कर दिया है, जिसने भारतीय विशिष्ट पहचान प्राधिकरण (Unique Identification Authority of India -UIDAI) को निजी फर्मों के लिये आधार आधारित प्रमाणीकरण अथवा प्रमाणीकरण इतिहास से संबंधित मेटाडेटा को संग्रहीत करने की अनुमति दी थी। तदनुसार, भारतीय बीमा विनियामक एवं विकास प्राधिकरण (Insurance Regulatory and Development Authority of India -IRDAI) ने बीमा कंपनियों को निर्देश दिया है कि वे ग्राहक को जानें (Know Your Customer - KYC) संबंधी आवश्यकताओं के लिये अनिवार्य रूप से आधार विवरण न मांगें या UIDAI से E-KYC का उपयोग करके प्रमाणीकरण न करें। अतः कथन 2 सही है और कथन 3 सही नहीं है।
  • इसके अतिरिक्त, आधार (वित्तीय एवं अन्य सब्सिडी, लाभ एवं सेवाओं का लक्षित वितरण) अधिनियम, 2016 की धारा 7 में किये गए संशोधन को बरकरार रखा गया है। यह एक शर्त निर्धारित करता है कि राज्य सरकार सब्सिडी, लाभ या सेवा प्राप्त करने के लिये लाभार्थियों के आधार प्रमाणीकरण के उपयोग को अनिवार्य कर सकती है, जिसके लिये भारत के समेकित कोष से व्यय किया जाता है। अतः कथन 4 सही है।

https://www.drishtiias.com/hindi/daily-updates/daily-news-editorials/aadhaar-counting-the-real-problems

2. राज्य सभा की लोक सभा के समान शक्तियाँ किस क्षेत्र में हैं?

(a) नई अखिल भारतीय सेवाएँ गठित करने के विषय में

(b) संविधान में संशोधन करने के विषय में

(c) सरकार को हटाने के विषय में

(d) कटौती प्रस्ताव प्रस्तुत करने के विषय में

उत्तर : (b) 

व्याख्या

राज्यसभा को लोकसभा की तुलना में कुछ विशेष शक्तियाँ प्राप्त हैं जो इस प्रकार हैं:

  • एक निर्दिष्ट अवधि के लिये एक विषय को राज्य सूची से संघ सूची में स्थानांतरित करने की शक्ति (अनुच्छेद 249)।
  • अतिरिक्त अखिल भारतीय सेवाएँ सृजित करना (अनुच्छेद 312)।
  • जब लोकसभा भंग रहती है तो सीमित अवधि के लिये अनुच्छेद 352 के अंतर्गत आपातकाल का समर्थन करना।अत: विकल्प (a) सही नहीं है।
  • ऐसे महत्त्वपूर्ण मामले जिनके संबंध में दोनों सदनों को समान शक्तियाँ प्राप्त हैं, उनमें राष्ट्रपति का चुनाव और महाभियोग, उपराष्ट्रपति का चुनाव, संविधान में संशोधन, आपातकाल की घोषणा को मंजूरी, राज्यों में संवैधानिक तंत्र की विफलता और वित्तीय आपातकाल की घोषणा आदि शामिल है। अत: विकल्प (b) सही है।
  • मंत्रिपरिषद सामूहिक रूप से लोकसभा के प्रति उत्तरदायी होती है, जिसका अर्थ है कि सरकार तब तक अस्तित्त्व में रहती है जब तक उसे लोकसभा के अधिकांश सदस्यों का विश्वास प्राप्त रहता है। अतः विकल्प (c) सही नहीं है।
  • संसद में बजट पेश किये जाने के बाद उस पर सामान्य चर्चा होती है तत्पश्चात् लोकसभा विभागानुसार ‘अनुदान की मांगों’ पर चर्चा करती है और उन्हें स्वीकृति देती है। लेकिन ‘अनुदान की मांगों’ (Demands For Grants) पर चर्चा के दौरान यदि कोई सदस्य चाहता है कि बजट में किसी विभाग के लिये आवंटित राशि में कटौती की जाए तो वह सदस्य एक नोटिस देकर इस संबंध में प्रस्ताव पेश कर सकता है, इस प्रस्ताव को कटौती प्रस्ताव या Cut Motion कहते हैं। यदि कटौती प्रस्ताव स्वीकृत हो जाता है, तो यह एक अविश्वास मत के बराबर होता है और यदि सरकार निचले सदन में संख्याओं को जोड़ने में विफल रहती है, तो वह सदन के मानदंडों के अनुसार इस्तीफा देने के लिए बाध्य होती है। अतः विकल्प (d) सही नहीं है।

अतः विकल्प (b) सही उत्तर है।

https://www.drishtiias.com/hindi/daily-updates/daily-news-editorials/the-need-for-a-second-chamber

3. संसद सदस्य स्थानीय क्षेत्र विकास योजना (MPLADS) के अंतर्गत निधियों के संदर्भ में निम्नलिखित कथनों में से कौन-से सही हैं?

  1. MPLADS निधियाँ टिकाऊ परिसंपत्तियों जैसे स्वास्थ्य, शिक्षा, आदि की भौतिक आधारभूत संरचनाओं के निर्माण में ही प्रयुक्त हो सकती हैं।
  2. प्रत्येक सांसद की निधि का एक निश्चित अंश अनुसूचित जाति/जनजाति जनंसख्या के लाभार्थ प्रयुक्त होना आवश्यक है।
  3. MPLADS निधियाँ वार्षिक आधार पर स्वीकृत की जाती हैं और अप्रयुक्त निधि को अगले वर्ष के लिये अग्रेनीत नहीं किया जा सकता।
  4. कार्यान्वित हो रहे सभी कार्यों में से कम-से-कम 10 प्रतिशत कार्यों का ज़िला प्राधिकारी द्वारा प्रति वर्ष निरीक्षण अनिवार्य है।

नीचे दिये गए कूट का प्रयोग कर सही उत्तर चुनिये-

(a) केवल 1 और 2

(b) केवल 3 और 4

(c) केवल 1, 2 और 3

(d) केवल 1, 2 और 4

उत्तर : (d) 

व्याख्या

  • संसद सदस्य स्थानीय क्षेत्र विकास योजना (MPLADS) वर्ष 1993 में बनाई गई थी ताकि सांसद टिकाऊ सामुदायिक परिसंपत्तियों के निर्माण पर ज़ोर देने के साथ अपने निर्वाचन क्षेत्र में प्रति वर्ष 5 करोड़ की राशि तक के कार्यों के लिये ज़िला कलेक्टर को सिफारिश कर सकें। स्थानीय रूप से महसूस की गई आवश्यकता के आधार पर टिकाऊ परिसंपत्तियों में पीने का पानी, प्राथमिक शिक्षा, सार्वजनिक स्वास्थ्य, स्वच्छता और सड़कें आदि शामिल हैं। अतः कथन 1 सही है।
  • 5 करोड़ की राशि में से, एक सांसद अनुसूचित जाति की आबादी वाले क्षेत्रों के लिये कम से कम 15% और अनुसूचित जनजाति की आबादी वाले क्षेत्रों के लिये 7.5% की सिफारिश कर सकेंगे। अतः कथन 2 सही है।
  • MPLADS के अंतर्गत सांसदों को 2.5 करोड़ रुपए की दो किस्तों में 5 करोड़ रुपये मिलते हैं। MPLADS के अंतर्गत  निधियाँ गैर व्यपगत होती हैं। अर्थात यह निधियाँ वार्षिक आधार पर स्वीकृत की जाती हैं और अप्रयुक्त निधि को अगले वर्ष के लिये अग्रेनीत किया जा सकता है। अतः कथन 3 सही नहीं है।
  • कार्यान्वित हो रहे सभी कार्यों में से कम-से-कम 10 प्रतिशत कार्यों का ज़िला प्राधिकारी द्वारा प्रति वर्ष निरीक्षण अनिवार्य है। अतः कथन 4 सही है।

अतः विकल्प (d) सही उत्तर है।

https://www.drishtiias.com/hindi/daily-updates/daily-news-analysis/pm-mp-to-take-30-salary-cut

https://www.drishtiias.com/hindi/printpdf/mp-funds-and-challenges

4. निम्नलिखित मूल अधिकारों के किस संवर्ग में अस्पृश्यता के रूप में किये गए विभेदन के विरुद्ध संरक्षण समाविष्ट है?

(a) शोषण के विरुद्ध अधिकार

(b) स्वतंत्रता का अधिकार

(c) सांविधानिक उपचार का अधिकार

(d) समता का अधिकार

उत्तर : (d) 

व्याख्या

  • भारतीय संविधान के तहत मौलिक अधिकारों की छ: श्रेणियाँ हैं:
    • समानता का अधिकार (अनुच्छेद 14-18)
    • स्वतंत्रता का अधिकार (अनुच्छेद 19-22)
    • शोषण के विरुद्ध अधिकार (अनुच्छेद 23-24)
    • धर्म की स्वतंत्रता का अधिकार (अनुच्छेद 25-28)
    • सांस्कृतिक और शैक्षिक का अधिकार (अनुच्छेद 29-30)
    • संवैधानिक उपचार का अधिकार (अनुच्छेद 32)
  • समानता के अधिकार के तहत अनुच्छेद 17 (अनुच्छेद 14-18), अस्पृश्यता के उन्मूलन और इसके अभ्यास पर रोक लगाने की बात करता है।
  • यह 'अस्पृश्यता' को समाप्त करता है और किसी भी रूप में इसके अभ्यास पर रोक लगाता है। अस्पृश्यता से उत्पन्न होने वाली किसी भी अक्षमता का प्रवर्तन कानून के अनुसार दंडनीय अपराध होगा। अत: विकल्प (d) सही उत्तर है।

https://www.drishtiias.com/hindi/daily-updates/daily-news-editorials/the-need-for-an-anti-discrimination-law

5. भारत में, न्यायपालिका का कार्यपालिका से पृथक्करण, किससे व्यादेशित है?

(a) संविधान की उद्देशिका द्वारा

(b) राज्य की नीति के निदेशक तत्त्व द्वारा

(c) सातवीं अनुसूची द्वारा

(d) परंपरागत व्यवहार द्वारा

उत्तर : (b) 

व्याख्या

  • राज्य के नीति निदेशक सिद्धांतों को संविधान के भाग IV (अनुच्छेद 36 से 51) में वर्णित किया गया है।
  • संविधान के अनुच्छेद 50 में न्यायपालिका का कार्यपालिका से पृथक्करण का प्रावधान किया गया है तथा राज्य की सार्वजनिक सेवाओं में न्यायपालिका को कार्यपालिका से अलग करने के लिये राज्य कदम उठाएगा। अत: विकल्प (b) सही उत्तर है।

https://www.drishtiias.com/hindi/current-affairs-news-analysis-editorials/news-editorials/20-11-2018

6. वित्त मंत्री संसद में बजट प्रस्तुत करते हुए उसके साथ अन्य प्रलेख भी प्रस्तुत करते हैं जिनमें वृहद आर्थिक रूपरेखा विवरण (The Macro Economic Framework Statement) भी सम्मिलित रहता है। यह पूर्वोक्त प्रलेख निम्न आदेशन के कारण प्रस्तुत किया जाता है:

(a) चिरकालिक संसदीय परंपरा के कारण

(b) भारत के संविधान के अनुच्छेद 112 तथा अनुच्छेद 110 (1) के कारण

(c) भारत के संविधान के अनुच्छेद 113 के कारण

(d) राजकोषीय उत्तरदायित्व एवं बजट प्रबंधन अधिनियम, 2003 के प्रावधानों के कारण

उत्तर : (d) 

व्याख्या

  • भारतीय संविधान के अनुच्छेद 112 के अनुसार, एक वर्ष के केंद्रीय बजट को वार्षिक वित्तीय विवरण (AFS) कहा जाता है। यह एक वित्तीय वर्ष में सरकार की अनुमानित प्राप्तियों और व्यय का विवरण है। 
  • 'मनी बिल' को अनुच्छेद 110(1) के तहत परिभाषित किया गया है।
  • अनुच्छेद 113 के अनुसार, भारत की संचित निधि से व्यय का अनुमान (अनुदान की मांगों के रूप में) वार्षिक वित्तीय विवरण में शामिल किया जाना चाहिये और यह लोकसभा द्वारा मतदान के लिये आवश्यक है।
  • राजकोषीय उत्तरदायित्व और बजट प्रबंधन (FRBM) अधिनियम, 2003 वित्तीय अनुशासन को संस्थागत बनाने, भारत के राजकोषीय घाटे को कम करने, व्यापक आर्थिक प्रबंधन में सुधार करने और एक संतुलित बजट की ओर बढ़ते हुए सार्वजनिक निधियों का समग्र प्रबंधन करने का प्रावधान करता है।
  • यह अधिनियम केंद्र सरकार को वार्षिक वित्तीय विवरण और अनुदान मांगों के साथ संसद के सदनों, व्यष्टि अर्थशास्त्रीय संरचना विवरण (मैक्रो-इकोनॉमिक फ्रेमवर्क स्टेटमेंट), मध्यम अवधि राजकोषीय नीति विवरण तथा राजकोषीय नीति रणनीति विवरण को सदनों के समक्ष प्रस्तुत करने के लिये बाध्य करता है। अत: विकल्प (d) सही उत्तर है।

https://www.drishtiias.com/hindi/daily-updates/daily-news-analysis/fiscal-deficit-target-and-covid-19

7. परिभाषा से, संवैधानिक सरकार का अर्थ है-

(a) विधान मंडल द्वारा सरकार

(b) लोकप्रिय सरकार

(c) बहु-दलीय सरकार

(d) सीमित सरकार

उत्तर : (d) 

व्याख्या

  • एक संवैधानिक सरकार को एक संविधान के अस्तित्त्व से परिभाषित किया जाता है जो लिखित या अलिखित हो सकता है तथा जिसे राजनीति के मौलिक कानून के रूप में स्वीकार किया जाता है और जो प्रभावी रूप से राजनीतिक शक्ति के प्रयोग को नियंत्रित करता है।
  • संवैधानिक सरकार का मुख्य घटक "कानून के नियम" या "बुनियादी कानूनों" का एक समूह है जो सार्वजनिक कार्यालय-धारकों और समाज के सभी सदस्यों (अर्थात् नागरिकों) को एक निश्चित क्षेत्र के भीतर बाँधता है।
  • संवैधानिकता का सार इन शक्तियों को संतुलित करने में एक विशेष दृष्टिकोण के साथ सरकार की कार्यकारी, विधायी और न्यायिक शाखाओं के बीच सत्ता के वितरण द्वारा शक्ति का नियंत्रण है।
  • संवैधानिक सरकार एक प्रकार का शासन है जो इस तथ्य की विशेषता है कि सरकार कानूनी और संस्थागत बाधाओं के एक समूह में कार्य करती है जो दोनों अपनी शक्ति (सीमित सरकार) को सीमित करती है और व्यक्तिगत स्वतंत्रता की रक्षा करती है।
  • एक संवैधानिक सरकार के पास एक द्विदलीय सरकार हो सकती है लेकिन विधायिका नहीं हो सकती है और इसे एक लोकप्रिय सरकार होने की आवश्यकता नहीं है. यह राजशाही भी हो सकती है। अत: विकल्प (d) सही है।

https://www.drishtiias.com/hindi/daily-updates/daily-news-editorials/the-four-phases-of-constitutional-interpretation

8. मूल अधिकारों के अतिरिक्त, भारत के संविधान का निम्नलिखित में से कौन-सा/से भाग मानव अधिकारों की सार्वभौम घोषणा 1948 (Universal Declaration of Human Rights 1948) के सिद्धांतों एवं प्रावधानों को प्रतिबिंबित करता/करते है/हैं?

  1. उद्देशिका
  2. राज्य की नीति के निदेशक तत्त्व
  3. मूल कर्त्तव्य

नीचे दिये गए कूट का प्रयोग कर सही उत्तर चुनिये-

(a) केवल 1 और 2

(b) केवल 2

(c) केवल 1 और 3

(d) 1, 2 और 3

उत्तर : (d) 

व्याख्या

  • वर्ष 1948 में संयुक्त राष्ट्र महासभा (UNGA) द्वारा अपनाई गई और घोषित मानव अधिकारों की सार्वभौम घोषणा (UDHR), प्रत्येक व्यक्ति की समानता और गरिमा को स्थापित करती है तथा यह निर्धारित करती है कि हर सरकार का एक मुख्य कर्त्तव्य है कि वह सभी नागरिकों तक इनकी पहुँच को सक्षम बनाए। उनके अविच्छेद्य अधिकार और स्वतंत्रता।
  • प्रस्तावना: प्रस्तावना के उद्देश्य जैसे न्याय (सामाजिक, आर्थिक और राजनीतिक), समानता और स्वतंत्रता भी UDHR के सिद्धांतों को दर्शाते हैं। अतः कथन 1 सही है।
  • राज्य के नीति निदेशक सिद्धांत (DPSP): अनुच्छेद 36 से 51 के तहत प्रदान किए गए, DPSP ऐसे सिद्धांत हैं जिनका उद्देश्य सामाजिक और आर्थिक न्याय प्रदान करना है तथा कल्याणकारी राज्य की दिशा में मार्ग निर्धारित करना है। ये DPSP राज्य पर दायित्व के रूप में कार्य करते हैं और मानवाधिकारों के अनुरूप हैं। कुछ DPSP जो मानवाधिकारों के साथ तालमेल बिठाते हैं, वे इस प्रकार हैं:
  • अनुच्छेद 38: कल्याणकारी राज्य को बढ़ावा देना
  • अनुच्छेद 39: असमानताओं को कम करना
  • अनुच्छेद 39ए: निशुल्क कानूनी सहायता
  • अनुच्छेद 41: समाज के कमज़ोर वर्गों जैसे बेरोज़गार, बीमार, विकलांग और वृद्ध व्यक्तियों की सहायता करना।
  • अनुच्छेद 43: निर्वाह मज़दूरी की सुरक्षा। अतः कथन 2 सही है।
  • मौलिक कर्त्तव्य (अनुच्छेद 51ए): ये मूल रूप से भारत के सभी नागरिकों के नागरिक और नैतिक दायित्व हैं। अभी तक, भारत में 11 मौलिक कर्त्तव्य हैं, जो संविधान के भाग IV ए में हैं। अनुच्छेद 51ए (के) माता-पिता या अभिभावक द्वारा 6 से 14 वर्ष की आयु के बच्चे को शिक्षा के अवसर प्रदान करने के बारे में बात करता है। यह पहलू किसी तरह शिक्षा के माध्यम से बच्चों की गरिमा सुनिश्चित करने से संबंधित है। अतः कथन 3 सही है। अतः विकल्प (d) सही है।

https://www.drishtiias.com/hindi/important-institution/national-organization/national-human-rights-commission 

https://www.drishtiias.com/hindi/burning-issues-of-the-month/human-rights-in-india  

9. भारत में, विधिक सेवा प्रदान करने वाले प्राधिकरण (Legal Services Authorities) निम्नलिखित में से किस प्रकार के नागरिकों को नि:शुल्क विधिक सेवाएँ प्रदान करते हैं?

  1. 1,00,000 रुपए से कम वार्षिक आय वाले व्यक्ति को
  2. 2,00,000 रुपए से कम वार्षिक आय वाले ट्रांसजेंडर को
  3. 3,00,000 रुपए से कम वार्षिक आय वाले अन्य पिछडे़ वर्ग (OBC) के सदस्य को
  4. सभी वरिष्ठ नागरिकों को

नीचे दिये गए कूट का प्रयोग कर सही उत्तर चुनिये-

(a) केवल 1 और 2 

(b) केवल 3 और 4

(c) केवल 2 और 3

(d) केवल 1 और 4

उत्तर : (a) 

व्याख्या

  • 1987 में, विधिक सेवा प्राधिकरण अधिनियम संसद द्वारा अधिनियमित किया गया था जो 9 नवंबर, 1995 को समान अवसर के आधार पर समाज के कमज़ोर वर्गों को सक्षम कानूनी सेवाएँ प्रदान करने के लिये एक राष्ट्रव्यापी समान नेटवर्क स्थापित करने के लिये लागू हुआ था।
  • अधिनियम की धारा 12 के तहत, निशुल्क कानूनी सेवाएँ प्राप्त करने के पात्र व्यक्तियों में शामिल हैं:
    • महिलाएँ और बच्चे
    • अनुसूचित जाति / अनुसूचित जनजाति के सदस्य
    • औद्योगिक कामगार
    • आपदा, हिंसा, बाढ़, सूखा, भूकंप के शिकार
    • दिव्यांग व्यक्ति
    • हिरासत में लोग
    • मानव तस्करी या बेगार के शिकार
  • इसके अलावा, अधिनियम की धारा 12(h) के अनुसार, जिन व्यक्तियों की वार्षिक आय 9000 रुपए से कम है या ऐसी अन्य उच्च राशि जो राज्य सरकार द्वारा निर्धारित की जा सकती है यदि मामला सर्वोच्च न्यायालय के अलावा किसी अन्य अदालत के समक्ष है तथा 12000 रुपए से कम या ऐसी अन्य उच्च राशि जो केंद्र सरकार द्वारा निर्धारित की जा सकती है यदि मामला सर्वोच्च न्यायालय के समक्ष है तो वे भी निशुल्क कानूनी सेवाओं के लिये पात्र हैं। अधिनियम की धारा 12(h) के तहत निर्धारित आय सीमा, विभिन्न राज्यों में अलग-अलग है। यह लक्षद्वीप में 9000 रुपए से लेकर आंध्र प्रदेश में 3,00,000 रुपए तक है। अतः कथन 1 सही है।
  • निशुल्क कानूनी सहायता के लिये वरिष्ठ नागरिकों की पात्रता इस संबंध में संबंधित राज्य सरकारों द्वारा बनाए गए नियमों पर निर्भर करती है। इसलिये सभी वरिष्ठ नागरिक निशुल्क कानूनी सहायता के पात्र नहीं हैं। अतः कथन 4 सही नहीं है।
  • अन्य पिछड़ा वर्ग (OBC) के लिये इस तरह का कोई स्पष्ट सीमांकन नहीं है यानी क्या OBC का कोई सदस्य जिसकी वार्षिक आय 3,00,000 रुपए से कम है, अधिनियम के तहत निशुल्क कानूनी सहायता प्राप्त करेगा। अतः कथन 3 सही नहीं है।
  • अत: विकल्प (a) सही उत्तर है।

https://www.drishtiias.com/hindi/daily-updates/daily-news-analysis/national-legal-service-day

10. निम्नलिखित युग्मों पर विचार कीजिये-

अंतर्राष्ट्रीय समझौता/संगठन      विषय

  1. अल्मा-आटा घोषणा           : लोगों के स्वास्थ्य की देखभाल
  2. हेग समझौता                  : जैविक एवं रासायनिक शस्त्र
  3. तलानोआ संवाद               : वैश्विक जलवायु परिवर्तन
  4. अंडर2 गठबंधन               : बाल अधिकार

उपर्युक्त में से कौन-सा/से युग्म सही सुमेलित है/हैैं?

(a) केवल 1 और 2

(b) केवल 4

(c) केवल 1 और 3

(d) केवल 2, 3 और 4 

उत्तर : (c) 

व्याख्या

  • अल्मा-आटा घोषणा: इसे वर्ष 1978 में अल्माटी, कज़ाखस्तान में आयोजित प्राथमिक स्वास्थ्य देखभाल (PHC) पर अंतर्राष्ट्रीय सम्मेलन में अपनाया गया था। इसने सभी सरकारों, स्वास्थ्य देखभाल श्रमिकों तथा विकास कार्यकर्त्ताओं से सभी लोगों के प्राथमिक स्वास्थ्य को बढ़ावा देने और उनकी रक्षा करने का आग्रह किया। अतः युग्म 1 सही सुमेलित है।
  • हेग समझौता: विभिन्न विषयों पर हेग समझौते की एक शृंखला है जैसे सशस्त्र संघर्ष की स्थिति में सांस्कृतिक संपत्ति के संरक्षण के लिये समझौता, अंतर्राष्ट्रीय बाल अपहरण के नागरिक पहलुओं पर हेग समझौता आदि। लेकिन यह जैविक और रसायनिक शस्त्र से संबंधित नहीं है। अतः युग्म 2 सही सुमेलित नहीं है।
  • तलानोआ संवाद: इस संवाद को वर्ष 2017 में बॉन (जर्मनी) में संयुक्त राष्ट्र जलवायु परिवर्तन सम्मेलन (COP 23) में लॉन्च किया गया था। तलानोआ एक पारंपरिक शब्द है जिसका उपयोग फिजी और प्रशांत क्षेत्र में समावेशी, भागीदारी और पारदर्शी संवाद की प्रक्रिया को दर्शाने के लिये किया जाता है। तलानोआ का उद्देश्य कहानियों को साझा करना, सहानुभूति का निर्माण करना तथा सामूहिक कल्याण के लिये विवेकपूर्ण निर्णय लेना है। अतः युग्म 3 सही सुमेलित है।
  • अंडर2 गठबंधन: अंडर2 गठबंधन राज्य और क्षेत्रीय सरकारों का एक वैश्विक समुदाय है जो पेरिस समझौते के अनुरूप महत्त्वाकांक्षी जलवायु कार्रवाई के लिये प्रतिबद्ध है। गठबंधन 220 से अधिक उप-राष्ट्रीय सरकारों को एक साथ लाता है जो 1.3 बिलियन से अधिक लोगों और वैश्विक अर्थव्यवस्था के 43% का प्रतिनिधित्व करते हैं। वर्तमान में, तेलंगाना और छत्तीसगढ़ भारत से इस समझौते के हस्ताक्षरकर्त्ता हैं। हस्ताक्षरकर्त्ता 1.5 डिग्री सेल्सियस तक पहुँचने के प्रयासों के साथ वैश्विक तापमान वृद्धि को 2 डिग्री सेल्सियस से नीचे रखने के लिये प्रतिबद्ध हैं। अतः युग्म 4 सही सुमेलित नहीं है।

अत: विकल्प (c) सही उत्तर है।

https://www.drishtiias.com/hindi/daily-updates/daily-news-analysis/lessons-from-kerala

https://www.drishtiias.com/hindi/daily-updates/daily-news-editorials/importance-of-the-hague-convention-and-arguments-of-india-disagreement

https://www.drishtiias.com/hindi/daily-updates/daily-news-editorials/why-india-is-not-ready-to-ratify-the-hague-convention

https://www.drishtiias.com/hindi/daily-updates/daily-news-analysis/need-for-consolidation-of-efforts-in-climate-governance

11. संसदीय व्यवस्था वाली सरकार वह होती है, जिसमें 

(a) संसद के सभी राजनीतिक दलों का सरकारों में प्रतिनिधित्व होता है।

(b) सरकार संसद के प्रति उत्तरदायी होती है और उसके द्वारा हटाई जा सकती है।

(c) सरकार लोगों के द्वारा निर्वाचित होती है और उनके द्वारा हटाई जा सकती है।

(d) सरकार संसद के द्वारा चुनी जाती है किंतु निर्धारित समयावधि के पूर्ण होने के पूर्व हटाई नहीं जा सकती।

उत्तर : (b) 

व्याख्या

  • सरकार की संसदीय प्रणाली वह है जिसमें सरकार संसद के प्रति उत्तरदायी होती है और इसके द्वारा उसे हटाया जा सकता है। ऐसी व्यवस्था में, राष्ट्रपति या राजा की भूमिका मुख्य रूप से औपचारिक होती है और कैबिनेट के साथ-साथ प्रधानमंत्री प्रभावी शक्ति का इस्तेमाल करते हैं।
  • भारत के संविधान के अनुच्छेद 75 (3) के अनुसार, मंत्रिपरिषद सामूहिक रूप से लोकसभा के प्रति उत्तरदायी है जो संसद का एक घटक है। लोकसभा के नियम इस सामूहिक उत्तरदायित्त्व के परीक्षण के लिए एक तंत्र प्रदान करते हैं। वे किसी भी लोकसभा सांसद को मंत्रिपरिषद के खिलाफ अविश्वास प्रस्ताव पेश करने की अनुमति देते हैं, जो 50 सहयोगियों का समर्थन हासिल कर सकता है। यदि लोकसभा में अविश्वास प्रस्ताव पारित हो जाता है तो सरकार गिर जाती है।
  • अतः विकल्प (B) सही उत्तर है।

https://www.drishtiias.com/hindi/daily-updates/daily-news-editorials/why-it-is-time-for-india-to-adopt-the-presidential-form-of-government

12. भारत के संविधान का कौन-सा भाग कल्याणकारी राज्य के आदर्श की घोषणा करता है?

(a) राज्य की नीति के निदेशक तत्त्व

(b) मूल-अधिकार

(c) उद्देशिका

(d) सातवीं अनुसूची

उत्तर : (a) 

व्याख्या

भारतीय संविधान के भाग IV में अनुच्छेद 36 से 51 तक राज्य के नीति निदेशक सिद्धांतों (DPSP) का उल्लेख किया गया है। DPSP एक आधुनिक और कल्याणकारी राज्य के लिये एक व्यापक सामाजिक, आर्थिक और राजनीतिक कार्यक्रम का गठन करता है। ये सिद्धांत इस बात पर ज़ोर देते हैं कि राज्य लोगों को आश्रय, भोजन और वस्त्र जैसी बुनियादी सुविधाएँ प्रदान करके उनके कल्याण को बढ़ावा देने का प्रयास करेगा। वे एक 'कल्याणकारी राज्य' की अवधारणा को मूर्त रूप देते हैं जो औपनिवेशिक युग के दौरान अनुपस्थित थी। अतः विकल्प (A) सही उत्तर है

https://www.drishtiias.com/hindi/images/dlp-demo/upsc/gs-pack-5/Polity-Part-2.pdf

13. निम्नलिखित कथनों पर विचार कीजिये-

  1. भारत का संविधान अपने मूल ढाँचे को संघवाद, पंथनिरपेक्षता, मूल अधिकारों तथा लोकतंत्र के रूप में परिभाषित करता है।
  2. भारत का संविधान, नागरिकों की स्वतंत्रता तथा उन आदेशों जिन पर संविधान आधारित है, की सुरक्षा हेतु ‘न्यायिक पुनरवलोकन’ की व्यवस्था करता है।

उपर्युक्त कथनों में से कौन-सा/से सही है/हैं?

(a) केवल 1

(b) केवल 2

(c) 1 और 2 दोनों

(d) न तो 1, न ही 2

उत्तर : (b) 

व्याख्या

  • भारत का संविधान मूल ढाँचे को परिभाषित नहीं करता है, यह एक न्यायिक नवाचार है।
  • केशवानंद भारती बनाम केरल राज्य मामले (वर्ष 1973) में, सर्वोच्च न्यायालय ने फैसला सुनाया कि संसद संविधान के किसी भी हिस्से में तब तक संशोधन कर सकती है जब तक कि वह संविधान के मूल ढाँचे या आवश्यक विशेषताओं में परिवर्तन या संशोधन नहीं करती।
  • हालाँकि, न्यायालय ने '  मूल ढाँचे' शब्द को परिभाषित नहीं किया और केवल कुछ सिद्धांतों - संघवाद, धर्मनिरपेक्षता, लोकतंत्र - को इसके हिस्से के रूप में सूचीबद्ध किया।
  • किंतु बाद में '  मूल ढाँचे' के सिद्धांत में संविधान की सर्वोच्चता, कानून के शासन, न्यायपालिका की स्वतंत्रता, शक्तियों के पृथक्करण के सिद्धांत, संप्रभु लोकतांत्रिक गणराज्य, सरकार की संसदीय प्रणाली, स्वतंत्र और निष्पक्ष निर्वाचन,कल्याणकारी राज्य को शामिल किया गया है।अतः  कथन 1 सही नहीं है
  • संविधान का अनुच्छेद 13(2) निर्धारित करता है कि संघ या राज्य कोई भी ऐसा कानून नहीं बनाएँगे जो किसी भी मौलिक अधिकार को छीनता हो या उसके महत्त्व को कम करता हो और उपरोक्त जनादेश के उल्लंघन में बनाया गया कोई भी कानून, उल्लंघन की सीमा तक, शून्य हो। इस प्रकार, संविधान स्वयं नागरिकों की स्वतंत्रता और अधिकारों की रक्षा के लिये न्यायिक समीक्षा का प्रावधान करता है। अतः कथन 2 सही है। अतः  विकल्प (B) सही है।

https://www.drishtiias.com/hindi/images/dlp-demo/bpsc/gs-pack-1/Polity-Part-1.pdf

https://www.drishtiias.com/hindi/mains-practice-question/question-1442

14. गांधीवाद और मार्क्सवाद के बीच एक समान सहमति पाई जाती है। यह निम्नलिखित में से कौन-सी है?

(a) एक अंतिम लक्ष्य के रूप में राज्यविहीन समाज 

(b) वर्ग संघर्ष

(c) निजी संपत्ति की समाप्ति

(d) आर्थिक नियतिवाद

उत्तर : (a) 

व्याख्या

  • गांधीवाद और मार्क्सवाद दोनों का अंतिम उद्देश्य एक राज्यविहीन और वर्गविहीन समाज की स्थापना है, हालाँकि इस लक्ष्य को प्राप्त करने के उनके साधन अलग-अलग हैं। महात्मा गांधी इस लक्ष्य को अहिंसात्मक तरीकों से हासिल करना चाहते थे लेकिन मार्क्स इसे हिंसक तरीकों से हासिल करना चाहते थे।
  • वर्गविहीन समाज को प्राप्त करने के तरीके
    • मार्क्सवाद: पूंजीवाद को उखाड़ फेंकना; सामाजिक उत्पादन के साधनों का सामाजिक स्वामित्व; अनिवार्य श्रम।
    • गांधीवाद: ट्रस्टीशिप का सिद्धांत: स्वयं को सार्वजनिक संपत्ति का 'ट्रस्टी' मानने के लिये नैतिक अनुनय; श्रम की गरिमा बहाल करना।

अतः विकल्प (a) सही उत्तर है।

https://www.drishtiias.com/hindi/to-the-points/paper4/karl-marx

https://www.drishtiias.com/hindi/mind-map/gs-paper4/gandhian-ideology 

15. भारत के संदर्भ में, नौकरशाही का निम्नलिखित में से कौन-सा उपयुक्त चरित्र चित्रण है?

(a) संसदीय लोकतंत्र की व्याप्ति को विस्तार देने वाला अभिकरण 

(b) संघीय ढाँचे को सुदृढ़ करने वाला अभिकरण

(c) राजनीतिक स्थायित्व और आर्थिक वृद्धि को सुलभ बनाने वाला अभिकरण

(d) लोकनीति को कार्यान्वित करने वाला अभिकरण

उत्तर : (d) 

व्याख्या

  • प्रशासनिक सेवा या नौकरशाही स्थायी और वेतनभोगी कर्मचारियों का गठन करती है जो सरकार के कार्यकारी अंग का हिस्सा हैं। वे राजनीतिक रूप से तटस्थ हैं और उनका मुख्य काम विभिन्न सरकारी विभागों के प्रभावी कामकाज और सार्वजनिक नीति के कार्यान्वयन को सुगम बनाना है।
  • हालाँकि, वे मंत्रियों के नियंत्रण और नेतृत्त्व में काम करते हैं। नौकरशाही एक संगठन के भीतर एकरूपता और नियंत्रण बनाए रखने के लिये डिज़ाइन किया गया है।
  • अच्छी नीतियाँ और कानून वास्तव में अपने उद्देश्यों की पूर्ति तभी कर सकते हैं जब लोक सेवकों द्वारा इन्हें कुशलतापूर्वक लागू किया जाए।
अतः विकल्प (D) सही उत्तर है।

16. भारत के संविधान की उद्देशिका-

(a) संविधान का भाग है किंतु कोई विधिक प्रभाव नहीं रखती

(b) संविधान का भाग नहीं है और कोई विधिक प्रभाव भी नहीं रखती

(c) संविधान का भाग है और वैसा ही विधिक प्रभाव रखती है जैसा कि उसका कोई अन्य भाग

(d) संविधान का भाग है किंतु उसके अन्य भागों से स्वतंत्र होकर उसका कोई विधिक प्रभाव नहीं है।

उत्तर : (d) 

व्याख्या

  • संविधान की उद्देशिका उसका परिचयात्मक भाग है। इसमें संविधान के आदर्श, उद्देश्य और बुनियादी सिद्धांत शामिल हैं। संविधान की मुख्य विशेषताएँ इन उद्देश्यों से प्रत्यक्ष और अप्रत्यक्ष रूप से विकसित हुई हैं जो उद्देशिका से प्रवाहित होती हैं।
  • केशवानंद भारती केस (1973) में, सर्वोच्च न्यायालय ने कहा कि:
    • संविधान की उद्देशिका को संविधान का हिस्सा माना जाए।
    • उद्देशिका सर्वोच्च शक्ति या किसी प्रतिबंध या निषेध का स्रोत नहीं है, लेकिन यह उद्देशिका की विधियों और प्रावधानों की व्याख्या में महत्त्वपूर्ण भूमिका निभाती है।
  • केंद्र सरकार बनाम एलआईसी (LIC) ऑफ इंडिया केस (वर्ष1995) में, सर्वोच्च न्यायालय ने एक बार फिर माना है कि उद्देशिका संविधान का अभिन्न अंग है, लेकिन कानून में सीधे लागू करने योग्य नहीं है। इसके उद्देश्यों को विभिन्न अधिनियमों और नीतियों के माध्यम से कार्यान्वित किया जाता है।

अतः विकल्प (D) सही उत्तर है।

https://www.drishtiias.com/hindi/to-the-points/paper2/preamble-to-the-constitution-and-the-meanings-of-the-words-contained-in-it

https://www.drishtiias.com/hindi/mains-practice-question/question-1518

17. ‘स्वर्ण-ट्रांश’ (रिज़र्व ट्रांश) निर्दिष्ट करता है:

(a) विश्व बैंक की ऋण व्यवस्था

(b) केंद्रीय बैंक की किसी एक क्रिया को

(c) WTO द्वारा इसके सदस्यों को प्रदत्त एक साख प्रणाली को

(d) IMF द्वारा इसके सदस्यों को प्रदत्त एक साख प्रणाली को

उत्तर : (d) 

व्याख्या

  • ‘स्वर्ण-ट्रांश’ (रिज़र्व ट्रांश) अंतर्राष्ट्रीय मुद्रा कोष (International Monetary Fund-IMF) के अंतर्गत स्वर्ण या विदेशी मुद्रा के रूप में एक सदस्य देश का आरक्षित अंश है। जिसका भुगतान किसी भी सदस्य देश द्वारा उसके  कुल निर्धारित आरक्षित अंश में से 25% विदेशी मुद्रा या स्वर्ण के रूप में किया जाता है इसलिये, इसे ‘स्वर्ण-ट्रांश’ (रिज़र्व ट्रांश) कहा जाता है। जबकि शेष अंश (75%) का भुगतान  घरेलू मुद्राओं में हो सकता है एवं इसे ऋण ट्रांश कहा जाता है। यह मूल रूप से एक आपातकालीन खाता है जिसे IMF के सदस्य शर्तों पर सहमत हुए बिना या सेवा शुल्क का भुगतान किये बिना एक्सेस कर सकते हैं। अतः विकल्प (d) सही उत्तर है।

18. भारत के संविधान के भाग IV में अंतर्विष्ट प्रावधानों के संदर्भ में निम्नलिखित कथनों में से कौन-सा/से सही है/हैं?

  1. वे न्यायालयों द्वारा प्रवर्तनीय होंगे।
  2. वे किसी भी न्यायालय द्वारा प्रवर्तनीय नहीं होंगे।
  3. इस भाग में अधिकथित सिद्धांत राज्य के द्वारा कानून बनाने को प्रभावित करेंगे।

नीचे दिये गए कूट का प्रयोग कर सही उत्तर चुनिये:

(a) केवल 1

(b) केवल 2

(c) केवल 1 और 3

(d) केवल 2 और 3

उत्तर : (d) 

व्याख्या

  • संविधान के भाग IV (अनुच्छेद 36-51) में निहित राज्य के नीति निदेशक सिद्धांत (Directive Principles of State Policy-DPSP) की प्रकृति गैर-न्यायिक हैं जिसका अर्थ है कि मौलिक अधिकारों (भाग III) के विपरीत, न्यायालय उन्हें उल्लंघन के लिये लागू नहीं कर सकती हैं। अतः कथन 1 सही नहीं है।
  • संविधान (अनुच्छेद 37) में निहित है कि ये सिद्धांत देश के शासन में मौलिक हैं। कानून बनाने में इन सिद्धांतों को लागू करना राज्य का कर्त्तव्य होगा। साथ ही, निदेशक सिद्धांत कानून की संवैधानिक वैधता की जाँच और निर्धारण में न्यायालयों की मदद करते हैं। अतः कथन 3 सही है।
  • अतः विकल्प d सही है।

19. निम्नलिखित कथनों पर विचार कीजिये-

  1. भारत के संविधान के अनुसार, कोई भी ऐसा व्यक्ति, जो मतदान के लिये योग्य है, किसी राज्य में छह माह के लिये मंत्री बनाया जा सकता है तब भी, जब कि वह उस राज्य के विधानमंडल का सदस्य नहीं है।
  2. लोक प्रतिनिधित्व अधिनियम, 1951 के अनुसार, कोई भी ऐसा व्यक्ति, जो दांडिक अपराध के अंतर्गत दोषी पाया गया है और जिसे पाँच वर्ष के लिये कारावास का दंड दिया गया है, चुनाव लड़ने के लिये स्थायी तौर पर निरर्हत हो जाता है, भले ही वह कारावास से मुक्त हो चुका हो।

उपर्युक्त कथनों में से कौन-सा/से सही है/हैं?

(a) केवल 1

(b) केवल 2

(c) 1 और 2 दोनों

(d) न तो 1, न ही 2

उत्तर : (a) 

व्याख्या

  • संविधान के अनुच्छेद 164 के अनुसार, एक मंत्री जो लगातार छः महीने की अवधि तक राज्य विधानमंडल का सदस्य नहीं है, वह मंत्री नहीं रहेगा। प्रावधान, एक गैर-विधायक को छः महीने के लिये मुख्यमंत्री के कार्यालय सहित मंत्रिपरिषद में एक पद ग्रहण करने की अनुमति देता है। छः महीने के भीतर, उसे राज्य विधायिका के किसी भी सदन का सदस्य (या तो चुनाव या नामांकन द्वारा) बन जाना चाहिये अन्यथा, उसे मंत्री पद से बर्खास्त कर दिया जाता है।
  • इसके अतिरिक्त, राज्य विधानमंडल का सदस्य बनने और विधानपरिषद में सेवा करने के लिये एक व्यक्ति की आयु कम से कम 30 वर्ष तथा विधानसभा में सेवा करने के लिये 25 वर्ष होनी चाहिये। ऐसा व्यक्ति स्वचालित रूप से अनुच्छेद 326 के तहत मतदान करने के योग्य हो जाता है, मतदाता के रूप में पंजीकृत होने की न्यूनतम आयु 18 वर्ष है। अतः कथन 1 सही है।
  • जनप्रतिनिधित्व अधिनियम, 1951 की धारा 8(3) के अनुसार, किसी भी अपराध के लिये दोषी ठहराए गए व्यक्ति और कम से कम दो वर्ष के कारावास की सजा का आरोपित व्यक्ति, चुनाव लड़ने (विधायक या सांसद) के लिये अयोग्य हो जाएगा। सुनाई गई सजा और अपनी रिहाई के बाद से छः वर्ष की अवधि तक अयोग्य बना रहेगा। अतः कथन 2 सही नहीं है।

अतः विकल्प a सही है।

https://www.drishtiias.com/hindi/daily-updates/daily-news-editorials/legislative-councils-in-states

20. निम्नलिखित कथनों पर विचार कीजिये-

  1. भारत का राष्ट्रपति ऐसे स्थान पर, जिसे वह ठीक समझे, संसद का सत्र आहूत (आह्वान) कर सकता है।
  2. भारत का संविधान एक वर्ष में संसद के तीन सत्रों का प्रावधान करता है, किंतु सभी तीन सत्रों का चलाया जाना अनिवार्य नहीं है।
  3. एक वर्ष में दिनों की कोई न्यूनतम संख्या निर्धारित नहीं है जब संसद का चलना आवश्यक हो।

उपर्युक्त कथनों में से कौन-सा/से सही है/हैं?

(a) केवल 1

(b) केवल 2

(c) केवल 1 और 3

(d) केवल 2 और 3

उत्तर : (c) 

व्याख्या

  • संविधान का अनुच्छेद 85 (1) भारत के राष्ट्रपति यह विधायी शक्ति प्राप्त है कि वह समय -समय पर संसद के प्रत्येक सदन को ऐसे समय और स्थान पर, जैसा वह उचित समझे, संसद का सत्र आहूत (आह्वान) करने का अधिकार देता है, लेकिन एक सत्र में इसकी अंतिम बैठक और अगले सत्र में इसकी पहली बैठक के लिये नियत तिथि के बीच छह माह का अंतराल नहीं होगा। अतः कथन 1 सही है।
  • परंपरा के अनुसार (भारत के संविधान में इसका वर्णन नहीं किया गया है), संसद एक वर्ष में तीन सत्रों के लिये मिलती है। बजट सत्र वर्ष की शुरुआत में आहूत किया जाता है; जबकि मानसून सत्र जुलाई से अगस्त तीन सप्ताह तक का चलता है और फिर नवंबर-दिसंबर में शीतकालीन सत्र होता है। अतः कथन 2 सही नहीं है।
  • एक वर्ष में संसद की बैठक के लिये न्यूनतम दिनों की संख्या निर्धारित नहीं है - संसद की बैठक के दिनों की संख्या विगत वर्षों में कम हुई  है। संसद के पहले दो दशकों के दौरान, लोक सभा की बैठकें वर्ष में औसतन 120 दिनों से थोड़ी अधिक होती थी। किंतु पिछले एक दशक में यह संख्या घटकर लगभग 70 दिन रह गई है। हालाँकि, कई समितियों ने सिफारिश की है कि संसद को एक वर्ष में न्यूनतम 120 बैठकें आयोजित करनी चाहिये। अतः कथन 3 सही है।
  • अतः विकल्प (c) सही उत्तर है

https://www.drishtiias.com/hindi/daily-updates/daily-news-editorials/for-a-more-representative-house

https://www.drishtiias.com/hindi/daily-updates/daily-news-editorials/vibrant-democracy-dormant-parliament

21. भारत के इतिहास के संदर्भ में, निम्नलिखित युग्मों पर विचार कीजिये-

  1. औरंग - राजकोष का प्रभारी
  2. बेनियान - ईस्ट इंडिया कंपनी का भारतीय एजेंट
  3. मिरासिदार - राज्य का नामित राजस्व दाता

उपर्युक्त युग्मों में से कौन-सा/से सही सुमेलित है/हैं?

(a) केवल 1 और 2 

(b) केवल 2 और 3

(c) केवल 3

(d) 1, 2 और 3

उत्तर : (b) 

व्याख्या

  • औरंग: यह एक गोदाम के लिये फारसी शब्द है जहाँ बेचने से पहले सामान एकत्र किया जाता है। अतः युग्म 1 सही सुमेलित नहीं है।
  • बेनियान : बनिया (वानिया भी) शब्द संस्कृत वणिज से लिया गया है, जिसका अर्थ है 'एक व्यापारी'। इस शब्द का व्यापक रूप से भारत के पारंपरिक व्यापारिक या व्यापारिक जातियों के सदस्यों की पहचान करने के लिये उपयोग किया जाता है। बनिया बैंकर, साहूकार, व्यापारी और दुकानदार थे। ब्रिटिश शासन के दौरान, बनियों ने बंगाल में यूरोपीय व्यापारियों के लिये मध्यस्थ के रूप में काम किया। अत: युग्म 2 सही सुमेलित है।
  • मिरासिदार: दक्षिणी भारत में रैयतवारी बंदोबस्त प्रणाली के तहत ईस्ट इंडिया कंपनी ने किरायेदारों के अधिकारों को पूरी तरह से खारिज करते हुए मिरासिदार को भूमि के एकमात्र मालिक के रूप में मान्यता दी। मिरासिदार को कानून द्वारा अपनी ज़मीन बेचने से प्रतिबंधित किया गया था लेकिन इसकी खेती के बदले में इसे किरायेदारों को मामूली शुल्क पर पट्टे पर दिया जा सकता था। अतः युग्म 3 सही सुमेलित है।
  • अत: विकल्प (b) सही उत्तर है।

22. भारत के धार्मिक इतिहास के संदर्भ में, निम्नलिखित कथनों पर विचार कीजिये-

  1. स्थविरवादी महायान बौद्ध धर्म से संबद्ध हैं।
  2. लोकोत्तरवादी संप्रदाय बौद्ध धर्म के महासंघिक संप्रदाय की एक शाखा थी।
  3. महासंघिकों द्वारा बुद्ध के देवत्वारोपण ने महायान बौद्ध धर्म को प्रोत्साहित किया।

उपर्युक्त कथनों में से कौन-सा/से सही है/हैं?

(a) केवल 1 और 2 

(b) केवल 2 और 3

(c) केवल 3

(d) 1, 2 और 3

उत्तर : (b) 

व्याख्या

  • महासंघिका, जिसका अर्थ है महान संघ, एक प्रारंभिक बौद्ध संप्रदाय था जो दूसरी बौद्ध परिषद (383 ईसा पूर्व) के बाद गठित हुआ था जब इसने मठवासी प्रथाओं (विनय के रूप में जाना जाता है) में अंतर के कारण स्वयं को दूसरे संप्रदाय, स्थविरवाद (बुजुर्गों का संप्रदाय) से अलग कर लिया था।
  • स्थविरवाद ने परिषद में भिक्षुओं के लिये आचरण के अधिक कठोर नियमों पर ज़ोर दिया लेकिन उनके सुझावों को बहुमत द्वारा अस्वीकार कर दिया गया जिन्होंने तब स्वयं को "महासंघिक" कहा।
  • बुद्ध और अर्हत (संत) की प्रकृति पर महासांघिकों के विचारों ने बौद्ध धर्म के महायान रूप के अग्रदूत के रूप में कार्य किया। अत: कथन 1 सही नहीं है, जबकि कथन 3 सही है।
  • अगली सात शताब्दियों में महासांघिकों के और उपविभागों में लोकोत्तरवादी, एकव्यवाहरिक और कौक्कुटिक शामिल किये गए। अतः कथन 2 सही है।
  • अतः विकल्प (b) सही है।

https://www.drishtiias.com/hindi/current-affairs-news-analysis-editorials/prelims-facts/08-05-2020#4

23. निम्नलिखित में से कौन-सा कथन औद्योगिक क्रांति के द्वारा 19वीं शताब्दी के पूर्वार्द्ध में भारत पर पड़े प्रभाव की सही व्याख्या करता है?

(a) भारतीय दस्तकारी उद्योग नष्ट हो गए थे।

(b) भारत के वस्त्र उद्योग में मशीनों का बड़ी संख्या में प्रवेश हुआ था।

(c) देश के अनेक भागों में रेलवे लाइनें बिछाई गई थीं।

(d) ब्रिटिश उत्पादन के आयात पर भारी शुल्क लगाया गया था।

उत्तर : (a) 

व्याख्या

  • औद्योगिक क्रांति ने भारतीय समाज के लिये गंभीर परिणाम दिये। ब्रिटिश शासन के सबसे महत्त्वपूर्ण परिणामों में से एक शहरी तथा ग्रामीण हस्तशिल्प उद्योगों का पतन एवं विनाश था।
  • वर्ष 1815 से वाष्प-शक्ति की खोज और इसके अभ्यास ने भारतीय कपड़ा उद्योग के लिये खतरा उत्पन्न कर दिया। स्पिनिंग म्यूले और पावर लूम का आविष्कार भाप की शक्ति के कारण अधिक कुशल एवं प्रभावी हो गया। फलस्वरूप इसने ब्रिटिश वस्त्रों की लागत को बहुत कम कर दिया तथा इसे अंतर्राष्ट्रीय स्तर पर प्रतिस्पर्द्धी बना दिया।
  • ब्रिटिश वस्त्र उद्योग को समर्थन देने के लिये भारतीय वस्त्रों के निर्यात पर भारी शुल्क लगाया गया। भारतीय किसानों को कपास का उत्पादन करने के लिये मज़बूर किया गया ताकि यह अंग्रेज़ी कारखानों को ईंधन दे सके।
  • हालाँकि औद्योगिक क्रांति के कुछ सकारात्मक प्रभाव भी थे। उदाहरण के लिये,
    • कारखानों और विधानसभा लाइनों की शुरूआत,
    • विद्युत विकास,
    • रेलमार्ग (भारत में पहला रेलवे: वर्ष 1853 में बॉम्बे से ठाणे तक)
  • इन सभी ने वस्तुओं और सामग्रियों के तेज़ और अधिक कुशल उत्पादन में योगदान दिया। लेकिन ये घटनाक्रम मुख्य रूप से 19वीं शताब्दी के उत्तरार्द्ध में हुए। अत: विकल्प (a) सही है।

https://www.drishtiias.com/hindi/to-the-points/paper1/impact-on-the-life-of-the-industrial-revolution

24. भारत के इतिहास में निम्नलिखित घटनाओं पर विचार कीजिये-

  1. राजा भोज के अधीन प्रतिहारों का उदय
  2. महेंद्रवर्मन-I के अधीन पल्लव सत्ता की स्थापना 
  3. परांतक-I द्वारा चोल सत्ता की स्थापना
  4. गोपाल द्वारा पाल राजवंश की संस्थापना

उपर्युक्त घटनाओं का, प्राचीन काल से आरंभ कर, सही कालानुक्रम क्या है?

(a) 2 - 1 - 4 - 3

(b) 3 - 1 - 4 - 2

(c) 2 - 4 - 1 - 3

(d) 3 - 4 - 1 - 2

उत्तर : (c) 

व्याख्या

  • पल्लव वंश 275 CE से 897 CE तक अस्तित्व में था, जो दक्षिण भारत के एक हिस्से पर शासन कर रहा था। पल्लव महेंद्रवर्मन प्रथम (571-630 CE) के शासनकाल के दौरान एक प्रमुख शक्ति बन गए, जिन्होंने वर्तमान आंध्र क्षेत्र के दक्षिणी भाग और वर्तमान तमिलनाडु के उत्तरी क्षेत्रों पर शासन किया।
  • पाल वंश ने 8वीं से 12वीं शताब्दी तक बिहार और बंगाल में शासन किया। इसके संस्थापक गोपाल (750-770 CE) एक स्थानीय मुखिया थे जो अराजकता की अवधि के दौरान आठवीं शताब्दी के मध्य में सत्ता में आए।
  • आठवीं शताब्दी के मध्य से मध्यदेश पर प्रभुत्व राजस्थान में आदिवासी लोगों के बीच दो विशेष कुलों की महत्त्वाकांक्षा बन गया जिन्हें गुर्जर और प्रतिहार के रूप में जाना जाता है। 851 CE के एक समकालीन अरब खाते के अनुसार, राजा मिहिर भोज (840-851 CE), प्रतिहार राजाओं में सबसे महान, भारत के उन राजकुमारों में से थे जिन्होंने अरब आक्रमणकारियों के खिलाफ लड़ाई लड़ी थी।
  • चोल साम्राज्य की स्थापना विजयालय ने की थी। चोलों का शासन 9वीं शताब्दी में शुरू हुआ जब उन्होंने सत्ता में आने के लिये पल्लवों को पराजित किया।  मध्ययुगीन काल चोलों के लिये पूर्ण शक्ति और विकास का युग था। परान्तक प्रथम (907-953 तक शासन किया) ने राज्य की नींव रखी। उसने उत्तरी सीमा को नेल्लोर (आंध्र प्रदेश) तक ले लिया जहाँ राष्ट्रकूट राजा कृष्ण तृतीय के हाथों पराजय से उसकी उन्नति रुक गई। परान्तक दक्षिण में अधिक सफल रहा जहाँ उसने पांड्यों तथा गंगों दोनों को पराजित किया।
  • अत: विकल्प (c) सही उत्तर है।

25. निम्नलिखित में से कौन-सा उपवाक्य, उत्तर हर्ष कालीन स्रोतों में प्राय: उल्लिखित ‘हुंडी’ के स्वरूप की परिभाषा बताता है?

(a) राजा द्वारा अपने अधीनस्थों को दिया गया परामर्श

(b) प्रतिदिन का लेखा-जोखा अंकित करने वाली बही

(c) विनिमय पत्र

(d) सामंत द्वारा अपने अधीनस्थों को दिया गया आदेश

उत्तर : (c) 

व्याख्या

  • हुंडिका या हुंडी, व्यापार और ऋण लेन-देन में उपयोग के लिये मध्यकालीन भारत में एक वित्तीय प्रणाली के रूप में विकसित विनिमय का एक बिल था।
  • इसने देश के एक हिस्से से दूसरे हिस्से में धन के आसान हस्तांतरण की सुविधा प्रदान की।
  • हुंडियों के माध्यम से व्यापारियों ने क्रेडिट बनाया जो परिसंचरण और वित्तपोषित वाणिज्य, विशेष रूप से लंबी दूरी और अंतर्राष्ट्रीय व्यापार में धन का पूरक था।
  • सर्राफ (श्रॉफ) जो धन बदलने में माहिर थे, हुंडियों से निपटने में भी माहिर थे।
  • अत: विकल्प (c) सही उत्तर है।

26. स्वतंत्रता संग्राम के समय लिखी गई सखाराम गणेश देउस्कर की पुस्तक ‘देशेर कथा’ के संदर्भ में, निम्नलिखित कथनों पर विचार कीजिये-

  1. इस पुस्तक ने औपनिवेशिक राज्य द्वारा मस्तिष्क की सम्मोहक विजय के विरोध में चेतावनी दी।
  2. इस पुस्तक ने स्वदेशी नुक्कड़ नाटकों तथा लोक गीतों को प्रेरित किया।
  3. देउस्कर द्वारा ‘देश’ शब्द का प्रयोग, बंगाल क्षेत्र के विशिष्ट संदर्भ में किया गया था।

उपर्युक्त कथनों में से कौन-से सही हैं?

(a) केवल 1 और 2 

(b) केवल 2 और 3

(c) केवल 1 और 3

(d) 1, 2 और 3

उत्तर : (a) 

व्याख्या

  • श्री अरबिंदो के एक करीबी सहयोगी, सखाराम गणेश देउस्कर (1869-1912) ने 1904 में 'देशेर कथा' नामक पुस्तक प्रकाशित की। इस पुस्तक में भारत के ब्रिटिश वाणिज्यिक और औद्योगिक शोषण का विस्तृत विवरण दिया गया है। बंगाल सरकार ने वर्ष 1910 में इस पुस्तक पर प्रतिबंध लगा दिया और सभी प्रतियाँ जब्त कर लीं।
  • इस पुस्तक का बंगाल में अत्यधिक प्रभाव पड़ा तथा इसने युवा बंगाल के मन पर कब्जा कर लिया और स्वदेशी आंदोलन की तैयारी में किसी भी अन्य चीज से अधिक सहायता की। अतः कथन 1 और 2 सही हैं।
  • देउस्कर ने अपनी पुस्तक में 'देश' का प्रयोग पूरे देश के संदर्भ में किया है। अतः कथन 3 सही नहीं है।
  • अत: विकल्प (a) सही उत्तर है।

27. गांधी-इरविन समझौते में निम्नलिखित में से क्या सम्मिलित था/थे?

  1. राउंड टेबल कॉन्फ्रेंस में भाग लेने के लिये कॉन्ग्रेस को आमंत्रित करना
  2. असहयोग आंदोलन के संबंध में जारी किये गए अध्यादेशों को वापस लेना
  3. पुलिस की ज़्यादतियों की जाँच करने हेतु गांधीजी के सुझाव की स्वीकृति
  4. केवल उन्हीं कैदियों की रिहाई जिन पर हिंसा का अभियोग नहीं था

नीचे दिये गए कूट का प्रयोग कर सही उत्तर चुनिये-

(a) केवल 1 

(b) केवल 1, 2 और 4

(c) केवल 3

(d) केवल 2, 3 और 4

उत्तर : (b) 

व्याख्या

  • दिल्ली पैक्ट (5 मार्च 1931) ने कॉन्ग्रेस  को सरकार के साथ बराबरी पर ला खड़ा किया तथा इसे गांधी-इरविन पैक्ट के नाम से भी जाना जाता है।
  • वायसराय और भारत के गवर्नर-जनरल (1926-1931), लॉर्ड इरविन ब्रिटिश सरकार की ओर से महात्मा गांधी के साथ निम्नलिखित शर्तों पर सहमत हुए:
    • हिंसा के दोषी नहीं सभी राजनीतिक कैदियों की तत्काल रिहाई;
    • अभी तक एकत्र नहीं किये गए सभी जुर्माने की छूट;
    • सभी ज़मीनों की वापसी जो अभी तक तीसरे पक्ष को नहीं बेची गई हैं;
    • इस्तीफा देने वाले सरकारी कर्मचारियों के साथ उदार व्यवहार;
    • निजी उपभोग के लिये तटीय गाँवों में नमक बनाने का अधिकार (बिक्री के लिये नहीं);
    • शांतिपूर्ण और गैर-आक्रामक धरने का अधिकार; और
    • आपातकालीन अध्यादेशों को वापस लेना। अतः कथन 2 और 4 सही हैं।
  • हालाँकि वायसराय ने गांधीजी की दो मांगों को ठुकरा दिया:
    • पुलिस ज्यादतियों की सार्वजनिक जाँच, और
    • भगत सिंह और उनके साथियों की फाँसी की सजा को उम्रकैद में बदलने की मांग। अतः कथन 3 सही नहीं है।
  • कॉन्ग्रेस की ओर से गांधीजी ने सहमति व्यक्त की:
  • सविनय अवज्ञा आंदोलन को स्थगित करने के लिये, और
  • अगले गोलमेज सम्मेलन में भाग लेने के लिये (सम्मेलन का पहला दौर पहले ही नवंबर 1930 से जनवरी 1931 तक आयोजित किया गया था) संघ के तीन लिंचपिन, भारतीय ज़िम्मेदारी तथा आरक्षण और सुरक्षा उपायों के संवैधानिक प्रश्न पर भारत के हितों में आवश्यक हो सकता है ( रक्षा, विदेश मामले, अल्पसंख्यकों की स्थिति, भारत के वित्तीय ऋण तथा अन्य दायित्त्वों के निर्वहन जैसे क्षेत्रों को कवर करना)। अतः कथन 1 सही है।
  • अत: विकल्प (b) सही है।

https://www.drishtiias.com/hindi/mains-practice-question/question-287

28. अस्पृश्य समुदाय के लोगों को लक्षित कर, प्रथम मासिक पत्रिका विटाल-विध्वंसक किसके द्वारा प्रकाशित की गई थी?

(a) गोपाल बाबा वलंगकर

(b) ज्योतिबा फुले

(c) मोहनदास करमचंद गांधी

(d) भीमराव रामजी अंबेडकर

उत्तर : (a) 

व्याख्या

  • गोपाल बाबा वालंगकर (1840-1900) का जन्म महाराष्ट्र के रायगढ़ ज़िले में अछूत महार जाति के एक परिवार में हुआ था तथा इन्हें गोपाल कृष्ण के नाम से भी जाना जाता है।
  • वह महाराष्ट्र में महारों के अधिकारों के लिये लड़ने वाले पहले व्यक्ति थे।
  • वह वर्ष 1886 में सैन्य सेवा से सेवानिवृत्त हुए और लोगों को लामबंद किया तथा उन्हें उनके मानवाधिकारों के प्रति जागरूक किया।
  • वालंगकर ने वर्ष 1888 में वाइटल-विधवंसक (ब्राह्मणिक या औपचारिक प्रदूषण का विनाशक) नामक मासिक पत्रिका प्रकाशित करना शुरू किया जो अछूत लोगों को लक्षित दर्शकों के रूप में सबसे पहले था। अत: विकल्प (a) सही उत्तर है।

29. भारत के इतिहास के संदर्भ में, ‘कुल्यावाप’ तथा ‘द्रोणवाप’ शब्द क्या निर्दिष्ट करते हैं?

(a) भू-माप

(b) विभिन्न मौद्रिक मूल्यों के सिक्के

(c) नगर की भूमि का वर्गीकरण

(d) धार्मिक अनुष्ठान

उत्तर : (a) 

व्याख्या

  • गुप्त काल में कुल्यवाप, द्रोणवाप और अधवापा शब्द मुख्य रूप से बंगाल में पाए गए ताम्रपत्र शिलालेखों में उल्लिखित भूमि माप से संबंधित थे।
  • ये शब्द भूमि के उस क्षेत्र को इंगित करते हैं जो क्रमशः एक कुल्य, द्रोण और अधक के वजन के बीज अनाज बोने के लिये आवश्यक था। "वाप" धातु से व्युत्पन्न "कुल्यवाप" यौगिक में "वाप" तत्त्व का अर्थ बोने की क्रिया (फेंकना या बिखेरना) से है।
  • सुभाकीर्ति की मिदनापुर प्लेट में 40 द्रोण भूमि और एक द्रोणवाप ग्राम कुम्भरपद्रका गाँव में दाम्यस्वामिन नाम के एक ब्राह्मण को उपहार में दिया गया था।
  • बंगाल के शिलालेखों में द्रोणवाप के साथ-साथ इसके अन्य डिवीजनों और गुणकों जैसे अधवाप और कुल्यवाप का भी उपयोग किया गया था।
  • संस्कृत शब्दकोशों के अनुसार, एक द्रोणवाप कुल्यवाप के एक-आठ के बराबर था और इसकी पुष्टि पुरालेखीय साक्ष्यों से भी होती है।
  • अत: विकल्प (a) सही उत्तर है।

30. निम्नलिखित में से किस शासक ने अपनी प्रजा को इस अभिलेख के माध्यम से परामर्श दिया?

‘‘कोई भी व्यक्ति जो अपने संप्रदाय को महिमा-मंडित करने की दृष्टि से अपने धार्मिक संप्रदाय की प्रशंसा करता है या अपने संप्रदाय के प्रति अत्यधिक भक्ति के कारण अन्य संप्रदायों की निंदा करता है, वह अपितु अपने संप्रदाय को गंभीर रूप से हानि पहुँचाता है।’’ 

(a) अशोक

(b) समुद्रगुप्त

(c) हर्षवर्धन

(d) कृष्णदेव राय

उत्तर : (a) 

व्याख्या

  • अपने राजाज्ञा XII में, अशोक किसी और के धर्म की कीमत पर अपने स्वयं के धर्म को ऊपर उठाने की प्रथा की निंदा करता है:
  • सार में वृद्धि भिन्न-भिन्न प्रकार से की जा सकती है लेकिन इन सबके मूल में वाणी का संयम है अर्थात् अपने धर्म की प्रशंसा न करना या बिना किसी कारण के दूसरों के धर्म की निन्दा करना और अगर आलोचना का कोई कारण है, तो उसे विनम्र तरीके से किया जाना चाहिये। लेकिन इस कारण से अन्य धर्मों का सम्मान करना बेहतर है।
  • ऐसा करने से अपने धर्म का लाभ होता है और इसी प्रकार दूसरे धर्म भी ऐसा करते समय अन्यथा अपने धर्म और दूसरों के धर्म को हानि पहुँचाते हैं।
  • जो व्यक्ति अत्यधिक भक्ति के कारण अपने धर्म की प्रशंसा करता है और दूसरों की निंदा इस विचार से करता है कि 'मुझे अपने धर्म की महिमा करनी चाहिये', वह केवल अपने धर्म का नुकसान करता है। दूसरों द्वारा बताए गए सिद्धांतों को सुनना और उनका सम्मान करना चाहिये।
  • यह आदेश इस चेतावनी के साथ समाप्त होता है कि एक व्यक्ति का धर्म धम्म के माध्यम से बढ़ता है तथा इसलिये सभी धर्मों में सहिष्णुता और समझ में सुधार होता है।
  • अत: विकल्प (a) सही उत्तर है।

31. भारत के सांस्कृतिक इतिहास के संदर्भ में, ‘परामिता’ शब्द का सही विवरण निम्नलिखित में से कौन-सा है?

(a) सूत्र पद्धति में लिखे गए प्राचीनतम धर्मशास्त्र पाठ

(b) वेदों के प्राधिकार को अस्वीकार करने वाले दार्शनिक संप्रदाय

(c) परिपूर्णताएँ जिनकी प्राप्ति से बोधिसत्व पथ प्रशस्त हुआ

(d) आरंभिक मध्यकालीन दक्षिण भारत की शक्तिशाली व्यापारी श्रेणियाँ

उत्तर : (c) 

व्याख्या

  • परामिता या परामी (क्रमशः संस्कृत और पाली में) एक बौद्ध मत में प्रचलित शब्द है जिसे अक्सर "पूर्णता" के रूप में अनुवादित किया जाता है।
  • महायान बौद्ध धर्म में, बोधिसत्व छह पारमिताओं या पारलौकिक सिद्धियों का अभ्यास करते हैं जो उदारता, अनुशासन, धैर्य, परिश्रम, ध्यान और ज्ञान हैं।
  • बौद्ध भाष्य में पारमिता का वर्णन आमतौर पर प्रबुद्ध प्राणियों से जुड़े महान चरित्र गुणों के रूप में किया गया है।

अतः विकल्प (c) सही उत्तर है।

32. भारतीय इतिहास के संदर्भ में, 1884 का रखमाबाई मुकदमा किस पर केंद्रित था?

  1. महिलाओं का शिक्षा पाने का अधिकार
  2. सहमति की आयु
  3. दांपत्य अधिकारों का प्रत्यास्थापन

नीचे दिये गए कूट का प्रयोग कर सही उत्तर चुनिये-

(a) केवल 1 और 2

(b) केवल 2 और 3

(c) केवल 1 और 3

(d) 1, 2 और 3

उत्तर : (b) 

व्याख्या

  • रखमाबाई (1864-1955) ने इतिहास में अपनी पहचान उस मुकदमे के कारण बनाई, जिसके कारण सहमति की आयु अधिनियम, 1891 को प्राख्यापित किया गया।
  • वर्ष 1885 में विवाह के 12 वर्ष के उपरांत, इनके पति ने "वैवाहिक अधिकारों की बहाली" की मांग की,इस मुकदमे के निर्णय में  रखमाबाई को अपने पति के साथ रहने या छह महीने जेल में बिताने का आदेश दिया गया। अतः कथन 3 सही है।
  • रखमाबाई ने उस आदमी के साथ रहने से इनकार कर दिया जिससे उनकी बचपन में शादी हुई थी, क्योंकि शादी में उनकी कोई भूमिका नहीं थी। इस संबंध में रखमाबाई ने महारानी विक्टोरिया को पत्र लिखा। रानी ने न्यायालय के फैसले को खारिज कर दिया और विवाह को भंग कर दिया।
  • इस मुकदमे ने जो लहर पैदा की, उसी के परिणामस्वरूप सहमति की आयु अधिनियम, 1891 को पारित  किया गया, जिसने संपूर्ण ब्रिटिश साम्राज्य में बाल विवाह को अवैध बना दिया। अतः कथन 2 सही है।
  • हालाँकि रखमाबाई ब्रिटिश भारत में चिकित्सा का अभ्यास करने वाली पहली महिला डॉक्टर बनीं, किंतु  यह मुकदमा महिलाओं के शिक्षा प्राप्त करने के अधिकार से संबंधित नहीं था। अतः कथन 1 सही नहीं है।

अतः विकल्प (b) सही उत्तर है।

33. निम्नलिखित में से किस कारण से भारत में बीसवीं शताब्दी के आरंभ में नील की खेती का ह्रास हुआ?

(a) नील के उत्पादकों के अत्याचारी आचरण के प्रति काश्तकारों का विरोध

(b) नई खोजों के कारण विश्व बाज़ार में इसका अलाभकर होना

(c) नील की खेती का राष्ट्रीय नेताओं द्वारा विरोध किया जाना

(d) उत्पादकों के ऊपर सरकार का नियंत्रण

उत्तर : (b) 

व्याख्या

  • नील एक नीला रंग है जो नील के पौधे से प्राप्त होता है। ईस्ट इंडिया कंपनी के शासन के दौरान एवं बाद में ब्रिटिश राज के दौरान भारत में इसकी कृषि व्यापक रूप से की गई। इसे अक्सर "ब्लू गोल्ड" कहा जाता था तथा यूरोपीय बाज़ार में इसका व्यापक रूप से कारोबार किया जाता था।
  • 18वीं और 19वीं शताब्दी में बंगाल और बिहार में बढ़ती मांग और उत्पादन के कारण नील के उत्पादन में उल्लेखनीय वृद्धि हुई थी। हालाँकि  20वीं शताब्दी की शुरुआत में, सिंथेटिक नील के आविष्कार के साथ, जो कि सस्ता था और नील की कृषि की तुलना में, उत्पादन कम समय में होता था, भारत में नील की खेती की मांग में काफी गिरावट आई थी। कृत्रिम नील ने प्राकृतिक नील को किसानों और व्यापारियों के लिये गैर-लाभप्रद बना दिया।

अतः विकल्प (b) सही उत्तर है।

34. वेलेज़ली ने कलकत्ता में फोर्ट विलियम कॉलेज की स्थापना किस लिये की थी?

(a) उसे लंदन में स्थित बोर्ड ऑफ डायरेक्टर्स ने ऐसा करने के लिये कहा था

(b) वह भारत में प्राच्य ज्ञान के प्रति अभिरुचि पुन: जाग्रत करना चाहता था

(c) वह विलियम कैरी तथा उसके सहयोगियों को रोज़गार प्रदान करना चाहता था

(d) वह ब्रिटिश नागरिकों को भारत में प्रशासन हेतु प्रशिक्षित करना चाहता था

उत्तर : (d) 

व्याख्या

  • ब्रिटिश अधिकारियों को प्रशिक्षित करने के लिये, फोर्ट विलियम कॉलेज की स्थापना वर्ष 1800 में बंगाल के गवर्नर-जनरल (1798 से 1805) लॉर्ड रिचर्ड वेलेज़ली द्वारा की गई थी। कॉलेज का उद्देश्य ऐसे सिविल सेवकों का निर्माण करना था जो भारतीय भाषाओं, इतिहास, संस्कृति और स्थानीय कानूनों से परिचित हों।
  • साथ ही, यह  पश्चिमी भाषाओं और प्रशासन की कला में भी प्रशिक्षण प्राप्त करेंगे।

अतः विकल्प (d) सही उत्तर है।

https://www.drishtiias.com/hindi/images/dlp-demo/upsc/gs-pack-4/Modern-India-Part-I.pdf

35. भारत के इतिहास के संदर्भ में, ‘‘ऊलगुलान’’ अथवा महान उपद्रव निम्नलिखित में से किस घटना का विवरण था?

(a) 1857 के विद्रोह का

(b) 1921 के मापिला विद्रोह का

(c) 1859–60 के नील विद्रोह का

(d) 1899–1900 के बिरसा मुंडा विद्रोह का

उत्तर : (d) 

व्याख्या

  • बिरसा मुंडा (1875-1900) का जन्म मुंडा जनजाति में हुआ था जो बंगाल प्रेसीडेंसी के छोटा नागपुर क्षेत्र (वर्तमान झारखंड) में बसी हुई थी। उन्हें अक्सर 'धरती अब्बा' या पृथ्वी पिता के रूप में जाना जाता है।
  • बिरसा मुंडा ने जिस विद्रोह का नेतृत्त्व किया जिसे ऊलगुलान (विद्रोह) या ब्रिटिश सरकार द्वारा थोपे गए सामंती राज्य व्यवस्था के खिलाफ मुंडा विद्रोह के रूप में जाना जाने लगा।
  • उन्होंने जन जागरूकता का कार्य किया एवं ज़मींदारों के साथ-साथ अंग्रेज़ों के खिलाफ विद्रोह की शुरुआत में महत्त्वपूर्ण भूमिका निभाई।
  • आदिवासियों के खिलाफ शोषण और भेदभाव के खिलाफ उनके संघर्ष के कारण वर्ष 1908 में छोटानागपुर काश्तकारी अधिनियम पारित किया गया, जिसने आदिवासी लोगों से गैर-आदिवासियों के लिये भूमि के हस्तांतरण को प्रतिबंधित कर दिया।

अतः विकल्प (d) सही उत्तर है।

36. प्राचीन भारत के विद्वानों/साहित्यकारों के संदर्भ में, निम्नलिखित कथनों पर विचार कीजिये-

  1. पाणिनि पुष्यमित्र शुंग से संबंधित है।
  2. अमरसिंह हर्षवर्धन से संबंधित है।
  3. कालिदास चंद्रगुप्त-II से संबंधित है।

उपर्युक्त कथनों में से कौन-सा/से सही है/हैं?

(a) केवल 1 और 2

(b) केवल 2 और 3

(c) केवल 3

(d) 1, 2 और 3

उत्तर : (c) 

व्याख्या

  • तक्षशिला अपने शिक्षकों की विशेषज्ञता के कारण विद्यार्जन की जगह के रूप में प्रसिद्ध हुआ। इसके विख्यात शिष्यों में प्रसिद्ध भारतीय व्याकरणविद, पाणिनि थे। वह भाषा और व्याकरण के विशेषज्ञ थे और व्याकरण पर अष्टाध्यायी (500 ईसा पूर्व) नामक सबसे महान कार्यों में से एक के लेखक थे।
  • शुंग साम्राज्य मगध का एक प्राचीन भारतीय राजवंश था जिसने लगभग 185 से 75 ईसा पूर्व तक मध्य और पूर्वी भारतीय उपमहाद्वीप के क्षेत्रों को नियंत्रित किया था। मौर्य साम्राज्य के पतन के बाद, पुष्यमित्र शुंग द्वारा राजवंश की स्थापना की गई थी। अतः कथन 1 सही नहीं है।
  • माना जाता है कि अमरसिंह, एक संस्कृत व्याकरणविद और कवि, उन नौ रत्नों में से एक थे, जो विक्रमादित्य (चंद्रगुप्त द्वितीय) के दरबार में सुशोभित थे, जिनका शासनकाल लगभग 375 CE था। हर्षवर्धन ने 606 से 647 CE तक उत्तर भारत पर शासन किया। अतः कथन 2 सही नहीं है
  • शास्त्रीय संस्कृत लेखक एवं नाटककार कालिदास, चंद्रगुप्त द्वितीय के शासनकाल के दौरान प्रसिद्ध हुए और उनका समय चौथी-पाँचवी शताब्दी था। अतः कथन 3 सही है
  • अतः विकल्प (C) सही है।

37. मानव प्रजनन तकनीकी में अभिनव प्रगति के संदर्भ में, ‘‘प्राक्केंद्रिक स्थानांतरण’’ (Pronuclear Transfer) का प्रयोग किस लिये होता है?

(a) इन विट्रो अंड के निषेचन के लिये दाता शुक्राणु का उपयोग

(b) शुक्राणु उत्पन्न करने वाली कोशिकाओं का आनुवंशिक रूपांतरण

(c) स्टेम (Stem) कोशिकाओं का कार्यात्मक भ्रूूणों में विकास

(d) संतान में सूत्रकणिका वाले रोगों का निरोध

उत्तर : (d) 

व्याख्या

  • प्राक्केंद्रिक स्थानांतरण में एक युग्मनज से दूसरे युग्मनज में प्राक्केंद्रक का स्थानांतरण शामिल है। इस तकनीक में पहले डोनेट किये गए स्वस्थ डिम्ब (माइटोकॉन्ड्रियल डोनर द्वारा प्रदान किये गए) को इच्छित नर जनक के शुक्राणु के साथ निषेचन की आवश्यकता होती है। इसके साथ ही, इच्छुक मादा जनक के प्रभावित अंडाणु को इच्छुक पिता के शुक्राणु के साथ निषेचित किया जाता है।
  • 'मैटरनल स्पिंडल ट्रांसफर' तकनीक के प्रयोग द्वारा, मातृ DNA को एक डोनर मादा के डिम्ब में डाल दिया जाता है, जिसे बाद में नर जनक के शुक्राणु का उपयोग करके निषेचित किया जाता है।
  • प्रक्रिया को मौजूदा इन-विट्रो-फर्टिलाइजेशन (IVF) उपचार जिसमें मातृ जनकों को माइटोकॉन्ड्रियल रोग होते हैं, में मदद करने के लिये विकसित किया गया था ।
  • मातृ DNA में उत्परिवर्तन, माइटोकॉन्ड्रियल रोग विकारों के विषम संग्रह का एक कारण है, जो भ्रूण या फिर प्रसव उपरांत शिशु की मृत्यु का कारण बन सकता है। अधिकांश माइटोकॉन्ड्रियल रोगों में विशिष्ट उपचार की कमी होती है, और जो महिलाएँ जिनमें ऐसे उत्परिवर्तन होते हैं, उनके शिशुओं में रोग प्रसारित करने के जोखिम उच्च होते हैं। अतः विकल्प d सही है।

38. विकास की वर्तमान स्थिति में, कृत्रिम बुद्धिमत्ता (Artificial Intelligence), निम्नलिखित में से किस कार्य को प्रभावी रूप से कर सकती है?

  1. औद्योगिक इकाइयों में विद्युत् की खपत कम करना
  2. सार्थक लघु कहानियों और गीतों की रचना
  3. रोगों का निदान
  4. टेक्स्ट से स्पीच (Text- to- Speech) में परिवर्तन
  5. विद्युत् ऊर्जा का बेतार संचरण

नीचे दिये गए कूट का प्रयोग कर सही उत्तर चुनिये-

(a) केवल 1, 2, 3 और 5

(b) केवल 1, 3 और 4

(c) केवल 2, 4 और 5

(d) 1, 2, 3, 4 और 5

उत्तर : (d) 

व्याख्या

  • गूगल अपने डेटा केंद्रों में ऊर्जा खपत को 30% तक कम करने के लिये अपने डीप माइंड एक्वीज़ीशन से इंटरनेट ऑफ थिंग्स (IoT) और आर्टिफिशियल इंटेलिजेंस (AI) का उपयोग कर रहा है। अतः विकल्प 1 सही है।
  • संगीत या संगीतकारों की सहायता के लिये एक उपकरण के रूप में AI का उपयोग काफी समय से किया जा रहा है। 1990 के दशक में डेविड बॉवी ने वर्बेसाइजर के निर्माण में योगदान दिया, जिसने साहित्यिक स्रोतों को लिया और नए संयोजनों का निर्माण करने के लिये शब्दों को बेतरतीब ढंग से पुनर्व्यवस्थित किया, जिन्हें गीत के रूप में इस्तेमाल किया जा सकता था। वर्ष 2016 में, सोनी ने द बीटल्स की शैली में एक राग बनाने के लिये फ्लो मशीन नामक सॉफ्टवेयर का उपयोग किया। संगीत बनाने वाला AI सॉफ्टवेयर पिछले कुछ वर्षों में उन्नत हुआ है। यह एक व्यवहार्य उपकरण है जो रचनात्मक प्रक्रिया में मदद करने के लिये प्रोड्यूसर्स द्वारा उपयोग किया जा सकता है और किया जा रहा है। अतः विकल्प 2 सही है।
  • रोबोटिक्स और इंटरनेट ऑफ मेडिकल थिंग्स (IoMT) के साथ AI स्वास्थ्य देखभाल के लिये नई तंत्रिका प्रणाली हो सकती है, जो स्वास्थ्य संबंधी समस्याओं के समाधान प्रस्तुत करती है। कैंसर देखभाल में AI प्रौद्योगिकी के एकीकरण से निदान की सटीकता और गति में सुधार हो सकता है, नैदानिक निर्णय लेने में मदद मिल सकती है एवं बेहतर स्वास्थ्य परिणाम प्राप्त हो सकते हैं। अतः विकल्प 3 सही है।
  • स्पीच सिंथेसिस, ह्यूमन स्पीच (मानवीय आवाज़) का कृत्रिम रूपांतरण है। यह भाषा को मानवीय आवाज़ (या भाषण) में बदलने का एक तरीका है। उदाहरण के लिये, Google का असिस्टेंट, Amazon का Echo, Apple का सिरी आदि। अत: विकल्प 4 सही है।
  • अप्रत्याशितता को कम करने और शक्ति संतुलन एवं उपयोग में दक्षता को बढ़ावा देने के लिये ऊर्जा प्रणाली की मॉडलिंग एवं पूर्वानुमान ऊर्जा क्षेत्र में AI के संभावित अनुप्रयोग हैं। विद्युत ऊर्जा का बेतार संचरण कोई नई तकनीक नहीं है। एक वायरलेस पॉवर ट्रांसमिशन सिस्टम के तहत ऊर्जा स्रोत से विद्युत ऊर्जा द्वारा संचालित एक ट्रांसमीटर डिवाइस, समय के साथ बदलते रहने वाला विद्युत चुम्बकीय क्षेत्र उत्पन्न करता है जो एक रिसीवर डिवाइस के निकटवर्ती क्षेत्र में ऊर्जा प्रसारित करता है और इस प्रकार स्रोत से ऊर्जा का निष्कर्षण कर विद्युत की आपूर्ति करता है। अत:  विकल्प (d) सही उत्तर है।

https://www.drishtiias.com/hindi/daily-updates/daily-news-editorials/adverse-consequences-of-ai 

https://www.drishtiias.com/hindi/loksabha-rajyasabha-discussions/artificial-intelligence-in-india 

39. दृश्य प्रकाश संचार (VLC) तकनीकी के संदर्भ में निम्नलिखित में से कौन-से कथन सही हैं?

  1. VLC, 375 से 780 nm वाली विद्युत्-चुंबकीय स्पेक्ट्रमी तरंगदैर्ध्यों का उपयोग करती है।
  2. VLC को दीर्घ-परासी प्रकाशी बेतार संचार के रूप में जाना जाता है।
  3. VLC ब्ल्यूटूथ की तुलना में डेटा की विशाल मात्रा को अधिक तेज़ी से प्रेषित कर सकती है।
  4. VLC में विद्युत्-चुंबकीय व्यतिकरण नहीं होता है।

नीचे दिये गए कूट का प्रयोग कर सही उत्तर चुनिये-

(a) केवल 1, 2 और 3 

(b) केवल 1, 2 और 4

(c) केवल 1, 3 और 4

(d) केवल 2, 3 और 4

उत्तर : (c) 

व्याख्या

  • दृश्य प्रकाश संचार (VLC) प्रणाली संचार के लिये दृश्य प्रकाश का उपयोग करती हैं जो विद्युत् चुंबकीय स्पेक्ट्रम को 375 nm से 780 nm तक ले जाता है। अतः कथन 1 सही है।
  • VLC को लघु-परासी प्रकाशी बेतार संचार के रूप में जाना जाता है। अतः कथन 2 सही नहीं है।
  • Li-Fi, एक तरह का VLC है जिसका परास लगभग 10 मीटर है और यह दीवारों या किसी ठोस वस्तु से नहीं गुजर सकता है।
  • VLC ब्लूटूथ की तुलना में बड़ी मात्रा में डेटा तेज़ी से प्रेषित कर सकता है। VLC 10 Gb/s तक उच्च गति इंटरनेट प्रदान करने के लिये संचार के लिये दृश्य प्रकाश का उपयोग करता है जबकि ब्लूटूथ 4.0 25 Mb/s तक की गति प्रदान करता है। अतः कथन 3 सही है।
  • VLC में कोई विद्युत् चुंबकीय व्यतिकरण नहीं है। रेडियो फ्रीक्वेंसी (RF) आधारित संकेतों में अन्य RF संकेतों के साथ व्यतिकरण की समस्या होती है जैसे विमान में पायलट नौवहन उपकरण संकेतों के साथ इसका व्यतिकरण। इसलिये उन क्षेत्रों में जो विद्युत् चुंबकीय विकिरण (जैसे वायुयान) के प्रति संवेदनशील हैं VLC एक बेहतर समाधान हो सकता है। अतः कथन 4 सही है।
  • अत: विकल्प (c) सही उत्तर है।

https://www.drishtiias.com/hindi/daily-updates/daily-news-analysis/potential-for-using-li-fi-in-india-is-simply-enormous

40. ‘‘ब्लॉकचेन तकनीकी’’ के संदर्भ में, निम्नलिखित कथनों पर विचार कीजिये-

  1. यह एक सार्वजनिक खाता है जिसका हर कोई निरीक्षक कर सकता है, परंतु जिसे कोई भी एक उपभोक्ता नियंत्रित नहीं करता।
  2. ब्लॉकचेन की संरचना और अभिकल्प ऐसा है कि इसका समूचा डेटा केवल क्रिप्टोकरेंसी के विषय में है।
  3. ब्लॉकचेन के आधारभूत वैशिष्ट्यों पर आधारित अनुप्रयोगों को बिना किसी व्यक्ति की अनुमति के विकसित किया जा सकता है।

उपर्युक्त कथनों में से कौन-सा/से सही है/हैं?

(a) केवल 1

(b) केवल 1 और 2

(c) केवल 2

(d) केवल 1 और 3

उत्तर : (d) 

व्याख्या

  • ब्लॉकचेन, सार्वजनिक बहीखाता का एक रूप है, जो ब्लॉकों की एक शृंखला है, जिस पर निर्दिष्ट नेटवर्क प्रतिभागियों द्वारा उपयुक्त प्रमाणीकरण और सत्यापन के बाद सार्वजनिक डेटाबेस पर लेनदेन विवरण दर्ज एवं संग्रहीत किये जाते हैं। सार्वजनिक खाते को किसी भी एकल उपयोगकर्त्ता द्वारा देखा जा सकता है लेकिन नियंत्रित नहीं किया जा सकता। अतः कथन 1 सही है।
  • यह पता चला है कि ब्लॉकचेन केवल क्रिप्टो-करेंसी के ही नहीं बल्कि वास्तव में अन्य प्रकार के लेनदेन के बारे में लेन-देन या डेटा संग्रहीत करने का एक बहुत ही विश्वसनीय तरीका है।
  • वास्तव में, ब्लॉकचेन तकनीक का उपयोग संपत्ति के आदान-प्रदान, बैंक लेनदेन, स्वास्थ्य सेवा, स्मार्ट अनुबंध, आपूर्ति शृंखला और यहाँ तक कि उम्मीदवार के लिये मतदान में भी किया जा सकता है। अतः कथन 2 सही नहीं है।
  • यद्यपि, क्रिप्टो-करेंसी को विनियमित किया जाता है जिसे केंद्रीय अधिकारियों के अनुमोदन की आवश्यकता होती है, ब्लॉकचेन तकनीक केवल क्रिप्टो-करेंसी से ही संबद्ध नहीं है, इसके विभिन्न उपयोग हो सकते हैं और प्रौद्योगिकी की बुनियादी विशेषताओं पर आधारित अनुप्रयोगों को बिना किसी की स्वीकृति के विकसित किया जा सकता है। अतः कथन 3 सही है। 
  • अतः विकल्प d सही है।

https://www.drishtiias.com/hindi/loksabha-rajyasabha-discussions/blockchain-technique

https://www.drishtiias.com/hindi/daily-updates/daily-news-analysis/npci-mulls-blockchain-to-boost-e-pay

41. कार्बन नैनोट्यूबों के संदर्भ में, निम्नलिखित कथनों पर विचार कीजिये-

  1. इनको मानव शरीर में औषधियों और प्रतिजनों के वाहकों के रूप में प्रयुक्त किया जा सकता है।
  2. इनको मानव शरीर के क्षतिग्रस्त भाग के लिये कृत्रिम रक्त कोशिकाओं के रूप में बनाया जा सकता है।
  3. इनका जैव-रासायनिक संवेदकों में उपयोग किया जा सकता है।
  4. कार्बन नैनोट्यूब जैव-निम्नीकरणीय (Biodegradable) होती हैं।

उपर्युक्त कथनों में से कौन-सा/से सही है/हैं?

(a) केवल 1 और 2

(b) केवल 2, 3 और 4

(c) केवल 1, 3 और 4

(d) 1, 2, 3 और 4

उत्तर : (d) 

व्याख्या

  • कार्बन नैनोट्यूब (CNTs) बेलनाकार अणु होते हैं जिनमें संकरित कार्बन परमाणुओं की एक हेक्सागोनल आकृति होती है, जिसे ग्राफीन की शीटों को रोल करके बनाया जा सकता है।
  • हाल के वर्षों में कार्बन नैनोट्यूब, कई शोधकर्त्ताओं के मध्य औषधियों और प्रतिजनों के वाहकों के रूप में आकर्षण का केंद्र रहा है। क्योंकि कार्बन नैनोट्यूब का उच्च सतह क्षेत्र होता है जिससे यह लाखों की संख्या में कोशिका में प्रवेशित होने की क्षमता रखते हैं जो कि एक वाहक के रूप में इनकी उच्च दक्षता को दर्शाता है। अतः कथन 1 सही है।
  • मानव शरीर के क्षतिग्रस्त भाग के लिये कार्बन नैनोट्यूब को कृत्रिम रक्त कोशिकाओं के रूप में बनाया जा सकता है क्योंकि यह औषधियों के लिये वाहक के रूप में कार्य करते हैं जो कि एंटीबॉडी, प्रोटीन या DNA जैसे विभिन्न जैव-अणुओं के साथ क्रियाशील हो सकते हैं। अतः कथन 2 सही है।
  • हाल ही में, नासा ने कार्बन नैनोट्यूब एरेज़ का उपयोग करते हुए  जैव-रासायनिक संवेदकों के रूप में  किया है। अतः कथन 3 सही है।
  • बैक्टीरिया और कवक सहित कई प्रकार के सूक्ष्मजीवों में कार्बन नैनोट्यूब (CNTs) के जैव-निम्नीकरण की क्षमता होती है। अतः कथन 4 सही है।

अतः विकल्प (d) सही उत्तर है।

https://www.drishtiias.com/hindi/daily-updates/daily-news-analysis/iit-bombay-team-produces-a-unique-device

42. निम्नलिखित गतिविधियों पर विचार कीजिये-

  1. खेत में फसल पर पीड़कनाशी छिड़कना
  2. सक्रिय ज्वालामुखियों के मुखों का निरीक्षण करना
  3. डी.एन.ए. विश्लेषण के लिये उत्क्षेपण करती हुई ह्वेलों के श्वास के नमूने एकत्र करना

तकनीकी के वर्तमान स्तर पर, उपर्युक्त गतिविधियों में से किसे, ड्रोन के प्रयोग से सफलतापूर्वक किया जा सकता है?

(a) केवल 1 और 2 

(b) केवल 2 और 3

(c) केवल 1 और 3  

(d) 1, 2 और 3

उत्तर : (d) 

व्याख्या

  • मानव रहित हवाई वाहन (UAV) या ड्रोन ऐसे विमान हैं जिन्हें मानव पायलट के बिना संचालित किया जा सकता है। GPS  ट्रैकिंग सिस्टम का उपयोग करके ड्रोन को सुदूर संवेदन के माध्यम से संचालित किया जा सकता है।
  • प्रारंभ में मुख्यतः इन्हें सैन्य अनुप्रयोगों के लिये विकसित किया गया था। हालाँकि, वर्तमान में इनके अनुप्रयोगों का विस्तार विविध क्षेत्रों तक हो गया है जिनमें वैज्ञानिक, वाणिज्यिक, शांति व्यवस्था,निगरानी, उत्पाद वितरण, हवाई फोटोग्राफी, कृषि, आदि क्षेत्र शामिल हैं।
  • खड़ी फसलों को कीटों से बचाव हेतु कृषि क्षेत्रों में कीटनाशकों का छिड़काव करने के लिये अब इनका तेज़ी से उपयोग किया जा रहा है। अतः कथन 1 सही है।
  • सक्रिय ज्वालामुखियों का अध्ययन करने के लिये वैज्ञानिक भी ड्रोन का उपयोग कर रहे हैं। ड्रोन वेल मछलियों के श्वास के नमूने एकत्र कर सकता है और सामान्य स्वास्थ्य स्थितियों का आकलन करने के लिये आकाश से इनकी उच्च-रिज़ॉल्यूशन तस्वीरें ले सकता है। अतः कथन 2 और 3 सही हैं।

अतः विकल्प (d) सही उत्तर है।

https://www.drishtiias.com/hindi/daily-updates/daily-news-analysis/where-can-you-fly-a-drone-in-delhi

43. ‘‘यह प्रयोग तीन ऐसे अंतरिक्षयानों को काम में लाएगा जो एक समबाहु त्रिभुज की आकृति में उड़ान भरेंगे जिसमें प्रत्येक भुजा एक मिलियन किलोमीटर लंबी है और यानों के बीच लेज़र चमक रही होंगी।’’ कथित प्रयोग किसे संदर्भित करता है?

(a) वॉयेजर-2

(b) न्यू हॉरायज़न्स

(c) LISA पाथफाइंडर

(d) इवोल्वड LISA

उत्तर : (d) 

व्याख्या

  • विकसित लेजर इंटरफेरोमीटर स्पेस एंटीना (eLISA) अंतरिक्ष में तीन अंतरिक्ष यान, एक मुख्य  और दो सहायक अंतरिक्ष यान स्थापित करने की एक महत्त्वपूर्ण योजना है, जो एक त्रिकोणीय गठन में उड़ान भरेगी एवं सूर्य के चारों ओर अपनी कक्षा में पृथ्वी का अनुगमन करेगी।
  • 50 मिलियन किमी. काल्पनिक त्रिभुज की प्रत्येक भुजा, मुख्य अंतरिक्ष यान से प्रत्येक सहायक अंतरिक्ष यान तक, लगभग एक लाख किमी. मापी जाएगी।
  • eLISA 0.1 मेगाहर्ट्ज से लेकर लगभग 100 मेगाहर्ट्ज तक की आवृत्ति रेंज में गुरुत्त्वीय तरंगों को मापने का प्रयास करता है। इसे प्राप्त करने हेतु, इंटरफेरोमीटर के लिये दस लाख किलोमीटर की भुजा की लंबाई होना आवश्यक है और पृथ्वी आधारित प्रणाली के साथ इसे प्राप्त करना असंभव है।

अतः विकल्प (d) सही उत्तर है।

https://www.drishtiias.com/hindi/daily-updates/daily-news-analysis/nasa-voyager-2

https://www.drishtiias.com/hindi/daily-updates/prelims-facts/prelims-facts-6-november-2019

44. निम्नलिखित कथनों पर विचार कीजिये-

  1. भावी माता-पिता के अंड या शुक्राणु उत्पन्न करने वाली कोशिकाओं में आनुवंशिक परिवर्तन किये जा सकते हैं।
  2. व्यक्ति का जीनोम जन्म से पूर्व प्रारंभिक भ्रूणीय अवस्था में संपादित किया जा सकता है।
  3. मानव प्रेरित बहुशक्त स्टेम (Pluripotent Stem) कोशिकाओं को एक शूकर के भ्रूण में अंतर्वेशित किया जा सकता है।

उपर्युक्त कथनों में से कौन-सा/से सही है/हैं?

(a) केवल 1

(b) केवल 2 और 3

(c) केवल 2

(d) 1, 2 और 3

उत्तर : (d) 

व्याख्या

  • जर्मलाइन जीन थेरेपी अंडे या शुक्राणु कोशिकाओं में जीन के प्रतिस्थापन को संदर्भित करती है जिसके साथ संतान नवीन आनुवांशिक गुण प्राप्त करती है। यह रोग उत्पन्न करने वाले जीनों में सुधार की अनुमति देता है जो निश्चित रूप से पीढ़ी दर पीढ़ी अंतरित होते हैं। अतः कथन 1 सही है।
  • CRISPR  (क्लस्टर्ड रेगुलर इंटरस्पेस्ड शॉर्ट पालिंड्रोमिक रिपीट) तकनीक मानव भ्रूण को महिलाओं के गर्भाशय में स्थानांतरित करने से पूर्व संशोधित करने के लिये उपयोग में लाई जाती है। हाल ही में, शोधकर्त्ताओं ने विश्व का पहला आनुवंशिक रूप से संपादित संतति विकसित किया है। CRISPR तकनीक का उपयोग करके भ्रूण के जीनोम को एक जीन, CCR5 को निष्क्रिय करने के लिये संपादित किया गया, जो HIV को कोशिकाओं को संक्रमित करने की अनुमति देता है। अतः कथन 2 सही है।
  • मनुष्यों के साथ साझा की जाने वाली कुछ शारीरिक विशेषताओं के कारण, सुअर को सर्जरी और  ज़ेनोट्रांसप्लांटेशन अध्ययनों में अद्वितीय लाभों के साथ मानव रोगों का एक महत्त्वपूर्ण जीव मॉडल माना जाता है। अतः कथन 3 सही है।

अतः विकल्प (d) सही उत्तर है।

https://www.drishtiias.com/hindi/current-affairs-news-analysis-editorials/prelims-facts/01-08-2020 

45. भारत में न्यूमोकोकल संयुग्मी वैक्सीन (Pneumococcal Conjugate Vaccine) के उपयोग का क्या महत्त्व है?

  1. ये वैक्सीन न्यूमोनिया और साथ ही तानिकाशोथ और सेप्सिस के विरुद्ध प्रभावी हैं।
  2. उन प्रतिजैविकियों पर निर्भरता कम की जा सकती है जो औषध-प्रतिरोधी जीवाणु के विरुद्ध प्रभावी नहीं हैं।
  3. इन वैक्सीन के कोई गौण प्रभाव (Side Effects) नहीं हैं और न ही ये वैक्सीन कोई प्रत्यूर्जता संबंधी अभिक्रियाएँ (Allergic Reactions) करती हैं।

नीचे दिये गए कूट का प्रयोग कर सही उत्तर चुनिये-

(a) केवल 1

(b) केवल 1 और 2

(c) केवल 3

(d) 1, 2 और 3

उत्तर : (b) 

व्याख्या

  • न्यूमोकोकल संयुग्मी वैक्सीन(PCV) न्यूमोकोकल रोगों को रोकते हैं। न्यूमोकोकल रोग न्यूमोकोकल बैक्टीरिया के कारण होने वाली किसी भी बीमारी को संदर्भित करता है। स्ट्रेप्टोकोकस न्यूमोनिया (न्यूमोकोकस) बच्चों में बैक्टीरियल निमोनिया, तानिकाशोथ और सेप्सिस का एक प्रमुख कारण है।
  • न्यूमोकोकल संयुग्मी वैक्सीन (PCV) विशेष रूप से छोटे बच्चों में जीवाणु रोग, निमोनिया, तानिकाशोथ , सेप्सिस और मध्यकर्णशोथ के प्रकरणों को संभावित रूप से रोक सकते हैं। अतः कथन 1 सही है।
  • आमतौर पर इस्तेमाल किये जाने वाले एंटीबायोटिक दवाओं में न्यूमोकोकल बैक्टीरिया का बढ़ता प्रतिरोध न्यूमोकोकल रोग को नियंत्रित करने के लिये टीकों के इस्तेमाल की तत्काल आवश्यकता को रेखांकित करता है। PCV, एंटीबायोटिक प्रतिरोधी न्यूमोकोकल संक्रमणों को रोकता है। अतः कथन 2 सही है।
  • त्वचा का लाल होना , सूजन, दर्द, या कोमलता जहां शॉट दिया जाता है, और बुखार, भूख न लगना, घबराहट (चिड़चिड़ापन), थकान महसूस करना, सिरदर्द एवं ठंड लगना हो सकता है। अतः कथन 3 सही नहीं है।

अतः  विकल्प (B) सही उत्तर है।

https://www.drishtiias.com/hindi/daily-updates/daily-news-analysis/pneumococcal-vaccine 

https://www.drishtiias.com/hindi/daily-updates/daily-news-analysis/a-new-vaccine-to-protect-against-various-diseases 

46. भारत में, ‘‘पब्लिक की इंफ्रास्ट्रक्चर’’ (Public Key Infrastructure) पदबंध किसके प्रसंग में प्रयुक्त किया जाता है?

(a) डिजिटल सुरक्षा आधारभूत संरचना

(b) खाद्य सुरक्षा आधारभूत संरचना

(c) स्वास्थ्य देखभाल और शिक्षा आधारभूत संरचना

(d) दूरसंचार और परिवहन आधारभूत संरचना

उत्तर : (a) 

व्याख्या

  • पब्लिक की इन्फ्रास्ट्रक्चर (Public Key Infrastructure) डिजिटल विश्व में उपयोगकर्त्ताओं और उपकरणों को प्रमाणित करने की तकनीक है। इस प्रणाली के अंतर्गत एक या अधिक विश्वसनीय पक्ष डिजिटल रूप से दस्तावेज़ों पर हस्ताक्षर करते हैं जो प्रमाणित करते हैं कि एक विशेष क्रिप्टोग्राफ़िक कुंजी किसी विशेष उपयोगकर्त्ता  या डिवाइस से संबंधित है। कुंजी को डिजिटल नेटवर्क में उपयोगकर्त्ता की पहचान के रूप में उपयोग किया जा सकता है।

अतः विकल्प (a) सही उत्तर है।

47. निम्नलिखित कथनों में से कौन-से पादप और प्राणि कोशिकाओं के बीच सामान्य अंतर के बारे में सही हैं?

  1. पादप कोशिकाओं में सेलुलोस कोशिका भित्तियाँ होती हैं जबकि प्राणि कोशिकाओं में वे नहीं होतीं।
  2. पादप कोशिकाओं में प्लाज़्मा झिल्ली नहीं होती जबकि इसके विपरीत प्राणि कोशिकाओं में वे होती हैं।
  3. परिपक्व पादप कोशिका में एक वृहत् रसधानी होती है जबकि प्राणि कोशिका में अनेक छोटी रसधानियाँ होती हैं।

नीचे दिये गए कूट का प्रयोग कर सही उत्तर चुनिये-

(a) केवल 1 और 2

(b) केवल 2 और 3

(c) केवल 1 और 3

(d) 1, 2 और 3

उत्तर : (c) 

व्याख्या

  • पादप और प्राणी कोशिकाओं के बीच अंतर होता है क्योंकि पादपों में कोशिका भित्ति, क्लोरोप्लास्ट, प्लास्टिड और एक बड़ी केंद्रीय रसधानी होती है जो कि प्राणी कोशिकाओं में अनुपस्थित होती है। अतः कथन 1 और 3 सही हैं।
  • दूसरी ओर प्राणी कोशिकाओं में सेंट्रीओल्स होते हैं जो लगभग सभी पादप की कोशिकाओं में अनुपस्थित होते हैं। पादप और प्राणी दोनों कोशिकाओं में प्लाज्मा झिल्ली होती है। अतः कथन 2 सही नहीं है।

अतः विकल्प (c) सही उत्तर है।

48. निम्नलिखित में से कौन-से कारण/कारक बेंज़ीन प्रदूषण उत्पन्न करते हैं?

  1. स्वचालित वाहन (Automobile) द्वारा निष्कासित पदार्थ
  2. तंबाकू का धुआँ
  3. लकड़ी का जलना
  4. रोगन किये गए लकड़ी के फर्नीचर का उपयोग 
  5. पॉलियूरिथेन से निर्मित उत्पादों का उपयोग

नीचे दिये गए कूट का प्रयोग कर सही उत्तर चुनिये-

(a) केवल 1, 2 और 3

(b) केवल 2 और 4

(c) केवल 1, 3 और 4

(d) 1, 2, 3, 4 और 5

उत्तर : (d) 

व्याख्या

  • बेंजीन (C6H6) एक रंगहीन, ज्वलनशील तरल पदार्थ है जिसमें मीठी गंध होती है। वायु के संपर्क में आने पर यह त्वरित वाष्पित हो जाता है। बेंजीन प्राकृतिक प्रक्रियाओं से निर्मित होता है, जैसे कि ज्वालामुखी एवं वनाग्नि, किंतु बेंजीन का अधिकांश उत्सर्जन मानव गतिविधियों का परिणाम है।
  • पर्यावरण में बेंजीन  के मुख्य स्रोतों में स्वचालित वाहन (Automobile) द्वारा निष्कासित पदार्थ, औद्योगिक इकाइयों एवं गैसोलीन फिलिंग स्टेशनों से ईंधन का वाष्पीकरण आदि शामिल है। अतः कथन 1 सही है।
  • इनडोर वायु में उच्च स्तर पर बेंजीन का पता लगाया जा सकता है। हालाँकि कुछ स्तर पर इसका उत्सर्जन निर्माण सामग्री (पेंट, चिपकाने वाले पदार्थ, आदि) से होता है, किंतु वृहत स्तर पर सिगरेट या तंबाकू से  धुँआ इसके प्रदूषण का प्रमुख कारक है। अतः कथन 2 और 4 सही हैं।
  • अतः विकल्प (d) सही उत्तर है।

49. यदि निकट भविष्य में दूसरा वैश्विक वित्तीय संकट होता है, तो निम्नलिखित में से कौन-से कार्य/नीतियाँ, भारत को सबसे अधिक संभावना के साथ, कुछ उन्मुक्ति प्रदान कर सकती है/हैं?

  1. अल्पकालीन विदेशी ऋणों पर निर्भर न रहना
  2. कुछ और विदेशी बैंकों को प्रारंभ करना
  3. पूंजी खाते में पूर्ण परिवर्तनीयता को बनाए रखना

नीचे दिये गए कूट का प्रयोग कर सही उत्तर चुनिये-

(a) केवल 1

(b) केवल 1 और 2

(c) केवल 3

(d) 1, 2 और 3

उत्तर : (a) 

व्याख्या

  • विदेशी ऋण एक सरकार, निगम या निजी घराने द्वारा दूसरे देश की सरकार या निजी उधारदाताओं से उधार ली गई राशि है। कुल विदेशी ऋण अल्पकालीन और दीर्घकालीन देनदारियों का संयोजन हो सकता है।
  • दीर्घकालीन विदेशी ऋणों की तुलना में, अल्पावधि विदेशी ऋण अधिक अस्थिर होते हैं और वैश्विक संकट के समय में तरलता की कमी पैदा कर सकते हैं। इसलिये, अल्पकालीन विदेशी ऋणों पर निर्भर नहीं होने से वैश्विक संकट के समय निश्चित रूप से भारत को कुछ छूट मिल सकती है। अतः कथन 1 सही है।
  • विदेशी बैंकों ने भारतीय ग्राहकों को ATM और क्रेडिट कार्ड से परिचित कराने में महत्त्वपूर्ण भूमिका निभाई है। इसके अलावा, भारतीय कंपनियाँ तेजी से विदेशों में निवेश की तलाश कर रही हैं, विदेशी बैंक उनके लिये राशि एकत्रित करने में महत्त्वपूर्ण भूमिका निभा सकते हैं, उन्हें वैश्विक ग्राहकों और उपभोक्ताओं से जोड़ सकते हैं।
  • हालाँकि, विदेशी पोर्टफोलियो निवेशक (FPI) आज कई विदेशी बैंकों के सबसे बड़े शेयरधारक हैं, जिसका अर्थ है कि वैश्विक संकट की स्थिति में, FPI अपना पैसा निकाल लेंगे तथा इसे कहीं और जमा कर देंगे, इस प्रकार, बाज़ार में अस्थिरता एवं नकद राशि की कमी होगी। अतः कथन 2 सही नहीं है।
  • पूँजी खाता परिवर्तनीयता विदेशी निवेशकों की भारतीय संपत्ति (जैसे इक्विटी, बॉन्ड, संपत्ति) और घरेलू नागरिकों को विदेशी वित्तीय संपत्ति खरीदने की स्वतंत्रता है। पूर्ण पूँजीगत खाते में परिवर्तनशीलता  में कुछ कमियाँ हैं जैसे-उच्च अस्थिरता, विदेशी ऋण का बढ़ता बोझ, व्यापार और निर्यात के संतुलन पर प्रभाव शामिल हैं। इस प्रकार, पूर्ण पूँजी खाते की परिवर्तनीयता की अनुमति देने से वैश्विक संकट के समय में अर्थव्यवस्था को नुकसान होगा। अतः कथन 3 सही नहीं है।

अत: विकल्प (a) सही उत्तर है।

50. यदि आप अपने बैंक के मांग जमा खाते (Demand Deposit Account) से  1,00,000 रुपए की नकद राशि निकालते हैं, तो अर्थव्यवस्था में तात्कालिक रूप से मुद्रा की समग्र पूर्ति पर इसका क्या प्रभाव पड़ेगा?

(a) मुद्रा की समग्र पूर्ति में 1,00,000 रुपए की कमी आएगी

(b) मुद्रा की समग्र पूर्ति में 1,00,000 रुपए की वृद्धि होगी

(c) मुद्रा की समग्र पूर्ति में 1,00,000 रुपए से अधिक की वृद्धि होगी

(d) मुद्रा की समग्र पूर्ति अपरिवर्तित रहेगी

उत्तर : (d) 

व्याख्या

  • किसी विशेष समय में लोक प्रचलन में मुद्रा का कुल भंडार मुद्रा आपूर्ति कहलाता है। यहाँ यह ध्यान देने की आवश्यकता है कि मुद्रा का कुल भंडार मुद्रा की कुल आपूर्ति से भिन्न होता है।
  • मुद्रा की आपूर्ति मुद्रा के कुल भंडार का केवल वह भाग है जो किसी विशेष समय में लोक प्रचलन में होती है।
  • परिसंचारी मुद्रा के अंतर्गत, मुद्रित नोट, जमा खातों में निधि और अन्य तरल संपत्तियाँ शामिल होती है।
  • RBI मुद्रा आपूर्ति के चार वैकल्पिक उपायों के लिये आँकड़े प्रकाशित करता है, जो कि M1, M2, M3 और M4 हैं।
    • M 1 = CU + DD
    • M 2 = M1 + डाकघर बचत बैंकों के साथ बचत जमा
    • M 3 = M1 + वाणिज्यिक बैंकों की शुद्ध सावधि जमा
    • M 4 = M3 + डाकघर बचत बैंकों के पास कुल जमा राशि (राष्ट्रीय बचत प्रमाणपत्र को छोड़कर)
  • CU जनता द्वारा धारित मुद्रा (नोट एवं सिक्के) है और DD वाणिज्यिक बैंकों द्वारा धारित शुद्ध मांग जमा है।
  • M1 और M2 को नैरो मनी कहा जाता है। M3 और M4 को ब्रॉड मनी के रूप में जाना जाता है। उनकी तरलता का क्रम है: M1 > M2 > M3 > M4।
  • M1 मुद्रा की आपूर्ति का एक संकीर्ण उपाय है जिसमें भौतिक मुद्रा, मांग जमा, ट्रैवेलर्स चेक और अन्य जाँच योग्य जमा शामिल हैं।
  • M1 लेनदेन के लिये सबसे अधिक तरल और आसान है। इसलिये मांग जमा खाते से नकद निकालने पर, अर्थव्यवस्था में कुल मुद्रा समग्र आपूर्ति पर कोई  तात्कालिक प्रभाव नहीं पड़ेगा।

अतः विकल्प (d) सही उत्तर है।

51. भारत में प्रत्यक्ष विदेशी निवेश के संदर्भ में, निम्नलिखित में से कौन-सी उसकी प्रमुख विशेषता मानी जाती है?

(a) यह मूलत: किसी सूचीबद्ध कंपनी में पूंजीगत साधनों द्वारा किया जाने वाला निवेश है।

(b) यह मुख्यत: ऋण सृजित न करने वाला पूंजी प्रवाह है।

(c) यह ऐसा निवेश है जिससे ऋण-समाशोधन अपेक्षित होता है।

(d) यह विदेशी संस्थागत निवेशकों द्वारा सरकारी प्रतिभूतियों में किया जाने वाला निवेश है।

उत्तर : (b)

व्याख्या

  • प्रत्यक्ष विदेशी निवेश (FDI) भारत से बाहर के निवासी व्यक्ति द्वारा पूंजी उपकरणों के माध्यम से किया गया निवेश है::  
    • एक असूचीबद्ध भारतीय कंपनी; या
    • एक सूचीबद्ध भारतीय कंपनी के पूरी तरह से ‘पेड-अप’ इक्विटी पूंजी का 10% या अधिक।
  • इस प्रकार FDI किसी सूचीबद्ध या गैर-सूचीबद्ध कंपनी में हो सकता है।
  • एफडीआई के माध्यम से भारत में निवेश की गई पूंजी गैर-ऋण उत्पन्नकर्त्ता पूंजी प्रवाह वाली है और इससे ऋण चुकाने की अनुमति नहीं है।
  • एक निवेश को विदेशी पोर्टफोलियो निवेश कहा जाता है, यदि पूंजी उपकरणों में भारत से बाहर के निवासी व्यक्ति (या संस्थागत निवेशक) द्वारा निवेश किया जाता है:

52. वर्तमान में भारत के अंतर्राष्ट्रीय व्यापार के संदर्भ में, निम्नलिखित में से कौन-सा/से कथन सही है/हैं?

  1. भारत के माल का निर्यात, माल के आयात से कम है।
  2. भारत के लोहे व इस्पात, रसायनों, उर्वरकों और मशीनों के आयात में हाल के वर्षों में कमी आई है।
  3. भारत की सेवाओं का निर्यात, सेवाओं के आयात से अधिक है।
  4. भारत को कुल मिलाकर व्यापार/चालू खाते का घाटा हो रहा है। 

नीचे दिये गए कूट का प्रयोग कर सही उत्तर चुनिये-

(a) केवल 1 और 2

(b) केवल 2 और 4

(c) केवल 3

(d) केवल 1, 2 और 4

उत्तर : (d)   

व्याख्या

  • अगस्त 2020 तक, भारत का व्यापारिक निर्यात 22.70 बिलियन अमेरिकी डॉलर था और व्यापारिक आयात 29.47 बिलियन अमेरिकी  डॉलर था जिसका स्पष्ट अर्थ है कि व्यापारिक निर्यात इसके व्यापारिक आयात से कम है। अतः कथन 1 सही है।
  • आर्थिक सर्वेक्षण 2020 के अनुसार, भारत के लोहे और इस्पात के आयात में कमी आई है लेकिन रसायनों, उर्वरकों तथा मशीनरी के आयात में वृद्धि हुई है। अतः कथन 2 सही नहीं है।
  • अप्रैल-अगस्त 2020-21 तक सेवा निर्यात 49.56 अरब अमेरिकी डॉलर के अनुमानित सेवा आयात की तुलना में 84.47 अरब अमेरिकी डॉलर रहने का अनुमान है। इसका अर्थ है, भारत का सेवाओं का निर्यात सेवाओं के आयात से अधिक है। अतः कथन 3 सही है।
  • व्यापार घाटा एक ऐसी स्थिति है जब वस्तुओं का निर्यात इसके आयात से कम होता है जबकि चालू खाता घाटा एक ऐसी स्थिति है जब वस्तुओं तथा सेवाओं में समग्र व्यापार नकारात्मक पक्ष पर होता है (अर्थात् देश निर्यात से अधिक आयात करता है)। अभी तक भारत का माल का निर्यात इसके आयात से कम है लेकिन सेवाओं का निर्यात इसके आयात से अधिक है। इसके अलावा उच्च व्यापार घाटे के कारण समग्र व्यापार संतुलन नकारात्मक है। इसलिये भारत समग्र व्यापार/चालू खाता घाटे से ग्रस्त है। अतः कथन 4 सही है।
  • अत: विकल्प (d) सही उत्तर है।

53. कभी-कभी समाचारों में पाया जाने वाला पद ‘वेस्ट टेक्सास इंटरमीडिएट (West Texas Intermediate)’ निम्नलिखित में से किस एक पदार्थ की श्रेणी से संबंधित है?

(a) कच्चे तेल की

(b) बहुमूल्य-धातु (Bullion) की

(c) दुर्लभ मृदा तत्त्वों की

(d) यूरेनियम की

उत्तर : (a)

व्याख्या

  • वेस्ट टेक्सास इंटरमीडिएट (WTI), जिसे टेक्सास लाइट स्वीट के रूप में भी जाना जाता है, कच्चे तेल का एक ग्रेड है जिसका उपयोग तेल मूल्य निर्धारण में एक बेंचमार्क के रूप में किया जाता है।
  • WTI को इसके अपेक्षाकृत कम घनत्व के कारण हल्के कच्चे तेल के रूप में वर्णित किया गया है और इसमें सल्फर की मात्रा कम होने के कारण इसे स्वीट के रूप में वर्णित किया गया है।
  • यह मुख्य रूप से टेक्सास, लुइसियाना और नॉर्थ डकोटा के अमेरिकी तेल क्षेत्रों से प्राप्त होता है। अतः विकल्प (a) सही है।

https://www.drishtiias.com/hindi/daily-updates/daily-news-analysis/oil-prices-fell-below-zero

54. भारतीय अर्थव्यवस्था के संदर्भ में, निम्नलिखित में से कौन-सा/से गैर-वित्तीय ऋण में सम्मिलित है/हैं?

  1. परिवारों का बकाया गृह ऋण
  2. क्रेडिट कार्डों पर बकाया राशि
  3. राजकोष बिल (Treasury bills)

नीचे दिये गए कूट का प्रयोग कर सही उत्तर चुनिये-

(a) केवल 1

(b) केवल 1 और 2

(c) केवल 3

(d) 1, 2 और 3

उत्तर : (d)

व्याख्या:

  • ब्याज के साथ ऋण यानी मौद्रिक कर्ज़/उधार चुकाना, संविदात्मक दायित्त्व है।
  • गैर-वित्तीय ऋण:
    • इसमें सरकारी संस्थाओं, परिवारों और व्यवसायों द्वारा जारी क्रेडिट उपकरण शामिल होते हैं जो कि वित्तीय क्षेत्र में शामिल नहीं हैं।
    • इसमें औद्योगिक अथवा वाणिज्यिक कर्ज, राजकोषीय बिल (ट्रेज़री बिल) और क्रेडिट कार्ड शेष (Balance) शामिल होते हैं।
    • ये बड़े पैमाने पर वित्तीय ऋण के समान होते हैं, इस अपवाद के साथ कि गैर-वित्तीय संस्थाएँ उन्हें जारी करती हैं। अतः कथन 1, 2 और 3 सही हैं।
    • अतः विकल्प (d) सही उत्तर है।

55. भारत में, क्यों कुछ परमाणु रिएक्टर ‘‘आई.ए.ई.ए. सुरक्षा उपायों’’ के अधीन रखे जाते हैं, जबकि अन्य इस सुरक्षा के अधीन नहीं रखे जाते?

(a) कुछ यूरेनियम का प्रयोग करते हैं और अन्य थोरियम का

(b) कुछ आयातित यूरेनियम का प्रयोग करते हैं और अन्य घरेलू आपूर्ति का

(c) कुछ विदेशी उद्यमों द्वारा संचालित होते हैं और अन्य घरेलू उद्यमों द्वारा

(d) कुछ सरकारी स्वामित्व वाले होते हैं और अन्य निजी स्वामित्व वाले

उत्तर : (b)

व्याख्या

  • विभिन्न परमाणु सुविधाओं को IAEA सुरक्षा उपायों के तहत रखा जाता है, यदि यूरेनियम का स्रोत, जो परमाणु रिएक्टर के लिये विखंडनीय सामग्री है, भारतीय क्षेत्र से बाहर से आता है या फिर नए रिएक्टर संयंत्र विदेशी सहयोग से स्थापित किये गए हैं।
  • इसका उद्देश्य यह सुनिश्चित करना है कि आयातित यूरेनियम को सैन्य उपयोग के लिये डायवर्ट नहीं किया जाएगा और साथ ही आयातित यूरेनियम का उपयोग नागरिक उद्देश्यों के लिये परमाणु ऊर्जा उत्पन्न करने हेतु किया जाएगा।
  • वर्तमान में 22 रिएक्टर परिचालन में हैं, जिनमें से 14 IAEA सुरक्षा उपायों के अधीन हैं क्योंकि ये आयातित ईंधन का उपयोग करते हैं।
  • सुरक्षा उपायों के समझौते के अनुसार, IAEA के पास यह सुनिश्चित करने का अधिकार और दायित्व है कि राज्य के एकमात्र नियंत्रण, अधिकार क्षेत्र के तहत सभी परमाणु सामग्री सुरक्षा उपायों के अधीन है। अतः विकल्प (b) सही है।

https://www.drishtiias.com/hindi/printpdf/un-opens-atomic-fuel-reserve-in-kazakhstan-to-ensure-supply

56. ‘व्यापार-संबंधित निवेश उपायों’ (TRIMs) के संदर्भ में, निम्नलिखित कथनों में से कौन-सा/से सही है/हैं?

  1. विदेशी निवेशकों द्वारा किये जाने वाले आयात पर ‘परिमाणात्मक निर्बंधन’ निषिद्ध होते हैं।
  2. ये वस्तुओं एवं सेवाओं दोनों के व्यापार से संबंधित निवेश उपायों पर लागू होते हैं।
  3. ये विदेशी निवेश के नियमन से संबंधित नहीं हैं।

नीचे दिये गए कूट का प्रयोग कर सही उत्तर चुनिये:

(a) केवल 1 और 2

(b) केवल 2

(c) केवल 1 और 3

(d) 1, 2 और 3

उत्तर:(c) 

व्याख्या

  • विश्व व्यापार संगठन (WTO) के व्यापार-संबंधित निवेश उपायों (TRIMS) पर समझौते के तहत, जिसे सामान्यत: TRIMS समझौते (उरुग्वे दौर 1986-1994 के दौरान बातचीत) के रूप में जाना जाता है, WTO सदस्य कुछ ऐसे निवेश उपायों को लागू नहीं करने पर सहमत हुए हैं जो भेदभाव करते हैं। विदेशी वस्तुओं के खिलाफ जो व्यापार (GATT अनुच्छेद III के तहत राष्ट्रीय व्यवहार) को प्रतिबंधित या विकृत करते हैं या मात्रात्मक प्रतिबंध (अनुच्छेद XI) की ओर ले जाते हैं जो दोनों आधारभूत WTO सिद्धांतों का उल्लंघन करते हैं। अतः कथन 1 सही है।
  • यह समझौता केवल उन उपायों पर लागू होता है जो माल के व्यापार को प्रभावित करते हैं। अतः कथन 2 सही नहीं है।
  • समझौते का संबंध विदेशी निवेश के नियमन से नहीं है। TRIMS समझौते के विषय उन निवेश उपायों पर ध्यान केंद्रित करते हैं जो GATT के अनुच्छेद III और XI का उल्लंघन करते हैं। दूसरे शब्दों में, यह उन निवेश उपायों पर ध्यान केंद्रित करता है जो आयातित और निर्यातित उत्पादों के बीच भेदभाव करते हैं। अतः कथन 3 सही नहीं है।
  • अत: विकल्प (c) सही उत्तर है।

57. यदि आर.बी.आई. प्रसारवादी मौद्रिक नीति का अनुसरण करने का निर्णय लेता है, तो वह निम्नलिखित में से क्या नहीं करेगा?

  1. वैधानिक तरलता को घटाकर उसे अनुकूलित करना
  2. सीमांत स्थायी सुविधा दर को बढ़ाना
  3. बैंक दर को घटाना तथा रेपो दर को भी घटाना

नीचे दिये गए कूट का प्रयोग कर सही उत्तर चुनिये-

(a) केवल 1 और 2

(b) केवल 2

(c) केवल 1 और 3

(d) 1, 2 और 3

उत्तर:(b) 

व्याख्या

  • विस्तारित मौद्रिक नीति या आसान मौद्रिक नीति, जब एक केंद्रीय बैंक अर्थव्यवस्था को प्रोत्साहित करने के लिये अपने उपकरणों का उपयोग करता है तो यह पैसे की आपूर्ति को बढ़ाता है, ब्याज दरों को कम करता है तथा मांग को बढ़ाता है। यह आर्थिक विकास को बढ़ावा देता है।
  • वैधानिक तरलता अनुपात (SLR) एक मौद्रिक नीति उपकरण है जिसका उपयोग भारतीय रिज़र्व बैंक (RBI) बैंकों के निपटान में नकदी का आकलन करने के लिये करता है। यह जमा का न्यूनतम प्रतिशत है जिसे एक वाणिज्यिक बैंक को नकदी, सोना या अन्य प्रतिभूतियों के रूप में बनाए रखना होता है। यह मूल रूप से आरक्षित आवश्यकता है जो बैंकों से ग्राहकों को ऋण देने से पहले रखने की अपेक्षा की जाती है।
  • SLR बढ़ाने से बैंकों को सरकारी प्रतिभूतियों में अधिक धन मिलता हैं और अर्थव्यवस्था में नकदी के स्तर को कम कम किया जाता है। इसके विपरीत करने से अर्थव्यवस्था में नकदी प्रवाह बनाए रखने में मदद मिलती है। SLR को कम करने से बैंकों के पास अधिक तरलता रह जाती है जो बदले में अर्थव्यवस्था में विकास और मांग को बढ़ा सकती है। अतः कथन 1 सही नहीं है।
  • मार्जिनल स्टैंडिंग फैसिलिटी (MSF) अनुसूचित बैंकों के लिये एक आपातकालीन स्थिति में भारतीय रिज़र्व बैंक से रातों-रात उधार लेने की एक विंडो है जब इंटरबैंक नकदी पूरी तरह से समाप्त हो जाती है। MSF दर में वृद्धि के साथ, बैंकों के लिये उधार लेने की लागत बढ़ जाती है जिसके परिणामस्वरूप उधार देने के लिये उपलब्ध संसाधन कम हो जाते हैं। अतः कथन 2 सही है।
  • रेपो दर या पुनर्खरीद दर, ब्याज की प्रमुख मौद्रिक नीति दर है जिस पर केंद्रीय बैंक या भारतीय रिज़र्व बैंक (RBI), सरकार की संपार्श्विक और अन्य अनुमोदित प्रतिभूतियों के खिलाफ नकदी समायोजन सुविधा (LAF) के तहत बैंकों को अल्पकालिक धन उधार देता है। बैंक दर वह ब्याज दर है जो RBI अपने दीर्घकालिक ऋणों पर वसूलता है। विस्तारवादी मौद्रिक नीति के तहत RBI बैंकिंग क्षेत्र में नकदी बढ़ाने के लिये रेपो दर और बैंक दर को कम करता है। अतः कथन 3 सही नहीं है।
  • अत: विकल्प (b) सही उत्तर है।

https://www.drishtiias.com/hindi/loksabha-rajyasabha-discussions/monetary-policy

58. 1991 के आर्थिक उदारीकरण के बाद की भारतीय अर्थव्यवस्था के संबंध में, निम्नलिखित कथनों पर विचार कीजिये-

  1. शहरी क्षेत्रों में श्रमिक की उत्पादकता (2004-05 की कीमतों पर प्रति श्रमिक) में वृद्धि हुई जबकि ग्रामीण क्षेत्रों में इसमें कमी हुई।
  2. कार्यबल में ग्रामीण क्षेत्रों की प्रतिशत हिस्सेदारी में सतत् वृद्धि हुई।
  3. ग्रामीण क्षेत्रों में, गैर-कृषि अर्थव्यवस्था में वृद्धि हुई।
  4. ग्रामीण रोज़गार की वृद्धि दर में कमी आई।

उपर्युक्त कथनों में से कौन-सा/से सही है/हैं?

(a) केवल 1 और 2

(b) केवल 3 और 4

(c) केवल 3

(d) केवल 1, 2 और 4

उत्तर:(b) 

व्याख्या

  • 2017 की नीति आयोग की रिपोर्ट, "रोज़गार और विकास के लिये भारत की ग्रामीण अर्थव्यवस्था की बदलती संरचना के निहितार्थ", ग्रामीण अर्थव्यवस्था के बारे में निम्नलिखित जानकारी प्रदान करती है।
  • ग्रामीण एवं शहरी दोनों क्षेत्रों के लिये श्रमिक उत्पादकता में वृद्धि हुई है। ग्रामीण क्षेत्रों के लिये यह वर्ष 2004-05 में 37273 रुपये तथा वर्ष 2011-12 में 101755 रुपये था जबकि शहरी क्षेत्रों के लिये यह वर्ष 2004-05 में 120419 रुपये और वर्ष 2011-12 में 282515 रुपये था। अतः कथन 1 सही नहीं है।
  • कुल कार्यबल में ग्रामीण हिस्सेदारी वर्ष 1999-2000 में 76.1% से घटकर वर्ष 2011-12 में 70.9% हो गई। अतः कथन 2 सही नहीं है।
  • ग्रामीण उत्पादन संरचना में महत्त्वपूर्ण परिवर्तनों में से एक गैर-कृषि क्षेत्र का बढ़ता हिस्सा है जो वर्ष 1980-81 में 37% से बढ़कर वर्ष 2009-10 में 65% हो गया और इस प्रकार यह दर्शाता है कि उत्पादन के मूल्य के संदर्भ में ग्रामीण अब केवल कृषि नहीं है। अतः कथन 3 सही है।
  • ग्रामीण रोज़गार ने पूर्व-सुधार अवधि के दौरान 2.16% वार्षिक वृद्धि दर दिखाई जो सुधार के बाद की अवधि में घटकर 1.45% हो गई और आर्थिक त्वरण की अवधि में नकारात्मक (-0.28%) हो गई। अतः कथन 4 सही है।
  • अत: विकल्प (b) सही उत्तर है।

59. निम्नलिखित कथनों पर विचार कीजिये-

  1. कृषि क्षेत्र को अल्पकालीन साख परिदान करने के संदर्भ में, ‘ज़िला केंद्रीय सहकारी बैंक (DCCBs)’ ‘अनुसूचित वाणिज्यिक बैंकों’ एवं ‘क्षेत्रीय ग्रामीण बैंकों’ की तुलना में अधिक ऋण देते हैं।
  2. डी.सी.सी.बी. (DCCBs) का एक सबसे प्रमुख कार्य ‘प्राथमिक कृषि साख समितियों’ को निधि उपलब्ध कराना है।

उपर्युक्त कथनों में से कौन-सा/से सही है/हैं?

(a) केवल 1

(b) केवल 2

(c) 1 और 2 दोनों

(d) न तो 1, न ही 2

उत्तर:(b)

व्याख्या

  • सहकारी बैंक सहकारी आधार पर स्थापित एक संस्था है और साधारण बैंकिंग व्यवसाय में काम करती है।
  • ग्रामीण भारत में एक 3-स्तरीय ग्रामीण सहकारी संरचना मौजूद है।
  • टियर- I: इसमें राज्य स्तर पर राज्य सहकारी बैंक (StCBs) शामिल हैं;
  • टियर- II: इसमें ज़िला स्तर पर केंद्रीय सहकारी बैंक (CCBs) शामिल हैं; और
  • टीयर- III: इसमें प्राथमिक कृषि ऋण समितियाँ (PACSs) शामिल हैं।
  • RBI की एक रिपोर्ट के अनुसार, वर्ष 2016-17 में अनुसूचित वाणिज्यिक बैंकों ने कृषि और संबद्ध ऋण में प्रमुख हिस्सेदारी (78-80%) का योगदान दिया। सहकारी संस्थाएँ भी कृषि ऋण देने में महत्त्वपूर्ण भूमिका निभाती हैं और सभी सहकारी बैंकों/संस्थानों (यानी StCBs, DCCBs और PACSs को मिलाकर) की हिस्सेदारी 15-16% होती है। RRB ने शेष 5% कृषि ऋण का योगदान दिया। अतः कथन 1 सही नहीं है।
  • ज़िला केंद्रीय सहकारी बैंक का सबसे महत्त्वपूर्ण कार्य ज़िले में इससे संबद्ध प्राथमिक सहकारी समितियों को वित्तीय सहायता प्रदान करना है। अतः कथन 2 सही है।
  • अत: विकल्प (b) सही उत्तर है।

60. भारत में, किसी व्यक्ति के साइबर बीमा कराने पर निधि की हानि की भरपाई एवं अन्य लाभों के अतिरिक्त सामान्यत: निम्नलिखित में से कौन-कौन से लाभ दिये जाते हैं?

  1. यदि कोई मालवेयर कंप्यूटर तक उसकी पहुँच बाधित कर देता है, तो कंप्यूटर प्रणाली को पुन: प्रचालित करने में लगने वाली लागत
  2. यदि यह प्रमाणित हो जाता है कि किसी शरारती तत्त्व द्वारा जान-बूझकर कंप्यूटर को नुकसान पहुँचाया गया है तो नए कंप्यूटर की लागत 
  3. यदि साइबर बलात्-ग्रहण होता है तो इस हानि को न्यूनतम करने के लिये विशेषज्ञ परामर्शदाता की सेवाएँ लेने पर लगने वाली लागत
  4. यदि कोई तीसरा पक्ष मुकदमा दायर करता है तो न्यायालय में बचाव करने में लगने वाली लागत

नीचे दिये गए कूट का प्रयोग कर सही उत्तर चुनिये-

(a) केवल 1, 2 और 4

(b) केवल 1, 3 और 4

(c) केवल 2 और 3

(d) 1, 2, 3 और 4

उत्तर:(b)

व्याख्या

  • साइबर बीमा को व्यवसायों को साइबर हमलों के संभावित प्रभावों से बचाने के लिये डिज़ाइन किया गया है। यह एक साइबर हमले/उल्लंघन के बाद लागत को ऑफसेट करके एक संगठन को जोखिम अनावरण को कम करने में मदद करता है अर्थात् साइबर बीमा को साइबर उल्लंघनों से संबंधी फीस, खर्च और कानूनी लागत को कवर करने के लिये डिज़ाइन किया गया है।
  • कवरेज में शामिल हैं:
    • उल्लंघन की घटनाओं पर प्रतिक्रिया (अधिसूचना, कॉल सेंटर सेवा, उल्लंघन समाधान, शमन सेवाएँ, जनसंपर्क और संकट प्रबंधन)।
    • वकीलों, पेशेवर फीस, प्रशासन लागत आदि सहित जाँच और जुर्माना।
    • फोरेंसिक, आईटी ऑडिट, संकट प्रबंधन, कानूनी लागत जैसे खर्च।
    • गोपनीयता और डेटा दायित्व
    • व्यक्तिगत पहचान योग्य जानकारी का नुकसान।
    • कॉर्पोरेट गोपनीय जानकारी का नुकसान।
    • नेटवर्क देयता जैसे DDoS अटैक।
    • कॉपीराइट मुद्दों सहित मल्टीमीडिया कवर।
    • आय हानि, व्यापार रुकावट लागत, सिस्टम क्षति और बहाली लागत, कोई अतिरिक्त खर्च।
    • साइबर थेफ्ट
    • फंड ट्रांसफर फ्रॉड
    • ई-थेफ्ट लॉस
    • ई-कम्युनिकेशन लॉस
    • साइबर क्राइम
  • अत: 1, 3 और 4 में बताए गए लाभ सही हैं। अत: विकल्प (b) सही है।

https://www.drishtiias.com/hindi/daily-updates/daily-news-editorials/economic-aspects-of-cyber-security

61. भारत में, निम्नलिखित में से किन्हें कृषि में सार्वजनिक निवेश माना जा सकता है।

  1. सभी फसलों के कृषि उत्पाद के लिये न्यूनतम समर्थन मूल्य निर्धारित करना
  2. प्राथमिक कृषि साख समितियों का कंप्यूटरीकरण 
  3. सामाजिक पूंजी विकास
  4. कृषकों को नि:शुल्क बिजली की आपूर्ति
  5. बैंकिंग प्रणाली द्वारा कृषि ऋण की माफी
  6. सरकारों द्वारा शीतागार सुविधाओं को स्थापित करना।

नीचे दिये गए कूट का प्रयोग कर सही उत्तर चुनिये-

(a) केवल 1, 2 और 5

(b) केवल 1, 3, 4 और 5

(c) केवल 2, 3 और 6

(d) 1, 2, 3, 4, 5 और 6

उत्तर : (c)

व्याख्या

  • सार्वजनिक निवेश राज्य (केंद्र, राज्य और स्थानीय सरकारों या सार्वजनिक स्वामित्व वाली कंपनियों के माध्यम से) द्वारा आधारभूत भौतिक अवसंरचना (जैसे सड़कों, पुलों, रेल लाइनों, हवाई अड्डों तथा जल वितरण),  नवीन गतिविधि (मूल अनुसंधान), हरित निवेश (स्वच्छ ऊर्जा स्रोत) और शिक्षा जो उच्च उत्पादकता और/या उच्च जीवन स्तर की ओर ले जाती है, के लिये संसाधनों को समर्पित करके देश के पूंजीगत स्टॉक का निर्माण करने के लिये एक निवेश है।
  • न्यूनतम समर्थन मूल्य (MSP) किसानों को कीमतों में गिरावट से बचाने के लिये उन्हें बीमा पॉलिसी के रूप में काम करने के लिये एक बाज़ार व्यतिकरण तंत्र प्रदान करता है। यह कृषि क्षेत्र में उत्पादकता बढ़ाने में मदद नहीं कर रहा है। इस प्रकार सभी फसलों की कृषि उपज के लिये MSP तय करने से राष्ट्रीय पूंजी स्टॉक में वृद्धि नहीं होती है या उच्च उत्पादकता नहीं होती है। अतः कथन 1 सही नहीं है।
  • प्राथमिक कृषि साख समितियों के कम्प्यूटरीकरण से कृषि क्षेत्र में उत्पादकता बढ़ेगी क्योंकि ऋण की आसान और समय पर पहुँच होगी। इसी तरह कोल्ड स्टोरेज सुविधाओं की स्थापना का भी प्रभाव होगा। अतः कथन 2 और 6 सही हैं।
  • सामाजिक पूंजी वह मूल्य है जो नेटवर्किंग तथा लोगों और संगठनों के बीच निर्मित विश्वास के माध्यम से निर्मित की जा सकती है। एक समुदाय के भीतर सामंजस्य बनाने से एक साथ काम करने की लेन-देन की लागत कम हो जाती है तथा बढ़ा हुआ विश्वास समुदायों को सामाजिक दुविधाओं को दूर करने में सक्षम बना सकता है। मज़बूत सामाजिक नेटवर्क संसाधन गरीब व्यक्तियों या समुदायों को नुकसान से निपटान में मदद करने के लिये सुरक्षा जाल के रूप में काम कर सकते हैं, विशेषत: जब औपचारिक प्रकार के जोखिम प्रबंधन जैसे कि क्रेडिट या बीमा अनुपलब्ध हो। कृषक समुदायों के भीतर सामाजिक पूंजी भी उत्पादकता में सुधार कर सकती है क्योंकि यह प्राकृतिक संसाधनों के प्रबंधन या नई प्रथाओं और प्रौद्योगिकियों को अपनाने के लिये एक पूर्व-आवश्यकता है। छोटे धारकों के लिये सामाजिक पूंजी का निर्माण भी नई तकनीकों को अपनाने पर सकारात्मक प्रभाव डाल सकता है जैसे कि उन्नत बीजों का उपयोग, मृदा और जल संरक्षण प्रथाओं और कृषि वानिकी। अतः कथन 3 सही है।
  • किसानों को मुफ्त बिजली, राज्य पर वित्तीय दबाव के अलावा, पानी के अत्यधिक उपयोग, अधिक पंप सेटों की स्थापना को प्रोत्साहित करने और भूजल स्तर को कम करने के परिणामस्वरूप हुई है। अतः कथन 4 सही नहीं है।
  • कृषि ऋण माफी निवेश नहीं है क्योंकि यह बैंकिंग क्षेत्र के स्वास्थ्य को प्रभावित करता है। अतः कथन 5 सही नहीं है।

अत: विकल्प (c) सही उत्तर है।

https://www.drishtiias.com/hindi/summary-of-important-reports/doubling-the-farmers-income-niti-aayog 

62. भारत में फर्म के ‘ब्याज-व्याप्ति अनुपात (Interest Coverage Ratio)’ पद का क्या महत्त्व है? 

  1. यह उस फर्म, जिसे बैंक ऋण देने जा रहा है, के वर्तमान जोखिम को समझने में मदद करता है।
  2. यह उस फर्म, जिसे बैंक ऋण देने जा रहा है, के आने वाले जोखिम के मूल्यांकन में मदद करता है।
  3. उधार लेने वाली फर्म का ब्याज-व्याप्ति अनुपात जितना अधिक होगा, उसकी ऋण समाशोधन क्षमता उतनी खराब होगी।

नीचे दिये गए कूट का प्रयोग कर सही उत्तर चुनिये-

(a) केवल 1 और 2

(b) केवल 2

(c) केवल 1 और 3

(d) 1, 2 और 3

उत्तर : (a)

व्याख्या

  • ब्याज-व्याप्ति अनुपात (Interest Coverage Ratio) एक ऋण अनुपात और लाभप्रदता अनुपात है जिसका उपयोग यह निर्धारित करने के लिये किया जाता है कि कोई कंपनी अपने बकाया ऋण पर कितनी आसानी से ब्याज का भुगतान कर सकती है। यह मापता है कि कोई कंपनी अपनी उपलब्ध कमाई के साथ अपने मौजूदा ब्याज भुगतान को कितनी बार कवर कर सकती है। इसकी गणना एक निश्चित अवधि के दौरान कंपनी के ब्याज तथा करों (EBIT) से पहले की कमाई को उसी अवधि के भीतर कंपनी के ब्याज भुगतान से विभाजित करके की जा सकती है। अतः कथन 1 सही है।
  • ICR का उपयोग सामान्यत: ऋणदाताओं, लेनदारों और निवेशकों द्वारा कंपनी के वर्तमान ऋण या भविष्य के उधार के सापेक्ष जोखिम का निर्धारण करने के लिये किया जाता है। अतः कथन 2 सही है।
  • ब्याज-व्याप्ति अनुपात जितना अधिक होगा उतना बेहतर होगा। अनुपात जितना कम होगा कंपनी पर ऋण व्यय का उतना ही अधिक बोझ होगा। जब किसी कंपनी का ब्याज-व्याप्ति अनुपात केवल 1.5 या उससे कम होता है तो ब्याज व्ययों को पूरा करने की उसकी क्षमता संदिग्ध हो सकती है। अतः कथन 3 सही नहीं है।
  • अत: विकल्प (a) सही है।

63. हाल के बीते दिनों में निम्नलिखित में से कौन-से कारक/कौन-सी नीतियाँ भारत में चावल के मूल्य को प्रभावित कर रही थीं?

  1. न्यूनतम समर्थन मूल्य
  2. सरकार द्वारा व्यापार करना
  3. सरकार द्वारा भंडारण करना
  4. उपभोक्ता सहायिकियाँ (subsidies)

नीचे दिये गए कूट का प्रयोग कर सही उत्तर चुनिये-

(a) केवल 1, 2 और 4

(b) केवल 1, 3 और 4

(c) केवल 2 और 3

(d) 1, 2, 3 और 4

उत्तर : (d)

व्याख्या

  • न्यूनतम समर्थन मूल्य (MSP) सरकार द्वारा तय किया गया एक न्यूनतम मूल्य है जिस पर सरकार किसानों से फसल खरीदती है। MSP का उद्देश्य किसानों को अच्छी फसल के मौसम में कृषि कीमतों में तेज़ गिरावट से बचाना है। हालाँकि किसी भी फसल पर निर्धारित MSP किसानों को उस विशिष्ट फसल का उत्पादन करने के लिये आकर्षित करता है जिससे उसका अधिक उत्पादन हो सकता है। MSP भी बाज़ार को विकृत करता है क्योंकि सरकारी खरीद एजेंसियाँ निजी कंपनियों को बाहर कर 70-80% चावल खरीदती हैं। यदि सरकार बाज़ार दर से अधिक MSP निर्धारित करती है तो निजी कंपनियाँ किसानों से फसल नहीं खरीदेंगे तथा फसल सरकार द्वारा खरीदी जाती है। जो बाज़ारों में अक्षमता उत्पन्न करता है। किसान आमतौर पर उन फसलों का उत्पादन करना पसंद नहीं करते हैं जिन पर कोई निश्चित MSP नहीं होता है जिससे अंततः उन फसलों की कीमत बढ़ जाती है। अत: कारक/नीति 1 सही है।
  • भारत में सरकार प्राय: देश के चावल उत्पादन का एक तिहाई से अधिक निश्चित मूल्य पर खरीदती है जो चावल की कीमत पर सीधा प्रभावित करती है। अत: कारक/नीति 2 सही है।
  • भारत में सरकार गरीबों को वितरण के लिये गेहूँ और चावल जैसे अनाज का भंडारण करती है जो चावल की कीमत को प्रभावित करती है। सरकार उपार्जित फसलों को सार्वजनिक वितरण प्रणाली के माध्यम से रियायती दरों पर बेचती है। अत: कारक/नीति 3 सही है।
  • उपभोक्ता सहायिकी (subsidy) का मतलब है कि सरकार लागत का हिस्सा चुकाती है। यह निर्माता और उपभोक्ता दोनों को प्रदान किया जा सकता है। भारत में उपभोक्ता सहायिकी वाले खाद्यान्न TPDS के माध्यम से वितरित किये जाते हैं जो उचित मूल्य की दुकानों के माध्यम से बेचे जाने वाले खाद्यान्नों की कीमत को प्रभावित करते हैं। अत: कारक/नीति 4 सही है।
  • अत: विकल्प (d) सही उत्तर है।

64. निम्नलिखित कथनों पर विचार कीजिये-

  1. पिछले दशक में भारत-श्रीलंका व्यापार के मूल्य में सतत् वृद्धि हुई है।
  2. भारत और बांग्लादेश के बीच होने वाले व्यापार में ‘कपड़े और कपड़े से बनी चीज़ों’ का व्यापार प्रमुख है। 
  3. पिछले पाँच वर्षों में, दक्षिण एशिया में भारत के व्यापार का सबसे बड़ा भागीदार नेपाल रहा है। 

उपर्युक्त कथनों में से कौन-सा/से सही है/हैं?

(a) केवल 1 और 2

(b) केवल 2

(c) केवल 3

(d) 1, 2 और 3

उत्तर : (b)

व्याख्या

  • वाणिज्य विभाग के आंँकड़ों के अनुसार, एक दशक (वर्ष 2007 से वर्ष 2016) के लिये भारत-श्रीलंका द्विपक्षीय व्यापार मूल्य क्रमशः 3.0, 3.4, 2.1, 3.8, 5.2, 4.5, 5.3, 7.0, 6.3, 4.8 (अरब अमेरिकी डाॅलर में) था जो व्यापार मूल्य की प्रवृत्ति में निरंतर उतार-चढ़ाव को दर्शाता है। समग्र वृद्धि के बावजूद इसे व्यापार मूल्य में लगातार वृद्धि के रूप में नहीं कहा जा सकता है। अतः कथन 1 सही नहीं है।
  • निर्यात में 5% से अधिक और आयात में 7% से अधिक की हिस्सेदारी के साथ बांग्लादेश, भारत के लिये एक प्रमुख कपड़ा व्यापार भागीदार देश रहा है। भारत का बांग्लादेश को सालाना कपड़ा निर्यात औसतन 2,000 मिलियन डॉलर और आयात 400 डॉलर (वर्ष 2016-17) का है। अत: कथन 2 सही है।
  • आंँकड़ों के अनुसार, वर्ष 2016-17 में बांग्लादेश, दक्षिण एशिया में भारत का सबसे बड़ा व्यापारिक भागीदार देश है, इसके बाद नेपाल, श्रीलंका, पाकिस्तान, भूटान, अफगानिस्तान और मालदीव का स्थान है। भारतीय निर्यात का स्तर भी इसी क्रम का अनुसरण करता है। अत: कथन 3 सही नहीं है।
  • अतः विकल्प (B) सही है।

 65. निम्नलिखित में से किस समूह के सभी चारों देश G20 के सदस्य हैं?

(a) अर्जेंटीना, मेक्सिको, दक्षिण अफ्रीका एवं तुर्की

(b) ऑस्ट्रेलिया, कनाडा, मलेशिया एवं न्यूज़ीलैंड

(c) ब्राज़ील, ईरान, सऊदी अरब एवं वियतनाम

(d) इंडोनेशिया, जापान, सिंगापुर एवं दक्षिण कोरिया

उत्तर : (a)

व्याख्या

  • G-20 अंतर्राष्ट्रीय मुद्रा कोष और विश्व बैंक के प्रतिनिधियों के साथ 19 देशों तथा यूरोपीय संघ का एक अनौपचारिक समूह है।
  • मज़बूत वैश्विक आर्थिक विकास के लिये सदस्य देश जो वैश्विक सकल घरेलू उत्पाद का 80% से अधिक का प्रतिनिधित्व और योगदान करते हैं, अंतर्राष्ट्रीय आर्थिक सहयोग के लिये प्रमुख मंच पर आए, जिस पर सितंबर 2009 में पेंसिल्वेनिया (USA) में पिट्सबर्ग शिखर सम्मेलन में नेताओं द्वारा सहमति व्यक्त की गई थी।
  • G-20 के सदस्यों में अर्जेंटीना, ऑस्ट्रेलिया, ब्राज़ील, कनाडा, चीन, फ्रांँस, जर्मनी, भारत, इंडोनेशिया, इटली, जापान, मैक्सिको, कोरिया गणराज्य, रूस, सऊदी अरब, दक्षिण अफ्रीका, तुर्की, यूनाइटेड किंगडम, संयुक्त राज्य अमेरिका और यूरोपीय संघ (EU) शामिल हैं।
  • अतः विकल्प (a) सही है।

https://www.drishtiias.com/hindi/daily-updates/daily-news-analysis/g20-ministerial-meeting

66. ‘किसान क्रेडिट कार्ड’ योजना के अंतर्गत, निम्नलिखित में से किन-किन उद्देश्यों के लिये कृषकों को अल्पकालीन ऋण समर्थन उपलब्ध कराया जाता है?

  1. फार्म परिसंपत्तियों के रख-रखाव हेतु कार्यशील पूंजी के लिये
  2. कंबाइन कटाई मशीनों, ट्रैक्टरों एवं मिनी ट्रकों के क्रय के लिये
  3. फार्म परिवारों की उपभोग आवश्यकताओं के लिये
  4. फसल कटाई के बाद खर्चों के लिये
  5. परिवार के लिये घर निर्माण तथा गाँव में शीतागार सुविधा की स्थापना के लिये।

नीचे दिये गए कूट का प्रयोग कर सही उत्तर चुनिये

(a) केवल 1, 2 और 5

(b) केवल 1, 3 और 4

(c) केवल 2, 3, 4 और 5

(d) 1, 2, 3, 4 और 5

उत्तर : (b)

व्याख्या

  • किसान क्रेडिट कार्ड (KCC) योजना को वर्ष 1998 में किसानों को उनकी खेती और बीज, उर्वरकों, कीटनाशकों आदि जैसे कृषि आदानों की खरीद तथा उनकी उत्पादन आवश्यकताओं के लिये नकदी निकालने जैसी अन्य आवश्यकताओं के लिये लचीली एवं सरलीकृत प्रक्रिया के साथ एक एकल खिड़की के तहत बैंकिंग प्रणाली से पर्याप्त तथा समय पर ऋण सहायता प्रदान करने हेतु शुरू किया गया था।
  • किसानों की निवेश ऋण आवश्यकताओं जैसे संबद्ध और गैर-कृषि गतिविधियों के लिये वर्ष 2004 में इसका विस्तार किया गया गया था।
  • किसान क्रेडिट कार्ड निम्नलिखित उद्देश्यों के साथ प्रदान किया जाता है:
    • फसलों के लिये अल्पकालिक ऋण आवश्यकता की पूर्ति
    • फसल के बाद का खर्च; 
    • विपणन ऋण का उत्पादन
    • किसान परिवार की खपत की आवश्यकताएंँ; 
    • कृषि परिसंपत्तियों और कृषि से संबंधित गतिविधियों, जैसे- डेयरी पशु, अंतर्देशीय मत्स्य पालन आदि के रखरखाव के लिये कार्यशील पूंजी, 
    • कृषि और संबद्ध गतिविधियों जैसे- पंपसेट, स्प्रेयर, डेयरी पशु, आदि के लिये निवेश ऋण की आवश्यकता। हालाँकि यह खंड दीर्घकालिक ऋण का है।
    • किसान क्रेडिट कार्ड योजना वाणिज्यिक बैंकों, आरआरबी, लघु वित्त बैंकों और सहकारी समितियों द्वारा कार्यान्वित की जाती है।
    • किसानों को कंबाइन हार्वेस्टर, ट्रैक्टर और मिनी ट्रक की खरीद एवं परिवार के घर के निर्माण और गाँव में कोल्ड स्टोरेज सुविधा की स्थापना के लिये अल्पकालिक ऋण सहायता नहीं दी जाती है। अतः विकल्प (b) सही उत्तर है।

https://www.drishtiias.com/hindi/daily-updates/daily-news-analysis/kisan-credit-card

67. निम्नलिखित कथनों पर विचार कीजिये-

  1. खाद्य वस्तुओं का ‘उपभोक्ता मूल्य सूचकांक’ (CPI) में भार (Weightage) उनके ‘थोक मूल्य सूचकांक’ (WPI) में दिये गए भार से अधिक है।
  2. WPI, सेवाओं के मूल्यों में होने वाले परिवर्तनों को नहीं पकड़ता, जैसा कि CPI करता है।
  3. भारतीय रिज़र्व बैंक ने अब मुद्रास्फीति के मुख्य मान हेतु तथा प्रमुख नीतिगत दरों के निर्धारण और परिवर्तन हेतु WPI को अपना लिया है।

उपर्युक्त कथनों में से कौन-सा/से सही है/हैं?

(a) केवल 1 और 2

(b) केवल 2

(c) केवल 3

(d) 1, 2 और 3

उत्तर : (a)

व्याख्या

  • थोक मूल्य सूचकांक (WPI) थोक बाज़ार या थोक स्तर पर वस्तुओं की कीमतों में औसत परिवर्तन का माप है। यह आर्थिक सलाहकार कार्यालय, वाणिज्य और उद्योग मंत्रालय द्वारा प्रकाशित किया जाता है।
  • उपभोक्ता मूल्य सूचकांक (CPI) परिवारों द्वारा खरीदी गई उपभोक्ता वस्तुओं और सेवाओं की एक बास्केट के मूल्य स्तर में बदलाव का माप है। आइटम बास्केट के आधार पर CPI के चार प्रकार हैं:
    • औद्योगिक श्रमिकों के लिये CPI (IW)
    • कृषि मज़दूर के लिये CPI (AL)
    • ग्रामीण मज़दूरों के लिये CPI (RL)
    • CPI (ग्रामीण/शहरी/संयुक्त)
  • इनमें से पहले तीन को श्रम और रोज़गार मंत्रालय के श्रम ब्यूरो द्वारा संकलित किया गया है। चौथा सांख्यिकी और कार्यक्रम कार्यान्वयन मंत्रालय में केंद्रीय सांख्यिकी संगठन (CSO) द्वारा संकलित किया गया है।
  • CPI में वस्तुओं का भार उपभोक्ता व्यय सर्वेक्षणों से लिये गए औसत घरेलू व्यय पर आधारित है। CPI में भोजन का भार WPI (लगभग 24%) की तुलना में कहीं अधिक (लगभग 46%) है। WPI आइटम बास्केट का एक महत्त्वपूर्ण अनुपात विनिर्माण आदानों और मध्यवर्ती वस्तुओं जैसे खनिज, मूल धातु, मशीनरी आदि का प्रतिनिधित्व करता है। अत: कथन 1 सही है।
  • इसके अलावा, WPI सेवाओं की कीमतों में परिवर्तन पर नियंत्रण नहीं करता है जो CPI करता है। अतः कथन 2 सही है।
  • WPI का उपयोग कुछ अर्थव्यवस्थाओं में मुद्रास्फीति के प्रमुख उपाय के रूप में किया जाता है। हालाँकि RBI अब रेपो दरों को निर्धारित करने सहित नीतिगत उद्देश्यों के लिये इसका उपयोग नहीं करता है। अप्रैल 2014 में, RBI ने मौद्रिक और क्रेडिट नीति निर्धारित करने के लिये मुद्रास्फीति के एक प्रमुख उपाय के रूप में CPI या खुदरा मुद्रास्फीति को अपनाया। अतः कथन 3 सही नहीं है।
  • अत: विकल्प (a) सही उत्तर है।

https://www.drishtiias.com/hindi/printpdf/inflation-in-india

https://www.drishtiias.com/hindi/daily-updates/daily-news-analysis/industrial-growth-rate-and-consumer-price-index-signs-of-growth 

68. निम्नलिखित युग्मों पर विचार कीजिये-

नदी           में जाकर मिलती है

  1. मेकॉन्ग - अंडमान सागर
  2. थेम्स - आयरिश सागर
  3. वोल्गा - कैस्पियन सागर
  4. ज़म्बेज़ी - हिंद महासागर

उपर्युक्त युग्मों में से कौन-सा/से सही सुमेलित है/हैं?

(a) केवल 1 और 2

(b) केवल 3

(c) केवल 3 और 4

(d) केवल 1, 2 और 4

उत्तर : (c)

व्याख्या

  • मेकॉन्ग नदी, तिब्बती हाइलैंड्स के बर्फीले हेडवाटर में उत्पन्न होती है, वियतनाम में एक विस्तारित डेल्टा बनाने और दक्षिण चीन सागर में खाली करने से पहले, निचले बेसिन देशों म्याँमार, लाओस, थाईलैंड और कंबोडिया के माध्यम से, चीन की खड़ी घाटी से बहती है जिसे ऊपरी बेसिन के रूप में जाना जाता है। अतः युग्म 1 सही सुमेलित नहीं है।
  • इंग्लैंड की सबसे लंबी नदी टेम्स नदी कॉटस्वॉल्ड से उत्तरी सागर तक 215 मील बहती है। टेम्स की मुख्य सहायक नदियाँ बस्कट, रीडिंग और किंग्स्टन हैं। अतः युग्म 2 सही सुमेलित नहीं है।
  • वोल्गा नदी, यूरोप की सबसे लंबी नदी है, जो कज़ाखस्तान की सीमा के दक्षिण में कैस्पियन सागर में बहने वाले डेल्टा के साथ रूस से होकर गुजरती है। अतः युग्म 3 सही सुमेलित है।
  • ज़म्बेज़ी अफ्रीका में कांगो/ज़ैरे, नील और नाइजर के बाद चौथी सबसे बड़ी नदी है। यह उत्तर-पश्चिमी ज़ाम्बिया में कालेन पहाड़ियों से निकलती है तथा हिंद महासागर में लगभग 3000 किलोमीटर तक पूर्व की ओर बहती है। अतः युग्म 4 सही सुमेलित है।
  • अत: विकल्प (c) सही उत्तर है।

69. निम्नलिखित कथनों पर विचार कीजिये-

  1. सभी अनाजों, दालों एवं तिलहनों का ‘न्यूनतम समर्थन मूल्य’ (MSP) पर प्रापण (खरीद) भारत के किसी भी राज्य/केंद्र-शासित प्रदेश (यू.टी.) में असीमित होता है
  2. अनाजों एवं दालों का MSP किसी भी राज्य/केंद्र-शासित प्रदेश में उस स्तर पर निर्धारित नहीं किया जाता है जिस स्तर पर बाज़ार मूल्य कभी नहीं पहुँच पाते।

उपर्युक्त कथनों में से कौन-सा/से सही है/हैं?

(a) केवल 1

(b) केवल 2

(c) 1 और 2 दोनों

(d) न तो 1, न ही 2

उत्तर : (d)

व्याख्या

  • भारत सरकार कृषि लागत और मूल्य आयोग (CACP) की सिफारिशों को ध्यान में रखते हुए प्रत्येक वर्ष दोनों फसल मौसमों में 22 प्रमुख कृषि वस्तुओं के लिये न्यूनतम समर्थन मूल्य (MSP) की घोषणा करती है
  • CACP 22 फसलों के लिए MSP तथा गन्ने के लिए उचित और लाभकारी मूल्य (FRP) की सिफारिश करता है। 
  • MSP द्वारा कवर की जाने वाली फसलों में शामिल हैं:
    • 7 प्रकार के अनाज (धान, गेहूँ, मक्का, बाजरा, ज्वार, रागी और जौ)।
    • 5 प्रकार की दालें (चना, अरहर/तूर, उड़द, मूँग और मसूर)।
    • 7 तिलहन (रेपसीड-सरसों, मूँगफली, सोयाबीन, सूरजमुखी, तिल, कुसुम, नाइज़रसीड)।
    • 4 व्यावसायिक फसलें (कपास, गन्ना, खोपरा, कच्चा जूट)।
  • खाद्य और सार्वजनिक वितरण विभाग गन्ना के लिए उचित और लाभकारी मूल्य (FRP) घोषित करता है।
  • कुल खरीद मात्रा आमतौर पर उस विशेष वर्ष/मौसम के लिये वस्तु के वास्तविक उत्पादन के 25% से अधिक नहीं होनी चाहिये। 25% की सीमा से अधिक खरीद के लिये कृषि विभाग (DAC) के पूर्व अनुमोदन की आवश्यकता होगी। अतः कथन 1 सही नहीं है। 
  • MSP को विभिन्न राज्यों द्वारा दिये गए MSP प्रस्तावों के औसत के आधार पर केंद्र सरकार द्वारा तय किया जाता है, जिनमें से कुछ प्रस्ताव केंद्र की सिफारिश से अधिक हो सकते हैं। जबकि इनपुट लागत पर आधारित प्रस्ताव अलग-अलग राज्यों में अलग-अलग होते हैं, मूल्य असमानता से बचने के लिये MSP को तय किया जाता है। जब बज़ाार की कीमतें MSP से नीचे के स्तर तक गिर जाती हैं, तो सरकारी एजेंसियाँ किसानों की सुरक्षा के लिये उपज को खरीद लेती हैं। ऐसे में बाज़ार में कीमतें MSP से ऊपर जा सकती हैं।
  • अतः कथन 2 सही नहीं है। अतः विकल्प D सही है।

70. भारतीय अर्थव्यवस्था के संदर्भ में निम्नलिखित कथनों पर विचार कीजिये-

  1. ‘वाणिज्यिक पत्र (Commercial Paper)’ अल्पकालीन प्रतिभूति-रहित वचन-पत्र है।
  2. ‘जमा प्रमाण-पत्र (Certificate of Deposit)’ भारतीय रिज़र्व बैंक द्वारा किसी निगम को निर्गत किया जाने वाला दीर्घकालीन प्रपत्र है।
  3. ‘शीघ्रावधि द्रव्य (Call Money)’ अंतरबैंक लेन-देनों के लिये प्रयुक्त अल्प अवधि का वित्त है।
  4. ‘शून्य-कूपन बॉण्ड (Zero-Coupon Bonds)’ अनुसूचित व्यापारिक बैंकों द्वारा निगमों को निर्गत किये जाने वाले ब्याज सहित अल्पकालीन बॉण्ड हैं।

उपर्युक्त कथनों में से कौन-सा/से सही है/हैं?

(a) केवल 1 और 2

(b) केवल 4

(c) केवल 1 और 3

(d) केवल 2, 3 और 4

उत्तर : (c)

व्याख्या

  • वाणिज्यिक पत्र  (CP) वचन पत्र के रूप में जारी किया गया एक असुरक्षित मुद्रा बाज़ार साधन है और SEBI द्वारा अनुमोदित और पंजीकृत किसी भी डिपॉजिटरी के माध्यम से अभौतिक रूप में जारी किया गया है। अतः कथन 1 सही है।
  • जमा प्रमाणपत्र (सीडी) कम जोखिम वाले निवेश हैं जो आपके द्वारा जमा किए गए धन पर ब्याज देते हैं। बैंक और क्रेडिट यूनियन सीडी जारी करते हैं। CD जारी किये जा सकते हैं; (i) क्षेत्रीय ग्रामीण बैंकों (RRB) और स्थानीय क्षेत्र के बैंकों (LAB) को छोड़कर अनुसूचित वाणिज्यिक बैंकों द्वारा, (ii) अखिल भारतीय वित्तीय संस्थानों (FIs) का चयन करके RBI द्वारा निर्धारित सीमा के भीतर अल्पकालिक संसाधन एकत्र करने के लिये RBI द्वारा। अतः कथन 2 सही नहीं है।
  • शीघ्रावधि द्रव्य (Call Money) एक वित्तीय संस्थान द्वारा दूसरे वित्तीय संस्थान को दिया गया 1 से 14 दिनों का अल्पकालिक, ब्याज-भुगतान ऋण है। अतः कथन 3 सही है।
  • शून्य-कूपन बॉण्ड (Zero-Coupon Bonds) एक ऋण सुरक्षा है जो ब्याज का भुगतान नहीं करता है बल्कि एक भारी छूट पर व्यापार करता है, परिपक्वता पर लाभ प्रदान करता है, जब बॉण्ड को उसके पूर्ण अंकित मूल्य के लिये भुनाया जाता है। अतः कथन 4 सही नहीं है।

अत: विकल्प (c) सही उत्तर है।

71. भारत के सांस्कृतिक इतिहास के संदर्भ में, निम्नलिखित युग्मों पर विचार कीजिये-

  1. परिव्राजक - परित्यागी व भ्रमणकारी
  2. श्रमण - उच्च पद प्राप्त पुजारी
  3. उपासक - बौद्ध धर्म का साधारण अनुगामी

उपर्युक्त युग्मों में से कौन-से सही सुमेलित हैं?

(a) केवल 1 और 2

(b) केवल 1 और 3

(c) केवल 2 और 3

(d) 1, 2 और 3

उत्तर:(b)

व्याख्या

  • यात्रा करने वाले भिक्षुकों को सामान्यत: परिव्राजक कहा जाता था। वे सत्य के साधक थे जो किसी एक स्थान पर स्थायी रूप से निवास नहीं करते थे, एक स्थान से दूसरे स्थान पर भटकते रहते थे। अतः युग्म 1 सही सुमेलित है।
  • संस्कृत में श्रमण का अर्थ है "वह जो प्रयास करता है" अर्थात् सत्य का एहसास करना। श्रमणों ने आध्यात्मिक मुक्ति की खोज में एक तपस्वी या सख्त और आत्म-वंचना, जीवन शैली का अभ्यास किया। वे आमतौर पर भिक्षुओं के रूप में जाने जाते थे। अतः युग्म 2 सही सुमेलित नहीं है।
  • उपासक संस्कृत का शब्द है और पाली भाषा में "परिचारक" के लिये उपयोग किया जाता है। यह बौद्ध धर्म के अनुयायियों (या ऐतिहासिक रूप से, गौतम बुद्ध के) का शीर्षक है जो बौद्ध क्रम में भिक्षु, नन या नौसिखिये मठवासी नहीं हैं तथा जो कुछ व्रतों का पालन करते हैं। अतः युग्म 3 सही सुमेलित है।
  • अत: विकल्प (b) सही है।

72. भारतीय हाथियों के संदर्भ में निम्नलिखित कथनों पर विचार कीजिये: (2020)

  1. हाथियों के समूह का नेतृत्व मादा करती है।
  2. हाथी की अधिकतम गर्भावधि 22 माह तक हो सकती है।
  3. सामान्यत: हाथी में 40 वर्ष की आयु तक ही संतति पैदा करने की क्षमता होती है।
  4. भारत के राज्यों में हाथियों की सर्वाधिक जीव संख्या केरल में है।

उपर्युक्त कथनों में से कौन-सा/से सही है/हैं?

(a) केवल 1 और 2

(b) केवल 2 और 4

(c) केवल 3

(d) केवल 1, 3 और 4

उत्तर:(a)

व्याख्या

  • हाथी के झुंड का नेतृत्व सबसे पुरानी और सबसे बड़ी उम्र की मादा करती है (मातृसत्ता के रूप में प्रचलित)। अतः कथन 1 सही है।
  • हाथियों की सभी स्तनधारियों में सबसे लंबी ज्ञात गर्भावधि होती है जो 680 दिन (22 माह) तक चलती है। अतः कथन 2 सही है।
  • 14 से 45 वर्ष के बीच की मादा लगभग प्रत्येक चार वर्ष में संतति को जन्म दे सकती हैं, औसत अंतर (जन्म अंतराल) 52 वर्ष की उम्र तक पाँच वर्ष और 60 वर्ष की उम्र तक छह वर्ष तक बढ़ जाता है। अत: कथन 3 सही नहीं है।
  • हाथियों की जनगणना (वर्ष 2017) के अनुसार, कर्नाटक में हाथियों की संख्या सबसे अधिक (6,049) है, इसके बाद असम (5,719) और केरल (3,054) का स्थान है। अतः कथन 4 सही नहीं है।
  • अत: विकल्प (A) सही उत्तर है।

73. निम्नलिखित में से कौन-सा ‘संरक्षित क्षेत्र’ कावेरी बेसिन में स्थित है?

  1. नागरहोले राष्ट्रीय उद्यान
  2. पापिकोंडा राष्ट्रीय उद्यान
  3. सत्यमंगलम बाघ आरक्षित क्षेत्र
  4. वायनाड वन्यजीव अभयारण्य

नीचे दिये गए कूट का प्रयोग कर सही उत्तर चुनिये-

(a) केवल 1 और 2

(b) केवल 3 और 4

(c) केवल 1, 3 और 4

(d) 1, 2, 3 और 4

उत्तर:(c)

व्याख्या

  • नागरहोल राष्ट्रीय उद्यान:
    • इस उद्यान को राजीव गांधी राष्ट्रीय उद्यान के नाम से भी जाना जाता है। यह कर्नाटक के दो ज़िलों अर्थात् मैसूर और कोडागु में स्थित है।
    • इस उद्यान को वर्ष 1955 में एक वन्यजीव अभ्यारण्य के रूप में स्थापित किया गया था और वर्ष 1988 में इसे एक राष्ट्रीय उद्यान में अपग्रेड किया गया था।
    • इस उद्यान को वर्ष 1999 में 37वें टाइगर रिज़र्व के रूप में घोषित किया गया था।
    • कावेरी नदी की एक सहायक नदी काबिनी, इस उद्यान से निकलने वाली सबसे बड़ी नदी है। अतः विकल्प 1 सही है।
  • पापिकोंडा राष्ट्रीय उद्यान:
    • यह उद्यान आंध्रप्रदेश के पूर्व और पश्चिम गोदावरी ज़िलों में 1012.86 वर्ग किलोमीटर में फैला हुआ है।
    • इसने ऐतिहासिक रूप से सुरक्षा के विभिन्न स्तरों का अनुभव किया है जिसकी शुरुआत वर्ष 1882 में एक आरक्षित वन, वर्ष 1978 में एक वन्यजीव अभयारण्य और वर्ष 2008 से एक राष्ट्रीय उद्यान के रूप में हुई थी।
    • यह उद्यान गोदावरी नदी के बाएँ तथा दाएँ किनारे पर स्थित है और पूर्वी घाट की पापिकोंडा पहाड़ी शृंखला से होकर गुजरता है। अतः विकल्प 2 सही नहीं है।
  • सत्यमंगलम टाइगर रिज़र्व:
    • सत्यमंगलम वन्यजीव अभ्यारण्य और टाइगर रिज़र्व तमिलनाडु राज्य के इरोड ज़िले में पश्चिमी घाट के साथ एक संरक्षित क्षेत्र और टाइगर रिज़र्व है।
    • इरोड ज़िले के उत्तरी भाग में पलार नदी बहती है और कावेरी नदी में मिल जाती है।
    • अतः विकल्प 3 सही है।
  • वायनाड वन्यजीव अभ्यारण्य:
    • केरल में स्थित, वायनाड वन्यजीव अभ्यारण्य (WWS) नीलगिरी बायोस्फीयर रिज़र्व का एक अभिन्न अंग है। इसकी स्थापना वर्ष 1973 में हुई थी।
    • यह कर्नाटक के नागरहोल तथा बांदीपुर और तमिलनाडु के मुदुमलाई के बाघ अभ्यारण्यों के निकट है।
    • काबिनी नदी (कावेरी नदी की एक सहायक नदी) अभयारण्य से होकर बहती है। अतः विकल्प 4 सही है।
  • अत: विकल्प (c) सही है।

74. भारत की जैव-विविधता के संदर्भ में सीलोन फ्रॉगमाउथ, कॉपरस्मिथ बार्बेट, ग्रे-चिन्ड मिनिवेट और ह्वाइट-थ्रोटेड रेडस्टार्ट क्या है?

(a) पक्षी

(b) प्राइमेट

(c) सरीसृप

(d) उभयचर

उत्तर:(a)

व्याख्या

  • ये सभी एवियन इकोलॉजी का हिस्सा हैं।
  • सीलोन फ्रॉगमाउथ:
    • यह भूरे रंग की रात्रिचर पक्षी प्रजाति है जो पश्चिमी घाट और श्रीलंका के वनों में पाई जाती है।
    • यह अपनी चौड़ी, झुकी हुई चोंच से पहचाना जाता है जिसमें भट्ठा जैसे नथुने और आगे की ओर आँखों वाला एक बड़ा सिर होता है।
    • वनों के विनाश के साथ, यह प्रजाति अब काजू के बागानों में शरण लेने के लिये मज़बूर है।
  • कॉपरस्मिथ बार्बेट:
    • इसे क्रिमसन-ब्रेस्टेड बार्बेट और कॉपरस्मिथ भी कहा जाता है, यह क्रिमसन फोरहेड और गले के साथ एक एशियाई बार्बेट है, जो अपने मेट्रोनोमिक कॉल के लिये जाना जाता है जो हथौड़े से धातु पर प्रहार करने वाले कॉपरस्मिथ के समान लगता है।
    • यह भारतीय उपमहाद्वीप और दक्षिण पूर्व एशिया के कुछ हिस्सों में रहने वाला पक्षी है। 
  • ह्वाइट-थ्रोटेड रेडस्टार्ट:
    • यह मस्किकापिडे परिवार में पक्षी की एक प्रजाति है।
    • यह भूटान, चीन, भारत, म्याँमार और नेपाल में पाया जाता है।
  • ग्रे-चिन्ड मिनिवेट:
    • यह कैंपफैगिडे परिवार में पक्षी की एक प्रजाति है।
    • यह बांग्लादेश, भूटान, कंबोडिया, चीन, भारत, इंडोनेशिया, लाओस, मलेशिया, म्याँमार, नेपाल, ताइवान, थाईलैंड और वियतनाम में पाया जाता है।
    • इसका प्राकृतिक आवास उपोष्णकटिबंधीय या उष्णकटिबंधीय आर्द्र निम्न वन है।
  • अत: विकल्प (a) सही उत्तर है।

https://www.drishtiias.com/hindi/daily-updates/daily-news-analysis/rare-bird-species-in-chinnar-wildlife-sanctuary

75. भारतीय अनूप मृग (बारहसिंगा) की उस उपजाति, जो पक्की भूमि पर फलती-फूलती है और केवल घासभक्षी है, के संरक्षण के लिये निम्नलिखित में से कौन-सा संरक्षित क्षेत्र प्रसिद्ध है?

(a) कान्हा राष्ट्रीय उद्यान

(b) मानस राष्ट्रीय उद्यान

(c) मुदुमलाई वन्यजीव अभयारण्य

(d) ताल छप्पर वन्यजीव अभयारण्य

उत्तर:(a)

व्याख्या

  • मध्य प्रदेश का राजकीय पशु हार्ड ग्राउंड दलदली हिरण या बारासिंघा (रूसेर्वस डुवाउसेली) कान्हा राष्ट्रीय उद्यान और टाइगर रिज़र्व (KNPTR) में एक पुनरुद्धार देख रहा है।
  • कान्हा राष्ट्रीय उद्यान में दलदली हिरण विलुप्त होने के करीब था। हालाँकि संरक्षण के प्रयासों के साथ, जनसंख्या वर्तमान में लगभग 800 है।
  • हिरण, सतपुड़ा पहाड़ियों की मैकाल रेंज पर कान्हा राष्ट्रीय उद्यान और टाइगर रिज़र्व के लिये स्थानिक है। बंदी प्रजनन और आवास सुधार जैसे उपायों का उपयोग किया गया। अत: विकल्प (a) सही है।

76. इस्पात स्लैग निम्नलिखित में से किसके लिये सामग्री हो सकता है?

  1. आधार-सड़क के निर्माण के लिये
  2. कृषि मृदा के सुधार के लिये
  3. सीमेंट के उत्पादन के लिये

नीचे दिये गए कूट का प्रयोग कर सही उत्तर चुनिये-

(a) केवल 1 और 2

(b) केवल 2 और 3

(c) केवल 1 और 3

(d) 1, 2 और 3

उत्तर:(d)

व्याख्या

  • स्टील स्लैग स्टील बनाने की प्रक्रिया का उप-उत्पाद है। यह स्टील बनाने वाली भट्टियों में अशुद्धियों से पिघले हुए स्टील को अलग करने के दौरान उत्पन्न होता है। लावा एक पिघले हुए तरल के रूप में होता है तथा सिलिकेट्स और ऑक्साइड का एक जटिल समाधान होता है जो ठंडा होने पर जम जाता है।
  • स्टील स्लैग का उपयोग आधार पाठ्यक्रम सामग्री, ऐस्फाल्ट सड़क, ट्रैक या सतह की सतह परत के नीचे की सामग्री के रूप में किया जाता है। अतः कथन 1 सही है।
  • मृदा की अम्लता को ठीक करने की क्षमता के कारण स्टील स्लैग का उपयोग कृषि क्षेत्र में किया जा सकता है, क्योंकि इसमें पौधों के लिये कुछ पोषक तत्त्व होते हैं और सिलिकेट उर्वरक के रूप में भी होते हैं जो पौधों को सिलिकॉन प्रदान करने में सक्षम होते हैं। अतः कथन 2 सही है।
  • सीमेंट के उत्पादन के लिये स्टील-स्लैग का उपयोग किया जा सकता है। इसके अलावा स्लैग सीमेंट को व्यापक रूप से एक अलग सीमेंटिटियस घटक के रूप में या मिश्रित सीमेंट के हिस्से के रूप में कंक्रीट में उपयोग किया जाता है। यह क्षमता बढ़ाने, पारगम्यता कम करने, रासायनिक प्रतिक्रियाओं के प्रतिरोध में सुधार करने और रिबार संक्षारण को रोकने के लिये पोर्टलैंड सीमेंट के साथ सहक्रियात्मक रूप से कार्य करता है। अतः कथन 3 सही है।
  • अत: विकल्प (d) सही उत्तर है।

77. निम्नलिखित में से कौन-से ऐसे सर्वाधिक संभावनीय स्थान हैं जहाँ कस्तूरी मृग अपने प्राकृतिक आवास में मिल सकता है?

  1. अस्कोट वन्यजीव अभयारण्य
  2. गंगोत्री राष्ट्रीय उद्यान
  3. किशनपुर वन्यजीव अभयारण्य
  4. मानस राष्ट्रीय उद्यान

नीचे दिये गए कूट का प्रयोग कर सही उत्तर चुनिये-

(a) केवल 1 और 2

(b) केवल 2 और 3

(c) केवल 3 और 4

(d) केवल 1 और 4

उत्तर:(a)

व्याख्या

  • कस्तूरी मृग मुख्य रूप से दक्षिणी एशिया के पहाड़ों, विशेष रूप से हिमालय में वनों और अल्पाइन स्क्रब आवासों में निवास करते हैं।
  • अस्कोट वन्यजीव अभ्यारण्य:
    • यह भारत के उत्तराखंड राज्य में अस्कोट के पास पिथौरागढ़ से 54 किमी दूर स्थित है।
    • यह अभ्यारण्य मुख्य रूप से कस्तूरी मृग और उसके आवास के संरक्षण के उद्देश्य से स्थापित किया गया है। अत: 1 सही है।
  • गंगोत्री राष्ट्रीय उद्यान:
    • वर्ष 1989 में स्थापित, यह उद्यान उत्तराखंड में भागीरथी नदी के ऊपरी जलग्रहण क्षेत्र में स्थित है।
    • यहाँ नीली भेड़, हिमालयन तहर, कस्तूरी मृग, हिम तेंदुआ आदि जंतु निवास करते हैं। अत: 2 सही है।
  • किशनपुर वन्यजीव अभ्यारण्य:
    • यह उत्तर प्रदेश में मैलानी के पास दुधवा टाइगर रिज़र्व का एक हिस्सा है। इसकी स्थापना वर्ष 1972 में हुई थी।
    • इस अभ्यारण्य में बाघ, चीतल, हॉग हिरण, जंगली सूअर, ऊदबिलाव तथा कई अन्य जंतु निवास करते हैं।
    • बाज़, ड्रोंगो, उल्लू, बगुला तथा मोर जैसे बड़ी संख्या में निवासी और प्रवासी पक्षियों को खुले घास के मैदानों में देखा जा सकता है जो बारहमासी धाराओं से घिरा हुआ है। अतः 3 सही नहीं है।
  • मानस राष्ट्रीय उद्यान:
    • मानस राष्ट्रीय उद्यान असम में हिमालय की तलहटी में स्थित है। यह भूटान में रॉयल मानस नेशनल पार्क के निकट है।
    • यह अपने दुर्लभ और लुप्तप्राय स्थानिक वन्य जीवन के लिये जाना जाता है जैसे कि असम रूफ्ड टर्टल, हिस्पिड खरगोश, सुनहरा लंगूर और पिग्मी हॉग।
    • मानस जंगली जल भैंसों की आबादी के लिये प्रसिद्ध है। अतः 4 सही नहीं है।
  • अत: विकल्प (a) सही उत्तर है।

https://www.drishtiias.com/hindi/daily-updates/daily-news-analysis/uttrakhand-approves-land-transfer

78. ग्रामीण सड़क निर्माण में, पर्यावरणीय दीर्घोपयोगिता को सुनिश्चित करने अथवा कार्बन पदचिह्न को घटाने के लिये निम्नलिखित में से किसके प्रयोग को अधिक प्राथमिकता दी जाती है?

  1. ताम्र स्लैग
  2. शीत मिश्रित ऐस्फाल्ट प्रौद्योगिकी
  3. जियोटेक्सटाइल्स
  4. उष्ण मिश्रित ऐस्फाल्ट प्रौद्योगिकी
  5. पोर्टलैंड सीमेंट

नीचे दिये गए कूट का प्रयोग कर सही उत्तर चुनिये-

(a) केवल 1, 2 और 3

(b) केवल 2, 3 और 4

(c) केवल 4 और 5

(d) केवल 1 और 5

उत्तर:(a)

व्याख्या

  • ताम्र स्लैग ताम्र के प्रगलन और शोधन के दौरान प्राप्त एक उप-उत्पाद है। बेकार ताम्र स्लैग का उपयोग अपघर्षक उपकरण, सड़क निर्माण और गिट्टी के रूप में किया जा सकता है। सीमेंट तथा कंक्रीट में कॉपर स्लैग का उपयोग संभावित पर्यावरण के साथ-साथ सभी संबंधित उद्योगों के लिये आर्थिक लाभ प्रदान करता है, विशेष रूप से उन क्षेत्रों में जहाँ कॉपर स्लैग का काफी मात्रा में उत्पादन होता है। अत: 1 सही है।
  • शीत मिश्रित ऐस्फाल्ट का उत्पादन बिना गर्म किये खनिज समुच्चय को इमल्सीफाइड बिटुमेन या फोमेड बिटुमेन के साथ मिलाकर किया जाता है। आधार और सतह पाठ्यक्रमों में उपयोग किये जाने पर शीत ऐस्फाल्ट मिश्रण प्राय: हल्के से मध्यम यातायात वाली सड़कों के लिये उपयुक्त होते हैं। अत: 2 सही है।
  •  जियोटेक्सटाइल एक सिंथेटिक पारगम्य कपड़ा सामग्री है जिसका उपयोग मृदा की विशेषताओं में सुधार करने के लिये किया जाता है और इसमें मृदा के साथ उपयोग किये जाने पर अलग करने, फिल्टर करने, सुदृढ़ करने, सुरक्षा करने और निकालने की क्षमता होती है। यह ज्यादातर सड़क निर्माण में छानने और जुदाई के लिये प्रयोग किया जाता है। अत: 3 सही है।
  • उष्ण मिश्रित ऐस्फाल्ट (HMA) लगभग 95% पत्थर, रेत, या बज़री का एक संयोजन है जो कच्चे तेल के उत्पाद ऐस्फाल्ट सीमेंट द्वारा एक साथ बंधा होता है। उष्ण मिश्रित प्रौद्योगिकी के व्यापक उपयोग से पर्यावरण प्रदूषण होता है क्योंकि ये पौधे भारी मात्रा में ग्रीनहाउस गैसों का उत्सर्जन करते हैं। अतः 4 सही नहीं है।
  • पोर्टलैंड सीमेंट एक महीन पिसे हुए पाउडर के रूप में एक बाध्यकारी सामग्री है जिसे चूना पत्थर और मृदा के मिश्रण को जलाकर और पीसकर बनाया जाता है। इसका उत्पादन ग्रीनहाउस गैसों का उत्पादन करता है। अतः 5 सही नहीं है।
  • अत: विकल्प (a) सही है।

https://www.drishtiias.com/hindi/daily-updates/daily-news-analysis/coir-geo-textile

79. निम्नलिखित कथनाें पर विचार कीजिये-

  1. कोयले की राख में आर्सेनिक, सीसा और पारद अंतर्विष्ट होते हैं।
  2. कोयला संचालित विद्युत् संयंत्र पर्यावरण में सल्फर डाइऑक्साइड और नाइट्रोजन के ऑक्साइड उत्सर्जित करते हैं।
  3. भारतीय कोयले में राख की अधिक मात्रा पाई जाती है।

उपर्युक्त कथनाें में से कौन-सा/से सही है/हैं?

(a) केवल 1 

(b) केवल 2 और 3

(c) केवल 3 

(d) 1, 2 और 3

उत्तर: (d)

व्याख्या

  • कोयले की राख, जिसे कोयला दहन अवशिष्ट या CCR भी कहा जाता है, मुख्य रूप से कोयले से चलने वाले बिजली संयंत्रों में कोयले के जलने से उत्पन्न होती है। इसमें पारा, कैडमियम और आर्सेनिक जैसे प्रदूषक होते हैं। उचित प्रबंधन के बिना, ये संदूषक जलमार्ग, भूजल, पेयजल और वायु को प्रदूषित कर सकते हैं। अतः कथन 1 सही है।
  • कोयला आधारित थर्मल पावर प्लांट सल्फर डाइऑक्साइड और नाइट्रोजन ऑक्साइड उत्सर्जन के प्रमुख स्रोत हैं। अतः कथन 2 सही है।
  • कोल इंडिया लिमिटेड के अनुसार, देश में उत्पादित कोयले की राख सामग्री सामान्यता 25 से 45% होती है जबकि आयातित कोयले की औसत राख सामग्री 10 से 20% तक होती है। अतः कथन 3 सही है।
  • अत: विकल्प (d) सही उत्तर है।

https://www.drishtiias.com/hindi/to-the-points/paper3/india-s-largest-emitter-of-sulfur-dioxide-so2

80. खेती में बायोचार का क्या उपयोग है?

  1. बायोचार ऊर्ध्वाधर खेती (Vertical Farming) में वृद्धिकर माध्यम के अंश के रूप में प्रयुक्त किया जा सकता है।
  2. जब बायोचार वृद्धिकर माध्यम के अंश के रूप में प्रयुक्त किया जाता है, तो वह नाइट्रोजन-यौगिकीकारी सूक्ष्मजीवाें की वृद्धि को बढ़ावा देता है।
  3. जब बायोचार वृद्धिकर माध्यम के अंश के रूप में प्रयुक्त किया जाता है, तब वह उस वृद्धिकर माध्यम की जलधारण क्षमता को अधिक लंबे समय तक बनाए रखने में सहायक होता है।

उपर्युक्त कथनाें में से कौन-सा/से सही है/हैं?

(a) केवल 1 और 2 

(b) केवल 2

(c) केवल 1 और 3 

(d) 1, 2 और 3

उत्तर:(d)

व्याख्या

  • बायोचार एक छिद्रयुक्त कार्बोनेसियस ठोस है जो विभिन्न बायोमास फीडस्टॉक्स को ऑक्सीजन-सीमित वातावरण में उच्च तापमान के तहत गर्म करके उत्पादित किया जाता है।
  • चूँकि बायोचार मृदा प्रोफाइल के माध्यम से लंबवत रूप से पलायन करता है, इसलिये ऊर्ध्वाधर खेती में बढ़ते माध्यम के हिस्से के रूप में इसका उपयोग किया जा सकता है। अतः कथन 1 सही है।
  • इसकी सोखने की क्षमता के कारण कुछ बायोचार में भारी धातुओं, कीटनाशकों, शाकनाशियों और हार्मोन को स्थिर करने, जलमार्ग में नाइट्रेट लीचिंग और मल बैक्टीरिया को रोकने तथा मिट्टी से N2O और CH4 उत्सर्जन कम करने की क्षमता होती है। अतः कथन 2 सही है।
  • बायोचार पौधों में वृद्धि के लिये मृदा में जल और पोषक तत्त्वों को बनाए रखने में मदद कर सकता है। अतः कथन 3 सही है।

अत: विकल्प (d) सही है।

81. यदि किसी पौधे की विशिष्ट जाति को वन्यजीव सुरक्षा अधिनियम, 1972 की अनुसूची VI में रखा गया है, तो इसका क्या तात्पर्य है?

(a) उस पौधे की खेती करने के लिये लाइसेंस की आवश्यकता है।

(b) ऐसे पौधे की खेती किसी भी परिस्थिति में नहीं हो सकती।

(c) यह एक आनुवंशिकत: रूपांतरित फसली पौधा है।

 (d) ऐसा पौधा आक्रामक होता है और पारितंत्र के लिये हानिकारक होता है।

उत्तर : (a) 

व्याख्या

  • वन्य जीवन संरक्षण अधिनियम, 1972 पौधों और जीवों की प्रजातियों के संरक्षण के लिये अधिनियमित किया गया है। अधिनियम वन्य जीवों, पक्षियों तथा पौधों की सुरक्षा प्रदान करता है। इसकी छह अनुसूचियाँ हैं जो अलग-अलग स्तर की सुरक्षा प्रदान करती हैं।
    • अनुसूची I और अनुसूची II का भाग II पूर्ण सुरक्षा प्रदान करते हैं - इनके तहत अपराधों के लिये उच्चतम दंड निर्धारित किये गए हैं।
    • अनुसूची III और अनुसूची IV में सूचीबद्ध प्रजातियाँ भी संरक्षित हैं लेकिन दंड बहुत कम हैं।
    • अनुसूची V में वे जीव शामिल हैं जिनका शिकार किया जा सकता है।
    • अनुसूची VI में निर्दिष्ट स्थानिक पौधों को खेती और रोपण से प्रतिबंधित किया गया है।
    • अनुसूची VI में, निम्नलिखित पौधों को शामिल किया गया है:
      • बेडडोम्स साइकैड (साइकस बेडडोमेई),
      • ब्लू वांडा (वांडा सोरुलेक),
      • कुथ (सौसुरिया लप्पा),
      • लेडीज़ स्लिपर ऑर्किड (पैपिओपेडिलम एसपीपी)
      • पिचर प्लांट (नेपेंथेस खासियाना),
      • रेड वांडा (रणंथेरा इमस्चुटियाना)
  • हालाँकि आगे यह भी कहा गया है कि बिना लाइसेंस के निर्दिष्ट पौधों की खेती प्रतिबंधित है। अधिनियम की धारा 17C के अनुसार, मुख्य वन्य जीव संरक्षक या राज्य सरकार द्वारा इस निमित्त प्राधिकृत किसी अन्य अधिकारी द्वारा दिये गए लाइसेंस के तहत और उसके अनुसार कोई भी व्यक्ति किसी निर्दिष्ट पौधे की खेती नहीं करेगा। अत: विकल्प (a) सही है।

https://www.drishtiias.com/hindi/daily-updates/daily-news-analysis/not-all-animals-migrate-by-choice-campaign

https://www.drishtiias.com/hindi/loksabha-rajyasabha-discussions/salman-khan-conviction

82. प्राचीन भारतीय गुप्त राजवंश के समय में संदर्भ में, नगर घंटाशाला, कदूरा तथा चौल किस लिये विख्यात थे?

(a) विदेशी व्यापार करने वाले बंदरगाह

(b) शक्तिशाली राज्याें की राजधानियाँ

(c) उत्कृष्ट प्रस्तर कला तथा स्थापत्य से संबंधित स्थान 

(d) बौद्ध धर्म के महत्त्वपूर्ण तीर्थस्थल

उत्तर : (a)

व्याख्या

  • प्राचीन काल में भारत का मिस्र, रोम, यूनानियों, अरबों, चीन और लगभग सभी दक्षिण पूर्व एशियाई देशों के साथ व्यापारिक और सांस्कृतिक संपर्क थे।
  • भारत के दोनों तटों को बंदरगाहों की एक कड़ी से सजाया गया है।
  • पश्चिमी तट पर स्थित बंदरगाहों में बैरगया, सुपारा, कैलीना, सेमिला, मंडागोर, पलेपटमे, मालिज़िगारा, और्रानोब्बास, नौरा, टिंडिस, मुज़िरिस और नेलसिंडा थे।
  • पूर्वी तट पर बंदरगाह ताम्रलिप्ति, चरित्रपुर, पलुरु, दांतापुर, कलिंगपट्टनम, पिथुंडा, सोपत्मा, घंटासला, कडूरा, पोडुका, पुहार, कोरकाई और कैमारा थे।
  • पश्चिमी तट पर मालवान, सोपारा, एलीफेंटा द्वीप, चौल, उदयवारा, होन्नावर, गोपाकपटना और भारत के पूर्वी तट पर पूम्पुहार और ट्रांक्यूबार के आसपास के क्षेत्र माणिकपटना, कलिंगपटना में तटवर्ती अन्वेषण किये गए हैं। अत: विकल्प (a) सही उत्तर है।

83. कृषि में शून्य-जुताई (Zero-Tillage) का/के क्या लाभ है/हैं?

  1. पिछली फसल के अवशेषाें को जलाए बिना गेहूँ की बुआई संभव है।
  2. चावल की नई पौध की नर्सरी बनाए बिना, धान के बीजाें का नम मृदा में सीधे रोपण संभव है।
  3. मृदा में कार्बन पृथक्करण संभव है।

नीचे दिये गए कूट का प्रयोग कर सही उत्तर चुनिये-

(a) केवल 1 और 2

(b) केवल 2 और 3

(c) केवल 3 

(d) 1, 2 और 3

उत्तर : (d)

व्याख्या

  • शून्य जुताई (ज़ीरो टिलेज) वह प्रक्रिया है जहाँ बीज को बिना पूर्व तैयारी और बिना मिट्टी तैयार किये तथा जहांँ पिछली फसल के बुलबुले (स्टबल) मौजूद होते हैं, वहाँ ड्रिलर्स के माध्यम से बोया जाता है। एक अध्ययन के अनुसार, यदि किसान अपने फसल अवशेषों को जलाना बंद कर दें तथा इसके बजाय शून्य जुताई खेती की अवधारणा को अपनाएंँ तो उत्तर भारत में किसान न केवल वायु प्रदूषण को कम करने में मदद कर सकते हैं, बल्कि अपनी मृदा की उत्पादकता में भी सुधार कर सकते हैं और अधिक लाभ कमा सकते हैं। ज़ीरो टिलेज के तहत बिना जुताई वाली मिट्टी में गेहूंँ की सीधी बुवाई और चावल के अवशेषों को छोड़ देना बहुत फायदेमंद साबित हुआ है। इसने जल, श्रम व कृषि रसायनों के उपयोग में कमी, ग्रीनहाउस गैस उत्सर्जन में कमी और मृदा के स्वास्थ्य एवं फसल की उपज में सुधार किया, इस तरह किसानों तथा समाज दोनों को बड़े पैमाने पर लाभ हुआ। अत: कथन 1 सही है।
  • चावल का प्रत्यक्ष बीजारोपण (DSR) जिसे 'बीज बिखेरना तकनीक (Broadcasting Seed Technique)' के रूप में भी जाना जाता है, धान की बुवाई की एक जल बचत विधि है। इस विधि में बीजों को सीधे खेतों में ड्रिल किया जाता है। नर्सरी से जलभराव वाले खेतों में धान की रोपाई की पारंपरिक जल-गहन विधि के विपरीत यह विधि भूजल की बचत करती है। इस पद्धति में कोई नर्सरी तैयारी या प्रत्यारोपण शामिल नहीं है। 
  • किसानों को केवल अपनी ज़मीन को समतल करना होता है और बुवाई से पहले सिंचाई करनी होती है। यह पाया गया है कि 1 किलो धान के उत्पादन के लिये 5000 लीटर तक पानी का उपयोग किया जाता है। हालांँकि पानी की बढ़ती कमी की स्थिति में न्यूनतम या शून्य जुताई के साथ DSR श्रम की बचत कर इस तकनीक के लाभों को और बढ़ाया जा सकता है। अत: कथन 2 सही है।
  • बिना जुताई वाली मृदा, जुताई वाली मृदा से आंशिक रूप में ठंडी होती है क्योंकि पौधे के अवशेषों की एक परत सतह पर मौज़ूद होती है। मिट्टी में कार्बन जमा हो जाता है तथा इसकी गुणवत्ता में वृद्धि होती है, जिससे ग्लोबल वार्मिंग का खतरा कम होता है। अत: कथन 3 सही है। 
  • अतः विकल्प (d) सही है।

https://www.drishtiias.com/hindi/current-affairs-news-analysis-editorials/news-editorials/01-03-2019

https://www.drishtiias.com/hindi/daily-updates/daily-news-editorials/sustainable-agriculture

84. भारत की जैव-ईंधन की राष्ट्रीय नीति के अनुसार, जैव-ईंधन के उत्पादन के लिये निम्नलिखित में से किनका उपयोग कच्चे माल के रूप में हो सकता है?

  1. कसावा
  2. क्षतिग्रस्त गेहूँ के दाने
  3. मूँगफली के बीज
  4. कुलथी (Horse Gram) 
  5. सड़ा आलू
  6. चुकंदर

नीचे दिये गए कूट का प्रयोग कर सही उत्तर चुनिये-

(a) केवल 1, 2, 5 और 6

(b) केवल 1, 3, 4 और 6

(c) केवल 2, 3, 4 और 5 

(d) 1, 2, 3, 4, 5 और 6

उत्तर : (a)

व्याख्या

  • जैव ईंधन पर राष्ट्रीय नीति, 2018 क्षतिग्रस्त खाद्यान्न जो मानव उपभोग के लिये अनुपयुक्त हैं जैसे- गेहूंँ, टूटे चावल आदि से इथेनॉल प्राप्त किया जाता है। 
  • यह नीति राष्ट्रीय जैव ईंधन समन्वय समिति के अनुमोदन के आधार पर खाद्यान्न की अधिशेष मात्रा को इथेनॉल में परिवर्तित करने की भी अनुमति प्रदान करती है।
  • यह नीति इथेनॉल उत्पादन में प्रयोग होने वाले तथा मानव उपभोग के लिये अनुपयुक्त पदार्थ जैसे- गन्ने का रस, चीनी युक्त सामग्री- चुकंदर, मीठा चारा, स्टार्च युक्त सामग्री तथा मकई, कसावा, गेहूंँ, टूटे चावल, सड़े हुए आलू के उपयोग की अनुमति प्रदान कर इथेनॉल उत्पादन हेतु कच्चे माल के दायरे का विस्तार करती है।
  • अतः विकल्प (a) सही उत्तर है।

https://www.drishtiias.com/hindi/daily-updates/daily-news-analysis/rajasthan-first-state-to-implement-biofuel-policy

85. निम्नलिखित में से कौन-सा कथन ‘कार्बन के सामाजिक मूल्य’ पद का सर्वोत्तम रूप से वर्णन करता है? आर्थिक मूल्य के रूप में यह निम्नलिखित में से किसका माप है?

(a) प्रदत्त वर्ष में एक टन CO2 के उत्सर्जन से होने वाली दीर्घकालीन क्षति     

(b) किसी देश की जीवाश्म ईंधनाें की आवश्यकता, जिन्हें जलाकर देश अपने नागरिकाें को वस्तुएँ और सेवाएँ प्रदान करता है

(c) किसी जलवायु शरणार्थी (Climate Refugee) द्वारा किसी नए स्थान के प्रति अनुकूलित होने हेतु किये गए प्रयास 

(d) पृथ्वी ग्रह पर किसी व्यक्ति विशेष द्वारा अंशदत कार्बन पदचिह्न

उत्तर : (a)

व्याख्या

  • कार्बन का सामाजिक मूल्य (SCC) आर्थिक नुकसान का एक अनुमान (डॉलर में) है, जो वातावरण में एक अतिरिक्त टन ग्रीनहाउस गैसों के उत्सर्जन के परिणामस्वरूप होता है।
  • SCC नीति निर्माताओं के लिये जलवायु परिवर्तन के प्रभाव को आर्थिक रूप से प्रभावित करता है और अन्य निर्णय निर्माताओं को उन निर्णयों के आर्थिक प्रभावों को समझने में मदद मिलती है, जो उत्सर्जन में वृद्धि या कमी करेंगे।
  •  कार्बन उत्सर्जन के लिये भारत का देश-स्तरीय सामाजिक मूल्य का उच्चतम अनुमान 86 डॉलर प्रति CO2 टन लगाया गया था। इसका तात्पर्य यह है कि भारतीय अर्थव्यवस्था को CO2 के प्रत्येक अतिरिक्त टन के उत्सर्जन से $86 का नुकसान होगा। भारत के बाद अमेरिका (48 डॉलर) और सऊदी अरब (47 डॉलर) का स्थान आता है।

अतः विकल्प (a) सही उत्तर है।

86. भारत में दालाें के उत्पादन के संदर्भ में, निम्नलिखित कथनाें पर विचार कीजिये-

  1. उड़द की खेती खरीफ और रबी दोनाें फसलाें में की जा सकती है।
  2. कुल दाल उत्पादन का लगभग आधा भाग केवल मूँग का होता है।
  3. पिछले तीन दशकाें में, जहाँ खरीफ दालों का उत्पादन बढ़ा है, वहीं रबी दालाें का उत्पादन घटा है।

उपर्युक्त कथनाें में से कौन-सा/से सही है/हैं?

(a) केवल 1

(b) केवल 2 और 3 

(c) केवल 2

(d) 1, 2 और 3

उत्तर : (a)

व्याख्या

  • भारत में सर्दियों (रबी के मौसम ) में उगाई जाने वाली महत्त्वपूर्ण दलहनी फसलें चना, मसूर, खेसारी दाल, मटर और राजमा हैं। हालाँकि, हरे चने, काले चने और लोबिया को रबी और खरीब दोनों मौसमों में उगाया जाता है।
  • काले चने की फसल 600 से 1000 मिमी तक की वार्षिक वर्षा तथा अधिक तापमान वाले क्षेत्रों में उगाई जाती है। यह मुख्य रूप से इसकी खेती अनाज-दाल फसल प्रणाली में मिट्टी के पोषक तत्वों को संरक्षित करने और विशेष रूप से चावल की खेती के बाद बची हुई मिट्टी की नमी का उपयोग करने के लिये की जाती है। हालाँकि इसे सभी मौसमों में उगाया जा सकता है, काले चने की अधिकांश खेती विशेष रूप से प्रायद्वीपीय भारत में या तो रबी या रबी के मौसम के उपरांत की जाती है। अतः कथन 1 सही है।
  • अर्थशास्त्र और सांख्यिकी निदेशालय (डीईएस) के अनुसार, वर्ष 2018-19 में दलहनी फसलों के उत्पादन का हिस्सा तुअर (15.34%), चना (43.29%), मूंग (हरा चना, 10.04%), उड़द (काला चना, 13.93%), मसूर (6.67%), और अन्य दालें (10%) है। अतः कथन 2 सही नहीं है।
  • पिछले तीन दशकों में खरीफ तथा रबी दोनों मौसमों के दौरान दालों के उत्पादन में वृद्धि हुई है। अतः कथन 3 सही नहीं है।

अतः, विकल्प (a) सही उत्तर है।

87. ‘‘यह फसल उपोष्ण प्रकृति की है। उसके लिये कठोर पाला हानिकारक है। इसके विकास के लिये कम-से-कम 210 पाला-रहित दिवसाें और 50-100 सेंटीमीटर वर्षा की आवश्यकता पड़ती है। हल्की सुअपवाहित मृदा जिसमें नमी धारण करने की क्षमता है इसकी खेती के लिये आदर्श रूप से अनुकूल है।’’ यह फसल निम्नलिखित में से कौन-सी है?

(a) कपास

(b) जूट

(c) गन्ना

(d) चाय

उत्तर : (a)

व्याख्या

  • कपास:
    • तापमान: 21-30 डिग्री सेल्सियस के मध्य
    • वर्षा: लगभग 50-100 सेमी.
    • मिट्टी का प्रकार: दक्कन पठार की बेहतर जल निकासी वाली काली कपास मिट्टी।
  • जूट:
    • तापमान: 15-34 डिग्री सेल्सियस के मध्य
    • वर्षा: लगभग 100-250 सेमी.
    • मिट्टी का प्रकार: जूट को चिकनी मिट्टी से लेकर रेतीली दोमट मिट्टी तक सभी प्रकार की मिट्टी पर उगाया जा सकता है, लेकिन दोमट जलोढ़ सबसे उपयुक्त होती हैं।
  • गन्ना:
    • तापमान: 28-32 डिग्री सेल्सियस के मध्य 
    • वर्षा: लगभग 75-120 सेमी.
    • मिट्टी का प्रकार: गन्ना काली कपास मिट्टी, दोमट, भूरी या लाल दोमट,  गहरी समृद्ध दोमट मिट्टी यहाँ तक कि लेटराइट सहित विभिन्न प्रकार की मिट्टी में उगाया जा सकता है।
  • चाय:
    • तापमान: 20-30 डिग्री सेल्सियस के मध्य
    • वर्षा: लगभग 150-300 सेमी.
    • मिट्टी का प्रकार: गहरी और उपजाऊ अच्छी जल निकासी वाली मिट्टी, ह्यूमस और कार्बनिक पदार्थों से भरपूर।
  • अतः  विकल्प (a) सही उत्तर है।

https://www.drishtiias.com/hindi/daily-updates/daily-news-editorials/the-flawed-spin-to-india-cotton-story

88. सौर जल पंपों के संदर्भ में, निम्नलिखित कथनाें पर विचार कीजिये-

  1. सौर ऊर्जा का प्रयोग पृष्ठीय पंपाें को चलाने के लिये हो सकता है और निमज्जनी (Submersible) पंपों के लिये नहीं।
  2. सौर ऊर्जा का प्रयोग अपेकेंद्री पंपाें को चलाने के लिये हो सकता है और पिस्टन वालों के लिये नहीं।

उपर्युक्त कथनाें में से कौन-सा/से सही है/हैं?

(a) केवल 1

(b) केवल 2

(c) 1 और 2 दोनों

(d) न तो 1, न ही 2

उत्तर : (d)

व्याख्या

  • सोलर पम्पिंग सिस्टम के मुख्य घटकों में एक फोटोवोल्टिक (पीवी) ऐरे, इलेक्ट्रिक मोटर और पंप शामिल हैं।
  • कार्यात्मक तंत्र के आधार पर सौर ऊर्जा संचालित पंप विभिन्न प्रकार के होते हैं। लेकिन मुख्य रूप से सौर जल पंप चार प्रकार के होते हैं - निमज्जनी (Submersible) पंप, सतह पंप, प्रत्यक्ष धारा (DC) पंप और वैकल्पिक धारा (AC) पंप। अतः कथन 1 सही नहीं है।
  • सौर ऊर्जा का उपयोग अपेकेंद्री और पिस्टन पंप दोनों को चलाने के लिये किया जा सकता है। अतः कथन 2 सही नहीं है।

अतः  विकल्प (d) सही उत्तर है।

https://www.drishtiias.com/hindi/daily-updates/daily-news-editorials/towards-solar-powered-agriculture 

89. भारत में गन्ने की खेती में वर्तमान प्रवृत्तियों के संदर्भ में, निम्नलिखित कथनों पर विचार कीजिये-

  1. जब ‘बड चिप सैटलिंग्स (Bud Chip Settlings)’ को नर्सरी में उगाकर मुख्य कृषि भूमि में प्रतिरोपित किया जाता है, तब बीज सामग्री में बड़ी बचत होती है।
  2. जब सेट्स का सीधे रोपण किया जाता है, तब एक-कलिका (Single-Budded) सेट्स का अंकुरण प्रतिशत कई-कलिका (Many Budded) सेट्स की तुलना में बेहतर होता है।
  3. खराब मौसम की दशा में यदि सेट्स का सीधे रोपण होता है, तब एक-कलिका सेट्स का जीवित बचना बड़े सेट्स की तुलना में बेहतर होता है।
  4. गन्ने की, खेती, ऊतक संवर्द्धन से तैयार की गई सैटलिंग से की जा सकती है।

उपर्युक्त कथनों में से कौन-सा/से सही है/हैं?

(a) केवल 1 और 2

(b) केवल 3

(c) केवल 1 और 4

(d) केवल 2, 3 और 4

उत्तर : (c)

व्याख्या

ऊतक संवर्धन प्रौद्योगिकी:

  • ऊतक संवर्धन एक ऐसी तकनीक है जिसमें पौधों के टुकड़े एक प्रयोगशाला में सुसंस्कृत और उगाए जाते हैं
  • यह मौजूदा वाणिज्यिक किस्मों के रोग मुक्त बीज गन्ने के तेज़ी से उत्पादन और आपूर्ति का एक नया तरीका प्रदान करता है।
  • यह मदर प्लांट को क्लोन करने के लिये मेरिस्टेम का उपयोग करता है।
  • यह आनुवंशिक पहचान को भी संरक्षित करता है।
  • ऊतक संवर्धन प्रौद्योगिकी, अपने जटिल संगठन और भौतिक परिसीमा के कारण अनार्थक होती जा रही है।

बड चिप प्रौद्योगिकी:

  • ऊतक संवर्धन के व्यवहार्य विकल्प के रूप में, यह द्रव्यमान को कम करता है और बीजों के त्वरित गुणन को सक्षम बनाता है।
  • यह विधि दो से तीन बड सेट लगाने की पारंपरिक विधि की तुलना में अधिक किफायती और सुविधाजनक साबित हुई है।
  • रोपण के लिये उपयोग की जाने वाली बीज सामग्री पर पर्याप्त बचत के साथ रिटर्न अपेक्षाकृत बेहतर है। अतः कथन 1 सही है।
  • शोधकर्त्ताओं ने पाया है कि दो बड्स वाले सेट बेहतर उपज के साथ लगभग 65 से 70% अंकुरण दे रहे हैं। अतः कथन 2 सही नहीं है।
  • खराब मौसम में बड़े सेट बेहतर जीवित रहते हैं लेकिन रासायनिक उपचार से संरक्षित होने पर सिंगल बडेड सेट भी 70% अंकुरण देते हैं। अतः कथन 3 सही नहीं है।
  • अत: विकल्प (c) सही उत्तर है।

90. भारत के संदर्भ में, निम्नलिखित में से किस/किन पद्धति/पद्धतियों को पारितंत्र-अनुकूली कृषि माना जाता है?

  1. फसल विविधरूपण
  2. शिंव आधिक्य (Legume intensification)
  3. टेंसियोमीटर का प्रयोग
  4. ऊर्ध्वाधर कृषि (Vertical Farming)

नीचे दिये गए कूट का प्रयोग कर सही उत्तर चुनिये:

(a) केवल 1, 2 और 3

(b) केवल 3

(c) केवल 4 

(d) 1, 2, 3 और 4

उत्तर : (d)

व्याख्या

  • फसल विविधीकरण:फसल विविधीकरण से तात्पर्य नई फसलों या फसल प्रणालियों से कृषि उत्पादन को जोड़ने से है, जिसमें एक विशेष कृषि क्षेत्र  पर कृषि उत्पादन के पूरक विपणन अवसरों के साथ मूल्यवर्द्धित फसलों से विभिन्न तरीकों से लाभ मिल रहा है। किस्मों की एक बड़ी शृंखला पेश करने से कृषि उत्पादन में विविधता भी आती है जिससे प्राकृतिक जैव विविधता में वृद्धि हो सकती है। इसके अलावा कृषि का विविधीकरण भूमि और जल के संरक्षण का एक वैकल्पिक तरीका है। अतः कथन 1 सही है।
  • शिंव आधिक्य:
  • एक फलीदार पौधा (लेग्यूम), पौधों का एक समूह है जिसमें सब्जियाँ होती हैं या जमीन की गांठों से उगाए जाने वाले खाद्य पदार्थ होते हैं जो नाइट्रोजन युक्त सामग्री की क्षमता को बढ़ाते हैं। उदाहरणों में बबूल, मटर, तिपतिया घास, आदि शामिल हैं।
  • लेग्यूम से मृदा स्वास्थ्य में सुधार होता है। जिसमें मृदा में नाइट्रोजन तथा कार्बनिक पदार्थों को समृद्ध करना शामिल है। लेग्यूम नाइट्रोजन उर्वरक की आवश्यकता को कम करके फसल प्रणालियों के ऊर्जा पदचिह्न को कम कर सकती हैं तथा कृषि-पारिस्थितिक तंत्र की स्थिरता एवं स्वास्थ्य में सुधार कर सकती हैं। अतः कथन 2 सही है।
  • टेन्सियोमीटर: इसका शाब्दिक अर्थ है नमी मापन। मृदा से जल सोखने के लिये पौधे को मृदा में पर्याप्त नमी की आवश्यकता होती है। इस तनाव को टेन्सियोमीटर द्वारा मापा जाता है इस प्रकार मृदा की आर्द्रता का संकेत उस गहराई पर दिया जाता है जिसमें इसे रखा गया था। टेन्सियोमीटर किसानों और अन्य सिंचाई प्रबंधकों को यह निर्धारित करने में मदद करता है कि मृदा को कब जल देना है। अतः कथन 3 सही है।
  • ऊर्ध्वाधर कृषि: यह वर्टिकल स्टैक्ड लेयर्स में फसलों को उगाने की प्रथा है और इसमें प्राय: नियंत्रित-पर्यावरण कृषि शामिल होती है जिसका उद्देश्य पौधों की वृद्धि तथा मृदा रहित खेती तकनीकों को अनुकूलित करना है। अतः कथन 4 सही है।

अत: विकल्प (d) सही उत्तर है।

91. कृषि में फर्टिगेशन (Fertigation) के क्या लाभ हैं?

  1. सिंचाई जल की क्षारीयता का नियंत्रण संभव है।
  2. रॉक फॉस्फेट और सभी अन्य फॉस्फेटिक उर्वरकों का सफलता के साथ अनुप्रयोग संभव है।
  3. पौधों के लिये पोषक बढ़ी हुई मात्रा में सुलभ किये जा सकते हैं।
  4. रासायनिक पोषकों के निक्षालन में कमी संभव है।

नीचे दिये गए कूट का प्रयोग कर सही उत्तर चुनिये:

(a) केवल 1, 2 और 3

(b) केवल 1, 2 और 4

(c) केवल 1, 3 और 4 

(d) केवल 2, 3 और 4

उत्तर : (c) 

व्याख्या

  • सिंचाई जल के माध्यम से खेतों में फसल उर्वरकों की आपूर्ति करने की प्रथा को फर्टिगेशन कहा जाता है। यह एक आधुनिक कृषि-तकनीक है जो उर्वरकों की दक्षता बढ़ाकर उसके उपयोग को कम करके निवेशित उर्वरक की प्रतिपूर्ति बढ़ाता है, जो उच्च उपज और कम पर्यावरण प्रदूषण की सुविधा प्रदान करती है।
  • पारंपरिक या ड्रॉप-फर्टिगेशन विधियों की तुलना में फर्टिगेशन विधि द्वारा होने वाले लाभों में निम्नलिखित बिंदु शामिल हैं:
    • पौधों द्वारा पोषक तत्त्वों के अवशोषण में वृद्धि।
    • 'माइक्रोडोज़' करने की क्षमता, पौधों का पर्याप्त मात्रा में भक्षण जिससे पोषक तत्त्वों को अवशोषित किया जा सके ताकि बारिश होने पर उन्हें वर्षा जल में धुलने के लिये न छोड़ा जाए।
    • उर्वरक, रसायन और जल की कमी।
    • जलापूर्ति में रसायनों की क्षारीयता कम करना।
    • जल की खपत में कमी जिससे पौधे की जड़ों में जल धारण करने की क्षमता बढ़ जाती है।
    • पोषक तत्त्वों के अनुप्रयोग को सटीक समय और आवश्यक दर पर नियंत्रित किया जा सकता है।
    • दूषित मृदा के माध्यम से उससे उत्पन्न होने वाली बीमारियों का न्यूनतम जोखिम।
    • मृदा क्षरण के मुद्दों को कम करना क्योंकि पोषक तत्त्वों को जल की ड्रिप प्रणाली के माध्यम से पंप किया जाता है। फर्टिगेशन को नियोजित करने के लिये उपयोग की जाने वाली विधियों के माध्यम से अक्सर क्षारीयता कम हो जाती है।
    • फर्टिगेशन के माध्यम से सिंचाई जल के पीएच को नियंत्रित किया जा सकता है।
    • कृषि में फर्टिगेशन विधि की सलाह नहीं दी जाती है क्योंकि उपयोग किया जाने वाला पदार्थ अवक्षेप बना सकता है और फॉस्फेट उच्च कैल्शियम और मैग्नीशियम सामग्री वाले पानी में अवक्षेपित हो सकता है। अतः कथन 2 सही नहीं है।
  • अतः विकल्प (c) सही है।

https://www.drishtiias.com/hindi/daily-updates/daily-news-editorials/micro-irrigation-in-india 

https://www.drishtiias.com/hindi/current-affairs-news-analysis-editorials/news-editorials/16-01-2019#

92. निम्नलिखित खनिजों पर विचार कीजिये-

  1. बेंटोनाइट
  2. क्रोमाइट
  3. कायनाइट
  4. सिलीमेनाइट

भारत में, उपर्युक्त में से कौन-सा/से आधिकारिक रूप से नामित प्रमुख खनिज (Major Minerals) है/हैं?

(a) केवल 1 और 2

(b) केवल 4

(c) केवल 1 और 3

(d) केवल 2, 3 और 4

उत्तर : (d) 

व्याख्या

  • राष्ट्रीय लेखा सांख्यिकी (2007) के अनुसार, खनिजों को सामान्य रूप में प्रमुख एवं गौण खनिजों में वर्गीकृत किया गया है।
  • प्रमुख खनिजों में निम्नलिखित शामिल हैं:
    • ईंधन खनिज: कोयला, लिग्नाइट, प्राकृतिक गैस, पेट्रोलियम (कच्चा)।
    • धात्विक खनिज: बॉक्साइट, क्रोमाइट, तॉंबा अयस्क, सोना, लौह अयस्क, सीसा (सांद्र), जस्ता (सांद्र), मैंगनीज अयस्क, चाँदी, टिन (सांद्र), टंगस्टन (सांद्र)।
    • गैर-धात्विक खनिज: अगेट, अंडालूसाइट, एपेटाइट, एस्बेस्टस, बॉल क्ले, बेराइट्स, कैल्साइट, चॉक, क्ले, कोरंडम, कैलकेरियस सैंड, डायमंड, डायस्पोर, डोलोमाइट, कानाइट, लेटराइट, लाइमस्टोन(चूना पत्थर), लाइमस्टोन कंकर, लाइम शेल, मैग्नेसाइट, माइका (क्रूड), गेरू, पायराइट्स, पाइरोफिलाइट, फॉस्फोराइट, क्वार्ट्ज, अशुद्ध क्वार्ट्ज, क्वार्टजाइट, फुकसाइट क्वार्टजाइट, सिलिका सैंड, नमक (रॉक), नमक (वाष्पीकृत), शेल, स्लेट, स्टीटाइट, सिलिमेनाइट, वर्मीक्यूलाइट, वोलास्टोनाइट।
    • लघु खनिज पदार्थों में बेंटोनाइट, बोल्डर, ब्रिक अर्थ, बिल्डिंग स्टोन्स, कैल्सेडनी या कोरंडम, फुलर्स अर्थ, बजरी, लाइम स्टोन, ड्यूनाइट, फेल्सपर, फायर क्ले, फेलसाइट, फ्लोराइट (ग्रेडेड), फ्लोराइट (कॉन्सेंट्रेट्स), जिप्सम, गार्नेट (एब्रेसिव्स) गार्नेट (जेम), ग्रेफाइट रन-ऑन-माइन, जैस्पर, काओलिन, मार्बल, मुर्रम, साधारण मृदा, साधारण रेत आदि शामिल हैं। 
    • साधारण मृदा, कंकड़, क्वार्टजाइट और सैंड स्टोन, रोड मेटल, साल्ट पेट्रे, शेल, शिंगल, स्लेट, क्रोमाइट, कानाइट और सिलिमेनाइट प्रमुख खनिज हैं, जबकि बेंटोनाइट एक गौण खनिज है। अतः विकल्प (d) सही है।

93. महासागर औसत तापमान (Ocean Mean Temperature/OMT) के संदर्भ में, निम्नलिखित में से कौन-सा/से सही है/हैं?

  1. OMT को 26ºC समताप रेखा की गहराई तक मापा जाता है जो जनवरी-मार्च में हिंद महासागर के दक्षिण-पश्चिम में 129 मीटर पर होती है।
  2. OMT जो जनवरी-मार्च में एकत्रित किया जाता है उसे यह निर्धारित करने के लिये प्रयोग किया जा सकता है कि मानसून में वर्षा की मात्रा एक निश्चित दीर्घकालीन औसत वर्षा से कम होगी या अधिक।

नीचे दिये गए कूट का प्रयोग कर सही उत्तर चुनिये-

(a) केवल 1

(b) केवल 2

(c) 1 और 2 दोनों

(d) न तो 1, न ही 2

उत्तर : (b) 

व्याख्या

  • पुणे के भारतीय उष्णकटिबंधीय मौसम विज्ञान संस्थान (IITM) के वैज्ञानिकों ने यह खोजा है कि सागरीय सतह के तापमान (SST) की तुलना में OMT में भारतीय ग्रीष्मकालीन मानसून की भविष्यवाणी करने की बेहतर क्षमता है।
  • SST, शीर्ष महासागरीय सतह के कुछ मिलीमीटर तक ही सीमित है, जो काफी हद तक तीव्र वायु, वाष्पीकरण या घने बादलों से प्रभावित है। इसके विपरीत OMT जिसे 26 डिग्री सेल्सियस समताप की गहराई तक मापा जाता है, जो अधिक स्थिर और सुसंगत है, जिसका क्षेत्रीय विस्तार भी कम है।
  • 26 डिग्री सेल्सियस समताप को 50-100 मीटर की भिन्न गहराई तक मापा जाता है। जनवरी-मार्च के दौरान, दक्षिण-पश्चिमी हिंद महासागर में औसत 26ºC समताप की गहराई 59 मीटर तक मापी जाती है। अतः कथन 1 सही नहीं है।
  • OMT के साथ, सफल भविष्यवाणी के अतिरिक्त मानसून वर्षा की मात्रा लंबी अवधि के औसत से अधिक या कम होगी जो दक्षिण-पश्चिम मानसून के आने से दो माह पूर्व इस बारे में जानकारी अप्रैल की शुरुआत में उपलब्ध कराएगी क्योंकि OMT का विश्लेषण जनवरी से मार्च की अवधि के दौरान महासागरीय तापीय ऊर्जा को मापकर किया जाता है। अतः कथन 2 सही है।
  • अतः विकल्प (b) सही है।

94. भारत में रासायनिक उर्वरकों के संदर्भ में, निम्नलिखित कथनों पर विचार कीजिये-

  1. वर्तमान में रासायनिक उर्वरकों का खुदरा मूल्य बाज़ार-संचालित है और यह सरकार द्वारा नियंत्रित नहीं है।
  2. अमोनिया जो यूरिया बनाने में काम आता है, वह प्राकृतिक गैस से उत्पन्न होता है।
  3. सल्फर, जो फॉस्फोरिक अम्ल उर्वरक के लिये कच्चा माल है, वह तेल शोधन कारखानों का उपोत्पाद है।

उपर्युक्त कथनों में से कौन-सा/से सही है/हैं?

(a) केवल 1 

(b) केवल 2 और 3

(c) केवल 2

(d) 1, 2 और 3

उत्तर : (b) 

व्याख्या

  • भारत सरकार उर्वरकों पर सब्सिडी इसलिये देती है जिससे यह सुनिश्चित किया जा सके कि किसानों को उर्वरक आसानी से उपलब्ध हो ताकि देश कृषि उत्पादन में आत्मनिर्भर बना रहे। इसे वृहत् स्तर पर उर्वरकों की कीमत और उत्पादन की मात्रा को नियंत्रित करके हासिल किया गया है। अतः कथन 1 सही नहीं है।
  • अमोनिया (NH3) को प्राकृतिक गैस से संश्लेषित किया जाता है। इस प्रक्रिया में प्राकृतिक गैस के अणु कार्बन और हाइड्रोजन के रूप में अपचयित हो जाते हैं। हाइड्रोजन को तब शुद्ध किया जाता है जब अमोनिया के उत्पादन के लिये नाइट्रोजन के साथ प्रतिक्रिया की जाती है। सिंथेटिक अमोनिया का उपयोग उर्वरक के रूप में किया जाता है, जिसे संश्लेषण के बाद या तो प्रत्यक्ष अमोनिया के रूप में या अप्रत्यक्ष रूप से यूरिया के रूप में अमोनियम नाइट्रेट और मोनोअमोनियम या डायमोनियम फॉस्फेट के रूप प्रयोग किया जाता है। अतः कथन 2 सही है।
  • सल्फर तेल शोधन और गैस प्रसंस्करण का एक प्रमुख उप-उत्पाद है। अधिकांश कच्चे तेल में कुछ सल्फर की मात्रा होती है, जिनमें से अधिकांश को परिष्कृत उत्पादों से सख्त सल्फर सामग्री की पूर्ति के लिये शोधन प्रक्रिया के दौरान हटा दिया जाता है। ऐसा आमतौर पर हाइड्रोट्रीटिंग के माध्यम से किया जाता है, इसके परिणामस्वरूप H2S गैस का उत्पादन होता है, जो मौलिक सल्फर में परिवर्तित हो जाता है। सल्फर का खनन भूमिगत, प्राकृतिक रूप से पाए जाने वाले निक्षेपों से भी किया जा सकता है, लेकिन यह तेल और गैस की तुलना में अधिक महँगा है, हालाँकि इसे काफी हद तक बंद कर दिया गया है। मोनोअमोनियम फॉस्फेट (MAP) और डायअमोनियम फॉस्फेट (DAP) दोनों के उत्पादन में सल्फ्यूरिक अम्ल का उपयोग किया जाता है। अतः कथन 3 सही है।

अतः विकल्प (b) सही है।

https://www.drishtiias.com/hindi/daily-updates/daily-news-analysis/high-level-of-ammonia-in-yamuna

95. भारत के ‘मरु राष्ट्रीय उद्यान’ के संदर्भ में, निम्नलिखित में से कौन-से कथन सही हैं? 

  1. यह दो ज़िलों में विस्तृत है।
  2. उद्यान के अंदर कोई मानव वास स्थल नहीं है।
  3. यह ‘ग्रेट इंडियन बस्टर्ड’ के प्राकृतिक आवासों में से एक है।

नीचे दिये गए कूट का प्रयोग कर सही उत्तर चुनिये-

(a) केवल 1 और 2

(b) केवल 2 और 3

(c) केवल 1 और 3

(d) 1, 2 और 3

उत्तर : (c) 

व्याख्या

  • मरु राष्ट्रीय उद्यान (DNP), 3162 किमी² का क्षेत्र है, जो राजस्थान में जैसलमेर और बाड़मेर ज़िलों में विस्तृत है। यह क्षेत्र देश के अति न्यून वर्षा क्षेत्र (<100 मिमी) के अत्यधिक गर्म एवं शुष्क क्षेत्र में आता है। DNB को वर्ष 1980 में सीमांकित और अधिसूचित किया गया था। अतः कथन 1 सही है।
  • थार मरुस्थल विश्व की सबसे घनी आबादी वाला मरुस्थल है जिसका औसत घनत्व 83 व्यक्ति/किमी² है। हालाँकि, DNP में मानव आबादी की बसाहट कम (4-5 व्यक्ति प्रति किमी²) है। उद्यान के भीतर लगभग 70 गाँव और बस्तियाँ या ढाणी मौजूद हैं। अतः कथन 2 सही नहीं है।
  • थार मरुस्थल वनस्पतियों और जीवों की विभिन्न प्रजातियों को आश्रय देता है। यह एकमात्र स्थान है जहाँ राजस्थान राज्य पक्षी (ग्रेट इंडियन बस्टर्ड) पाए जाते हैं। अतः कथन 3 सही है।
  • अतः विकल्प (c) सही है।

https://www.drishtiias.com/hindi/daily-updates/daily-news-analysis/great-indian-bustard

96. सियाचिन हिमनद कहाँ स्थित है?

(a) अक्साई चिन के पूर्व में

(b) लेह के पूर्व में

(c) गिलगिट के उत्तर में

(d) नुब्रा घाटी के उत्तर में

उत्तर : (d) 

व्याख्या

  • NJ9842 बिंदु के उत्तर-पूर्व में भारत और पाकिस्तान के बीच नियंत्रण रेखा समाप्त होती है, जो सियाचिन ग्लेशियर हिमालय में पूर्वी काराकोरम श्रेणी में स्थित है।
  • इसे ध्रुवीय और उपध्रुवीय क्षेत्रों के बाहर सबसे बड़ा ग्लेशियर होने का गौरव प्राप्त है।
  • यह अक्साई चिन के पश्चिम में, नुब्रा घाटी के उत्तर में और गिलगित के लगभग पूर्व में स्थित है। अतः विकल्प (d) सही है।

https://www.drishtiias.com/hindi/daily-updates/daily-news-analysis/no-more-waste-mounds-on-siachen-glacier

https://www.drishtiias.com/hindi/daily-updates/daily-news-analysis/new-map-of-pakistan

97. भारत के इतिहास के संदर्भ में, निम्नलिखित युग्मों पर विचार कीजिये-

प्रसिद्ध स्थल          वर्तमान राज्य

  1. भीलसा       –   मध्य प्रदेश
  2. द्वारसमुद्र     –   महाराष्ट्र
  3. गिरिनगर        गुजरात
  4. स्थानेश्वर      –   उत्तर प्रदेश

उपर्युक्त में से कौन-से युग्म सही सुमेलित हैं?

(a) केवल 1 और 3 

(b) केवल 1 और 4

(c) केवल 2 और 3

(d) केवल 2 और 4

उत्तर : (a) 

व्याख्या

  • विदिशा (जिसे पहले भेलसा के नाम से और प्राचीन काल में बेसनगर के नाम से जाना जाता था) मध्य प्रदेश राज्य का एक शहर है। यह राज्य की राजधानी भोपाल से लगभग 60 किमी उत्तर पूर्व में स्थित है। अतः युग्म 1 सही सुमेलित है।
  • हलेबिडु, जिसे पूर्व में दोरासमुद्र या द्वारसमुद्र के नाम से जाना जाता था, प्राचीन होयसल राजधानी में अलंकृत होयसलेश्वर और केदारेश्वर मंदिर स्थित हैं, जो कर्नाटक राज्य के पर्यटक आकर्षणों में से एक है। अतः युग्म 2 सही सुमेलित नहीं है।
  • गिरनार, जिसे गिरिनगर (पहाड़ी पर शहर) या रेवतक पर्वत के नाम से भी जाना जाता है, गुजरात के जूनागढ़ ज़िले में पहाड़ों का एक समूह है। यह स्थान जैनियों के लिये पवित्र है क्योंकि यह वह स्थान है जहाँ भगवान नेमिनाथ को मोक्ष प्राप्त हुआ था। अतः युग्म 3 सही सुमेलित है।
  • थानेसर या स्थानेश्वर एक ऐतिहासिक शहर, जो वर्तमान हरियाणा में नव निर्मित कुरुक्षेत्र शहर के निकट स्थित है। अतः युग्म 4 सही सुमेलित नहीं है।
  • अतः विकल्प (a) सही है।

98. निम्नलिखित कथनों पर विचार कीजिये-

  1. केंद्रीय भूमि जल प्राधिकरण (CGWA) ने भारत के 36% ज़िलों को ‘‘अतिशोषित’’ (Overexploited) अथवा ‘‘संकटपूर्ण’’ (Critical) वर्गीकृत किया हुआ है।
  2. CGWA का निर्माण ‘पर्यावरण (संरक्षण) अधिनियम’ के अंतर्गत हुआ।
  3. विश्व में भूजल सिंचाई के अंतर्गत सबसे अधिक क्षेत्र भारत में है।

उपर्युक्त कथनों में से कौन-सा/से सही है/हैं?

(a) केवल 1 

(b) केवल 2 और 3

(c) केवल 2

(d) केवल 1 और 3

उत्तर : (b) 

व्याख्या

  • भूजल स्तर के आधार पर देश भर के क्षेत्रों को तीन श्रेणियों में बाँटा गया है: अति-दोहन वाले क्षेत्र, गंभीर और कम गंभीर। अति-दोहन वाले क्षेत्रों में भूजल की पुनर्भरण दर की तुलना में अधिक दर (100 प्रतिशत से अधिक) से भूजल का दोहन किया जा रहा है। गंभीर स्थिति में भूजल दोहन, पुनर्भरण का 90-100 प्रतिशत है और कम गंभीर स्थिति में भूजल दोहन पुनर्भरण के सापेक्ष 70-90 प्रतिशत है।
  • भारत के गतिशील भूजल संसाधनों पर राष्ट्रीय संकलन रिपोर्ट 2017 के अनुसार देश में कुल 6881 मूल्यांकन इकाइयों (ब्लॉकों/मंडलों/ताल्लुकों) में से विभिन्न राज्यों में 1186 इकाइयों (17%) को अतिशोषित, 313 इकाइयों ( 5%) को गंभीर और 972 इकाइयों (14%) को कम गंभीर के रूप में वर्गीकृत किया गया है। अतः कथन 1 सही नहीं है।
  • नोट: भारत के गतिशील भूजल संसाधनों पर राष्ट्रीय संकलन, 2020 के अनुसार; देश में कुल 6965 मूल्यांकन इकाइयों (ब्लॉक/मंडल/ताल्लुकों) में से 16% को 'अति-शोषित, 4% को गंभीर, 15% को कम गंभीर और 64%' को 'सुरक्षित' के रूप में वर्गीकृत किया गया है। इनके अलावा 97 (1%) मूल्यांकन इकाइयाँ ऐसी हैं, जिन्हें खारे (Saline) के रूप में वर्गीकृत किया गया है।
  • केंद्रीय भूजल प्राधिकरण (CGWA) का गठन पर्यावरण (संरक्षण) अधिनियम, 1986 की धारा 3 (3) के तहत भूजल संसाधनों के विकास और प्रबंधन को विनियमित एवं नियंत्रित करने के लिये किया गया था। अत: कथन 2 सही है।
  • संयुक्त राष्ट्र के खाद्य और कृषि संगठन (FAO) की रिपोर्ट के अनुसार, भारत (39 मिलियन हेक्टेयर), चीन (19 मिलियन हेक्टेयर) और संयुक्त राज्य अमेरिका (17 मिलियन हेक्टेयर) भूजल से सिंचाई करने वाले सबसे बड़े देश हैं। अत: कथन 3 सही है।
  • अतः विकल्प (b) सही है।

99. निम्नलिखित कथनों पर विचार कीजिये-

  1. जेट प्रवाह केवल उत्तरी गोलार्द्ध में होते हैं।
  2. केवल कुछ चक्रवात ही केंद्र में वाताक्षि उत्पन्न करते हैं।
  3. चक्रवाती की वाताक्षि के अंदर का तापमान आसपास के तापमान से लगभग 10º C कम होता है।

उपर्युक्त कथनों में से कौन-सा/से सही है/हैं?

(a) केवल 1 

(b) केवल 2 और 3

(c) केवल 2 

(d) केवल 1 और 3

उत्तर : (c) 

व्याख्या

  • जेट स्ट्रीम एक भूस्थैतिक पवन है जो क्षोभमंडल की ऊपरी परतों में पश्चिम से पूर्व की ओर 20,000-50,000 फीट की ऊंँचाई पर क्षैतिज रूप से बहती है। जेट स्ट्रीम विभिन्न तापमान वाली वायुराशियों के मिलने पर विकसित होती है। अतः सतह का तापमान निर्धारित करती है कि जेट स्ट्रीम कहाँ बनेगी। तापमान में जितना अधिक अंतर होता है जेट स्ट्रीम का वेग उतना ही तीव्र होता है। जेट धाराएँ दोनों गोलार्द्धों में 20° अक्षांश से ध्रुवों तक फैली हुई हैं। अतः कथन 1 सही नहीं है।
  • चक्रवात दो प्रकार के होते हैं, उष्णकटिबंधीय चक्रवात और शीतोष्ण चक्रवात। उष्णकटिबंधीय चक्रवात के केंद्र को 'आंँख' के रूप में जाना जाता है, जहांँ केंद्र में हवा शांत होती है और वर्षा नहीं होती है। हालांँकि समशीतोष्ण चक्रवात में एक भी स्थान ऐसा नहीं है जहांँ हवाएंँ और बारिश नहीं होती है, अतः शीतोष्ण चक्रवात में आंँख नहीं पाई जाती है। अत: कथन 2 सही है।
  • सबसे गर्म तापमान आंँख/केंद्र में ही पाया जाता है, न कि आईवॉल बादलों में जहांँ तापमान उत्पन्न होता है। हवा केवल वहीं संतृप्त होती है जहांँ यह ऊर्ध्वाधर गति से ऊपर उठती है। आंँख के अंदर तापमान 28 डिग्री सेल्सियस से अधिक और ओस बिंदु 0 डिग्री सेल्सियस से कम होता है। ये गर्म व शुष्क स्थितियांँ अत्यंत तीव्र उष्णकटिबंधीय चक्रवातों की आंँख के लिये विशिष्ट हैं। अत: कथन 3 सही नहीं है। अतः विकल्प (C) सही है।

100. निम्नलिखित बाघ आरक्षित क्षेत्राें में ‘‘क्रांतिक बाघ आवास (Critical Tiger Habitat)’’ के अंतर्गत सबसे बड़ा क्षेत्र किसके पास है?

(a) कॉर्बेट

(b) रणथम्बौर

(c) नागार्जुनसागर-श्रीसैलम

(d) सुंदरबन

उत्तर : (c) 

व्याख्या

  • “क्रांतिक बाघ आवास (Critical Tiger Habitat), जिसे टाइगर रिज़र्व कोर क्षेत्र भी कहा जाता है, की पहचान वन्य जीवन संरक्षण अधिनियम (WLP), 1972 के अंतर्गत की गई है। वैज्ञानिक प्रमाणों के आधार पर, अनुसूचित जनजाति या ऐसे अन्य वनवासियों के अधिकारों को प्रभावित किये बिना ऐसे क्षेत्रों को बाघ संरक्षण के लिये सुरक्षित रखा जाना आवश्यक है। CTH की अधिसूचना राज्य सरकार द्वारा उद्देश्य के लिये गठित विशेषज्ञ समिति के परामर्श से की जाती है।
  • कोर/क्रांतिक बाघ आवास क्षेत्र:
    • कॉर्बेट (उत्तराखंड): 821.99 वर्ग किमी
    • रणथंभौर (राजस्थान): 1113.36 वर्ग किमी
    • सुंदरवन (पश्चिम बंगाल): 1699.62 वर्ग किमी
    • नागार्जुनसागर श्रीशैलम (आंध्र प्रदेश का हिस्सा): 2595.72 वर्ग किमी
  • अतः विकल्प (c) सही है।

https://www.drishtiias.com/hindi/daily-updates/prelims-facts/prelims-facts-19-november-2019

close
एसएमएस अलर्ट
Share Page
images-2
images-2
× Snow